Você está na página 1de 128

Matemática

Geometria de
posição e métrica
1
SISTEMA COC DE ENSINO
Direção-Geral: Sandro Bonás
Direção Pedagógica: Zelci C. de Oliveira
Direção Editorial: Roger Trimer
Gerência pedagógica: Luiz Fernando Duarte
Gerência Editorial: Osvaldo Govone
Gerência Operacional: Danilo Maurin
Gerência de Relacionamento: Danilo Lippi
Ouvidoria: Regina Gimenes
Conselho Editorial: José Tadeu B.
Terra, Luiz Fernando Duarte, Osvaldo
Govone e Zelci C. de Oliveira
PRODUÇÃO EDITORIAL
Autoria: Frederico R. F. do Amaral Braga
Editoria: Clayton Furukawa, José F. Rufato,
Marina A. Barreto e Paulo S. Adami
Coordenação editorial: Luzia H. Fávero F. López
Assistente Editorial: George R. Baldim
Projeto gráfico e direção de
arte: Matheus C. Sisdeli
Preparação de originais: Marisa A. dos Santos
e Silva e Sebastião S. Rodrigues Neto
Iconografia e licenciamento de texto:
Cristian N. Zaramella, Marcela Pelizaro
e Paula de Oliveira Quirino.
Diagramação: BFS bureau digital
Ilustração: BFS bureau digital
Revisão: Flávia P. Cruz, Flávio R. Santos,
José S. Lara, Leda G. de Almeida e
Maria Cecília R. D. B. Ribeiro.
Capa: LABCOM comunicação total
Fechamento: BFS bureau digital

Rua General Celso de Mello Rezende, 301 – Tel.: (16) 3238·6300


CEP 14095-270 – Lagoinha – Ribeirão Preto-SP
www.sistemacoc.com.br
CAPÍTULO 01 GEOMETRIA ESPACIAL DE POSIÇÃO 7
Sumário 1.
2.
Postulados
Posições relativas de duas retas
7
9
3. Determinação de planos 10
4. Posições relativas de reta e plano 12
5. Posições relativas de dois planos 13
6. Perpendicularismo 15
1. Superfície poliédrica 20
2. Poliedro convexo 20
3. Lema do teorema de Euler 20
4. Teorema de Euler 21
5. Propriedade 22
6. Poliedros de Platão 23
7. Poliedros regulares 25

CAPÍTULO 03 PRISMAS 27
1. Definição e elementos 27
2. Nomenclatura e classificação 27
3. Cubo 28
4. Paralelepípedos 28
5. Princípio de Cavalieri 34
6. Volume de um prisma qualquer 34
7. Área e volume de prismas regulares 35

CAPÍTULO 04 PIRÂMIDES 40
1. Generalidades 40
2. Área lateral e área total 41
3. Volume 41
4. Sólidos especiais 43

CAPÍTULO 05 CILINDROS 50
1. Generalidades 50
2. Áreas lateral e total 51
3. Volume 53

CAPÍTULO 06 CONES 58
1. Generalidades 58
2. Áreas lateral e total 59
3. Volume 60
4. Cilindro e cone equiláteros 63
CAPÍTULO 07 ESFERAS 66
1. Generalidades 66
2. Volume 67
3. Área da superfície esférica 68
4. Partes da esfera 68

CAPÍTULO 08 SÓLIDOS SEMELHANTES 72


1. Definição 72
2. Consequências 72

EXERCÍCIOS PROPOSTOS
Capítulo 01 81
Capítulo 02 90
Capítulo 03 93
Capítulo 04 100
Capítulo 05 108
Capítulo 06 111
Capítulo 07 115
Capítulo 08 119

GABARITO 124
Teoria
Geometria de posição e métrica Matemática

CAPÍTULO 01 GEOMETRIA ESPACIAL DE POSIÇÃO


As noções de ponto, reta e plano são primiti- Observação
vas, isto é, não definimos estes elementos da Os pontos A, B e C pertencem a uma mesma
geometria; no entanto, sabemos intuitivamen- reta, portanto, eles são colineares.
te o que são e como são.
• Num plano, bem como fora dele, exis-
Para estudarmos a geometria de posição no tem infinitos pontos.
espaço, é preciso que façamos a seguinte de-
D
finição: E

Espaço é o conjunto de todos os pontos. A C

Dessa forma, estudaremos neste capítulo as B


posições relativas de pontos, retas e planos no
espaço. F
G
1. Postulados Observação
A demonstração de uma propriedade normal- Os planos A, B e C pertencem a um mesmo
mente é feita com base em outras proprie- plano, portanto, eles são coplanares.
dades, já demonstradas anteriormente. No P2) Postulados da determinação
entanto, as primeiras propriedades de uma
teoria, uma vez que não há outras para apoiar • Dois pontos distintos determinam
as suas demonstrações, são simplesmente uma única reta.
“aceitas” como verdadeiras. B
Essas propriedades são chamadas de postu- A
lados.
São postulados da geometria euclidiana: Observações
P1) Postulados da existência 01. Dizemos que dois entes geométricos
• Existem ponto, reta e plano. são distintos quando eles não coinci-
dem.
P
Ponto P 02. Dizemos que uma reta fica determina-
da quando ela for única nas condições
r especificadas.
• Três pontos não colineares determi-
Reta r nam um único plano.
PV-13-31

A C

Plano α α B
α
P3) Postulado da inclusão
• Numa reta, bem como fora dela, exis-
tem infinitos pontos. • Uma reta que possui dois pontos dis-
tintos em um plano está contida nesse
E plano.
D
C B r
B
A A
F α

7
Matemática Geometria de posição e métrica

A ∈α  1º caso: P ∈ r
  P
B ∈α  ⇒ r = AB ⊂ α
r t
A ≠ B 
P4) Postulados da divisão
2º caso: P ∉ r
• Um ponto de uma reta divide-a em P
duas regiões denominadas semirre- t
tas. O ponto é a origem das semirretas
que são chamadas opostas.
r
A O B
Observação
 
OA e OB são semirretas opostas de origem O. No 2º caso, t e r são chamadas paralelas distin-
tas e, no 1º caso, paralelas coincidentes
• Uma reta de um plano divide-o em
Exemplos
duas regiões denominadas semipla-
nos. A reta é a origem dos semiplanos, 01. Classifique como verdadeiras ou falsas as
que são chamados opostos. afirmações.
r a. Dois pontos determinam uma reta.
b. Três pontos distintos determinam um
plano.
c. Três pontos são sempre coplanares.
d. Se o ponto P de uma reta r pertence a um
plano a, então a reta r está contida em a.
• Um plano divide o espaço em duas re-
giões denominadas semiespaços, que Resposta
são chamados opostos. a. F, pois os pontos podem ser coincidentes.
b. F, pois os pontos podem ser colineares.
c. V, pois o significado de “são sempre
coplanares” é: existe um plano que os
contém. Vamos analisar alguns casos:
α – Os três pontos são coincidentes. Nes-
se caso existem infinitos planos que os
contêm, logo existe um.
P5) Postulado da intersecção – Os três pontos são distintos, porém
Se dois planos distintos têm um ponto em colineares. Também nesse caso existem
PV-13-31

comum, então há uma única reta em comum infinitos.


passando por esse ponto. – Os três pontos são não colineares.
Nesse caso existe um único.
α r
d. F
r
β

P6) Postulado das paralelas (Euclides) α


Dado um ponto P e uma reta r, existe e é única
a reta que passa por P e é paralela a r. Se r ∩ a = {P}
r⊄a

8
Geometria de posição e métrica Matemática

02. A figura mostra dois planos, a e b, que se IV. Retas reversas


interceptam na reta t. Sendo r uma reta conti- Duas retas são reversas quando não forem co-
da no plano a, e os pontos A e B pertencentes planares.
a semiespaços distintos em relação a b, A ∈ a
e P ∈ b, existe algo de errado na figura; descu- b
bra o que é.

r a
α A t
α
β P
B
∃ a|a ⊂ a e b ⊄ a ⇔ a e b reversas

Resposta B. Ângulo entre duas retas reversas


O ponto P deveria estar na reta t. Consideremos duas retas reversas r e s.
Seja a um plano que contém s e intercepta r
2. Posições relativas de duas retas num ponto P. Traçando por P a reta s’ paralela
A. Definições a s, dizemos que o ângulo entre r e s é o ângulo
I. Retas concorrentes entre as retas concorrentes r e s’.
Duas retas são concorrentes quando têm um r
único ponto em comum.
p s' s
a
P α
α
b

a ∩ b = {P} ⇔ a e b concorrentes
II. Retas paralelas distintas Observação
Duas retas são paralelas distintas, quando forem Geralmente, quando nos referimos a um ângu-
coplanares e não tiverem ponto em comum. lo formado por duas retas, tomamos o ângulo
agudo que elas formam.
a
b
C. Ângulo reto
I. Retas perpendiculares
PV-13-31

α
a∩b={ } Duas retas são perpendiculares quando forem
⇔ a e b paralelas distintas concorrentes e formarem ângulo reto.
∃ a/a ⊂ a e b ⊂ a
Indicamos por r ⊥ s.
III. Retas paralelas coincidentes
Duas retas são paralelas coincidentes quando r
tiverem todos os seus pontos em comum. s
a b
α
r⊥s
α

a ∩ b = a = b ⇔ a e b paralelas coincidentes

9
Matemática Geometria de posição e métrica

II. Retas ortogonais Resposta


Duas retas são ortogonais quando forem re- a. V
versas e formarem ângulo reto. b. V
Indicamos por r ⊥ s. c. F
r d. V
s' s e. V

3. Determinação de planos
A. Plano determinado por três
pontos não colineares
r⊥s Consideremos três pontos, A, B e C, não coli-
Exemplos neares.
03. Classifique como verdadeiras (V) ou falsas Existe uma infinidade de planos que passam por
(F) as afirmações a seguir. 
A e B, entre eles os planos que contêm a reta AB .
a. Duas retas que não têm ponto em co-
mum são paralelas.
b. Duas retas que têm um ponto em co- A
mum são concorrentes.
c. Se a medida do ângulo entre duas re- C
tas é 90° elas são perpendiculares ou B
reversas.
Resposta
a. F, pois podem ser reversas. No entanto, apenas um desses planos passa
b. F, pois podem ser paralelas coinciden- também pelo ponto C. Assim, dizemos que:
tes. Para tornar a sentença verdadeira Três pontos não colineares
basta dizer que o ponto é único. determinam um plano.
c. V
Observação
04. Dada a pirâmide, associe (V) ou (F) a cada O plano determinado pelos pontos A, B e C é
afirmação. indicado por pl(ABC).
V
B. Plano determinado por uma
reta e um ponto fora dela
PV-13-31

Consideremos uma reta r e um ponto P que


C não pertence a r.
D
Sejam A e B dois pontos distintos de r. Os pon-
A B tos A, B e P, por não serem colineares, deter-
  minam um plano, e a reta r está contida nesse
a. ( ) VD e BC são reversas. plano (postulado da inclusão).
  
b. ( ) AB, BC e AD são coplanares. A
   
c. ( ) VA , VB, VC e VD são coplanares. P
  B
d. ( ) AD e BC podem ser paralelas. r
  
e. ( ) AB,BC e VB são concorrentes em B. Assim, dizemos que:

10
Geometria de posição e métrica Matemática

Uma reta e um ponto fora Observação


dela determinam um plano. O plano determinado por r e s é indicado por
pl (r, s).
Observação
O plano determinado pela reta r e pelo ponto
E. Quadriláteros plano e reverso
P é indicado por pl (∅ r, P ∅). Consideremos quatro pontos, A, B, C e D, de
modo que não existam três colineares.
C. Plano determinado por duas Em relação ao plano pl (ABC) determinado pe-
retas paralelas disti ntas los pontos A, B e C, o ponto D pode ou não
Consideremos duas retas, r e s, paralelas dis- pertencer a ele; assim, temos:
tintas. A própria definição do paralelismo exi- 01. D ∈ pl (ABC)
ge que elas estejam num mesmo plano (copla- Os pontos A, B, C e D são vértices de um qua-
nares). Esse plano é o único que contém duas drilátero que chamamos quadrilátero plano.
retas.
A D
r
s
C
α B

02. D ∉ pl (ABC)
Assim, dizemos que:
Os pontos A, B, C e D são vértices de um qua-
Duas retas paralelas distintas drilátero que chamamos quadrilátero reverso.
determinam um plano. D

Observação
O plano determinado por r e s é indicado por A
pl (r, s ∅). C
B
D. Plano determinado por
duas retas concorrentes Observação
Consideremos duas retas concorrentes, r e s, As diagonais de um quadrilátero plano estão
que se interceptam no ponto P. em retas concorrentes enquanto que as dia-
gonais de um quadrilátero reverso estão em
Seja A um ponto de r distinto de P e B um pon- retas reversas.
to de s também distinto de P.
Exemplo
Os pontos A, B e P não são colineares, então
05. Classifique as afirmações abaixo em verda-
PV-13-31

eles determinam um plano a, e as retas r e s


estão contidas nesse plano, pois têm dois pon- deiras (V) ou falsas (F).
tos distintos pertencentes a ele. a. Uma reta e um ponto sempre determi-
r nam um plano.
A
P b. Duas retas quaisquer sempre determi-
B nam um plano.
s
c. Três retas distintas, paralelas duas a
duas, determinam um ou três planos.
Este plano é o único que contém simultanea-
d. três retas concorrentes quaisquer são
mente r e s. Assim:
sempre coplanares.
Duas retas concorrentes determi- Respostas
nam um plano.
a. F, pois o ponto pode pertencer à reta.

11
Matemática Geometria de posição e métrica

b. F, retas paralelas coincidentes e retas C. Reta paralela ao plano


reversas não determinam planos.
Uma reta é paralela a um plano quando eles
c. V não têm ponto em comum.
s r
t
r

α ou α
s
t
r
r ∩ a = ∅ ⇔ r//a
Importante!
É fácil notarmos que, se uma reta r é paralela a
d. F um plano a, ela não tem ponto em comum com
t a e, desse modo, não tem ponto em comum com
as retas contidas em a. Assim, concluímos que:
s
Se uma reta é paralela a um plano, então ela é
r paralela ou reversa a qualquer reta do plano.
r
4. Posições relativas de reta e plano
t
A. Reta conti da no plano u
Uma reta está contida em um plano quando
ela tem dois pontos distintos pertencentes ao v
plano (postulado da inclusão).
D. Teorema do paralelismo
A B r entre reta e plano
Se uma reta não está contida num plano e é
α paralela a uma reta do plano, então ela é pa-
A≠B  ralela ao plano.

A ∈r e A ∈α  ⇔ r ⊂ α ⇔ r ∩ α = r r ⊄ α

B ∈r e B ∈α  Hipótese: s ⊂ α Tese: {r // α
r // s

B. Reta concorrente com o plano r
Uma reta e um plano são concorrentes ou se-
PV-13-31

cantes quando têm um único ponto em comum.


s

α
α
P
Demonstração
Consideremos um plano b determinado por r e s.
r
r e α
∃|P|r ∩ α = {P} ⇔  s
concorrentes
α
Observação
β
O ponto P é chamado “traço” da reta no plano.

12
Geometria de posição e métrica Matemática

Todos os pontos em comum de a e b estão si- B. Planos paralelos coincidentes


tuados na reta s. Assim, se existir algum ponto
Dois planos são paralelos coincidentes se têm
em comum entre r e a, esse ponto tem de per-
todos os pontos em comum.
tencer à reta s. Como r e s não têm ponto em
comum, pois são paralelas distintas, r e a não
terão ponto em comum, então r é paralela a a.
α≡β
Exemplos
06. Classifique como verdadeiras (V) ou falsas a≡b⇔a∩b=a=b
(F) as afirmações abaixo.
a. Uma reta e um plano que têm um pon-
C. Planos paralelos distintos
to em comum são concorrentes. Dois planos são paralelos distintos quando
b. Duas retas paralelas a um plano são pa- não têm ponto em comum.
ralelas entre si.
c. Se uma reta é paralela a um plano a, en-
α
tão ela é paralela a todas as retas de a.
β
d. Se uma reta é concorrente com um pla-
no a, então ela é concorrente ou rever-
a ∩ b = ∅ ⇔ a // b
sa com as retas de a.
Resposta D. Paralelismo entre
a. F, pois a reta pode estar contida. planos: teorema 1
b. F Se dois planos são paralelos distintos, qual-
s quer reta de um deles é paralela ao outro.
r
α ≠ β

Hipótese: α // β Tese: {r // β
r ⊂ α

α r
r
c. F α
β
s
Demonstração
α
Vamos analisar as possíveis posições relativas
d. V de r e b e concluir a tese por exclusão.
PV-13-31

01. Sabemos que r ⊄ b, pois se r ⊂ b, a reta


5. Posições relativas de dois planos r seria comum a a e b e os planos a e b
A. Planos concorrentes não seriam paralelos distintos.
Dois planos são concorrentes ou secantes se 02. A reta r não é concorrente com b, pois,
têm uma única reta em comum. se isso acontecesse, existiria um ponto
P de r pertencente a b e, portanto, a
r e b não seriam paralelos distintos, já
α β que o ponto P seria comum a a(r ⊂ a)
e a b.
Assim, como r ⊄ b e r não é concorrente com
α e β b, r // b.
∃|r |α ∩ β = r ⇔ 
concorrentes

13
Matemática Geometria de posição e métrica

E. Paralelismo entre Demonstração


planos: teorema 2 Vamos analisar as possíveis posições relativas
Se dois planos são paralelos distintos, toda de a e b e concluir a tese por exclusão.
reta concorrente com um deles é concorrente 01. Sabemos que a ≡ b, pois se a ≡ b a reta
com o outro. r estaria contida em b e, portanto, não
α ≠ β seria paralela a b.

Hipótese: α // β 02. Os planos a e b não são secantes, pois,
r ∩ α = {P} caso isto ocorresse, teríamos:

∃| t|t = a ∩ b
r e β
Tese:  α
concorrentes r
t
r
P s

β
P
α
β T
Como r // b e s // b, temos que r // t e s // t, o
que contraria o postulado de Euclides.
Demonstração Assim, como a ≠ b, a e b não são concorren-
Vamos analisar as possíveis posições relativas tes, então a // b.
de r e b e concluir a tese por exclusão. Exemplo
01. Sabemos que r ⊄ b, pois se r ⊂ b o pon- 07. Classifique as afirmações como verdadei-
to P de r seria comum aos dois planos ras (V) ou falsas (F).
a e b e os planos não seriam paralelos
distintos. a. Dois planos distintos são secantes.
02. A reta r não é paralela a b, pois, se r//b b. Se um plano contém duas retas distin-
e r ∩ a = {P} e a//b, então r estaria con- tas e paralelas a um outro plano, então
tida em a, o que contraria a hipótese. esses planos são paralelos.
Assim, como r ⊄ b e r//b, só podemos ter de r c. Se dois planos são secantes, então exis-
e b são concorrentes. te uma reta de um deles reversa a uma
reta do outro.
F. Paralelismo entre planos: teorema 3 d. Se dois planos são paralelos, então
toda reta paralela a um deles é paralela
Se um plano contém duas retas concorrentes
ou está contida no outro.
PV-13-31

que são paralelas a um outro plano, então es-


ses planos também são paralelos. Resposta
a. F – Podem ser paralelos distintos.
r ⊂ α e r // β
 b. F α
Hipótese: s ⊂ α e s // β r
r ∩ s = {P} s

β
Tese: {α // β
r
r//β, s//β e
P
α não é paralelo a β.
s
α c. V
β d. V

14
Geometria de posição e métrica Matemática

6. Perpendicularismo r
T
A. Reta e plano perpendiculares
α
Uma reta r é perpendicular a um plano a P
quando ela é concorrente com o plano e é per- A X B
b
pendicular a todas as retas de a que passam a x
pelo seu traço no plano. T'
r
As retas a e b são mediatrizes de TT', pois, por
hipótese, ambas são perpendiculares a esse
c α segmento e, por construção, elas passam pelo
seu ponto médio (P).
b
P Então: AT = AT’ e BT = BT’
a
Nos triângulos ABT e ABT’, temos:
AT = AT ' 
r∩a=P   LLL
AB é comum ⇒ ∆ABT ' ⇒
b a ⊂ a | P ∈ a  ⇒ r ⊥ α 
 BT = BT ' 
r⊥a 
A partir dessa congruência, tiramos que
TÂX = T'ÂX; então, nos triângulos ATX e AT’X,
B. Teorema do perpendicularismo temos:
entre reta e plano AT = AT ' 
Se uma reta é perpendicular a duas retas con-  LAL
AX é comum ⇒ ∆ATX ≅ ∆AT ' X
correntes de um plano, então ela é perpendi-
cular ao plano. TAX  
 = T ' AX

a ⊂ α , b ⊂ α A partir dessa congruência, tiramos que

Hipótese: r ⊥ a, r ⊥ b XT = XT’; então, a reta x é mediatriz de TT', pois
a ∩ b = P passa por P, médio de TT' e por X, equidistante
 de T e T’. Assim, a reta r é perpendicular a x.
Tese: {r ⊥ α
Como x é uma reta qualquer de a passando
por P, e x ⊥ r, então r ⊥ a.
r
Importante!
α
P Uma reta é perpendicular a um plano quando
PV-13-31

b ela forma ângulo reto (perpendicular ou orto-


a gonal) com duas retas concorrentes do plano.
C. Planos perpendiculares
Demonstração
Dois planos são perpendiculares se um deles
Consideremos na reta r dois pontos T e T’ per- contém uma reta perpendicular ao outro.
tencentes a semiespaços opostos, de modo
que PT = PT’. Nas retas a e b, tomamos os pon- t
tos A e B quaisquer. α r t ⊂ α
Seja x uma reta qualquer de a que passa por P, ⇔α ⊥ β
 β
t⊥β
e X o ponto onde a reta AB intercepta x.

15
Matemática Geometria de posição e métrica

Exemplo Resposta
08. Classifique as afirmações como verdadei- a. F – A reta pode estar contida no plano.
ras (V) ou falsas (F). r s r//s
a. Se uma reta é perpendicular a duas re-
tas paralelas de um plano, então ela é t
perpendicular ao plano.
b. Se duas retas são perpendiculares a b. V
um mesmo plano, então elas são pa- c. F
ralelas. s
c. Se uma reta e um plano são paralelos, r r
então toda a reta perpendicular à reta
dada é paralela ao plano.
d. Por um ponto de um plano existe uma
única reta perpendicular ao plano r//α s .. r
dado. e s não é paralela a a.
d. V

EXERCÍCIOS RESOLVIDOS
01. 02. Fuvest-SP
H G a. Defina retas paralelas.
b. Sejam r e s retas paralelas e P um ponto
não pertencente ao plano rs. Prove que
F a reta t, intersecção dos planos (Pr) e
E
(Ps), é paralela a r e a s.
C Resolução
D
a. Duas retas são paralelas quando coincidem
ou quando são coplanares e não têm ponto
A B em comum.
b.
Observe no cubo representando na figura an- t
P
terior os triângulos ABC, ABG, ABF, ACG e ACF.
Qual deles é um triângulo equilátero?
PV-13-31

Resolução
s
O triângulo ACF, que tem como lados três dia- r α
gonais dos quadrados que são faces do cubo.
As retas t e s não são reversas, pois são copla-
nares (t ⊂ pl (Ps)), o mesmo acontecendo com
t e r.
As retas t e s não são concorrentes, pois se
isso acontecesse o ponto de intersecção de t
e s também pertenceria a r, e as retas r e s não
seriam paralelas. O mesmo acontece com as
retas t e r.
Assim: t // s e t // r.

16
Geometria de posição e métrica Matemática

03. Vunesp-SP b. são perpendiculares.


a. Definir quando uma reta e um plano c. são reversas.
são paralelos. d. são ortogonais.
b. Seja b uma reta não contida no plano e. são coplanares.
a, mas paralela a ele. Mostre que qual-
quer plano passando por b e encon- Resolução
trando a o faz segundo uma paralela à Como a e b são paralelos, r também será pa-
reta b. ralela a s.
Resolução Como podem ser paralelas coincidentes, a al-
a. Uma reta e um plano são paralelos quando ternativa correta é a E.
eles não têm ponto em comum. Resposta
b. b E
α 06. UEL-PR
t
Dados o plano a e um ponto P não pertencen-
te a a, pelo ponto P:
Sendo b um plano que contém a reta b e inter- a. passa apenas uma reta perpendicular
cepta a na reta t. a a.
As retas b e t são coplanares (b ∈ b e t ∈ b) e b. passam infinitas retas perpendiculares
não têm ponto comum, pois, se isso aconte- a a.
cesse, o ponto comum de b e t pertenceria a a, c. passa apenas uma reta paralela a a.
e b não seria paralela a a. d. passa apenas um plano perpendicular
Assim, b e t são paralelas. a a.
e. passam infinitos planos paralelos a a.
04. Fuvest-SP
Resolução
São dados um plano p, um ponto P do mesmo,
e uma reta r oblíqua a p que o fura num ponto a. V r
P
distinto de P. Mostre que existe uma única reta
por P, contida em p, e ortogonal a r.
Resolução
r
α

t
b. F
P
PV-13-31

π c. F s
P
t
Se r é oblíqua a p, só existe uma reta t no plano α
p perpendicular a r. Para que uma reta do pla-
no p seja ortogonal a r, ela deve ser paralela a
t, e pelo postulado de Euclides só existe uma
reta passando por P e paralela a t. d. F – Qualquer plano que contenha r será
perpendicular a a.
05. PUC-SP e. F – O único plano nessas condições é o
Os planos a e b são paralelos. A reta r é per- plano determinado por s e t.
pendicular a a e a reta s é perpendicular a b. Resposta
Pode-se concluir que r e s:
A
a. não têm ponto comum.

17
Matemática Geometria de posição e métrica

07. Fuvest-SP 08. UCSal-BA


São dados um ponto P, uma reta r e um plano Sejam o plano a e a reta r, paralela a a. Nessas
a. condições, é verdade que:
a. Descreva o processo para construir um a. toda reta paralela a r está contida em
plano que contém P, é paralelo a r e a.
perpendicular a a. b. toda reta perpendicular a r é perpen-
b. Discuta o caso particular em que r é dicular a a.
perpendicular a a. c. toda reta ortogonal a r é perpendicular
Resolução a a.
a. Para existir o plano pedido, devemos ter que d. existem retas paralelas a r que são per-
o ponto P ∉ r. Assim: pendiculares a a.
r e. existem retas contidas em a que não
t são paralelas a r.
r'
P Resolução
a. Falsa; existem infinitas retas paralelas a r
que não estão contidas em a.
α b. Falsa; a reta pode ser paralela ou oblíqua a
a.
(s paralela a a)
s
1º) Traçamos por P uma reta r’ // r. r
2º) Traçamos por P uma reta t ⊥ a.
3º) O plano determinado por r’ e t é o pla- (s paralela a α)
no procurado. α

b. Quando r é perpendicular a a, existem in- c. Falsa; no exemplo anterior, basta tomar s


finitos planos nas condições apresentadas, se reversa (e ortogonal) com r.
P ∈ r. d. Falsa; as paralelas a r serão paralelas a a
r ou estarão contidas em a.
t r'
e. Verdadeira; podem ser reversas.
P Resposta
E

α
PV-13-31

18
Geometria de posição e métrica Matemática

09. UFSCar-SP 10. ITA-SP


Considere um plano a e um ponto P qualquer Qual das afirmações abaixo é verdadeira?
do espaço. Se por P traçarmos a reta perpendi- a. Três pontos, distintos dois a dois, deter-
cular a a, a intersecção dessa reta com a é um minam um plano.
ponto chamado projeção ortogonal do ponto
b. Um ponto e uma reta determinam um
P sobre a. No caso de uma figura S do espaço,
plano.
a projeção ortogonal de S sobre a é definida
pelo conjunto das projeções ortogonais de c. Se dois planos distintos têm um ponto
seus pontos. em comum, tal ponto é único.
Com relação a um plano a qualquer fixado, d. Se uma reta é paralela a um plano e
pode-se dizer que: não está contida neste plano, então ela
é paralela a qualquer reta desse plano.
a. a projeção ortogonal de um segmento
de reta pode resultar numa semirreta. e. Se a é o plano determinado por duas
retas concorrentes r e s, então toda
b. a projeção ortogonal de uma reta sem- reta m desse plano, que é paralela à r,
pre resulta numa reta. não será paralela à reta s.
c. a projeção ortogonal de uma parábola
Resolução
pode resultar num segmento de reta.
a. Falso. Podem estar alinhados
d. a projeção ortogonal de um triângulo
pode resultar num quadrilátero. b. Falso. Faltou dizer não contido na reta.
e. a projeção de uma circunferência pode c. Falso. Sempre será uma reta comum (dis-
resultar num segmento de reta. cutiremos melhor no próximo módulo).
Resolução d. Falso. Pode ser reversa (discutiremos me-
lhor no próximo módulo).
A projeção ortogonal de uma circunferência
e. Verdadeiro (Como m e s, são coplanares,
será um segmento de reta, quando ela estiver
caso m//r, o que nos levaria r//s que é uma
contida num plano perpendicular à a.
contradição, pois r × s. Assim m não será pa-
Resposta ralela a s.).
E Resposta
E
PV-13-31

19
Matemática Geometria de posição e métrica

CAPÍTULO 02 POLIEDROS

1. Superfície poliédrica 2. Poliedro convexo


Consideremos n (n ∈N*) polígonos convexos Consideremos um número finito n(n ≥ 4) de
(regiões poligonais) tais que: polígonos convexos (regiões poligonais), tais
01. dois quaisquer nunca são coplanares; que:
02. o plano contendo um deles deixa os de- 01. dois quaisquer nunca são coplanares;
mais no mesmo semiespaço; 02. o plano contendo um deles deixa os de-
03. cada lado de polígono está no máximo mais no mesmo semiespaço;
em dois polígonos. 03. cada lado de polígono é comum a dois
A união desses polígonos forma uma figura e somente a dois polígonos.
denominada superfície poliédrica convexa. Nessas condições, ficam determinados n se-
Os polígonos são as faces, e os seus lados, as miespaços, cada um dos quais com origem no
arestas da superfície poliédrica convexa. plano de um polígono e contendo os restan-
Quando uma superfície poliédrica possui ares- tes. A intersecção desses semiespaços é cha-
tas livres que formam um único ”contorno” mada de poliedro convexo.
fechado, ela é chamada aberta, e quando ela Os polígonos convexos são as faces do polie-
não possui arestas livres, fechada. dro; os lados dos polígonos são as arestas do
Quando uma superfície poliédrica é fechada poliedro e os vértices dos polígonos são os
ou aberta com um só contorno, dizemos que vértices do poliedro.
ela é simplesmente convexa ou de ”conexão A reunião das faces é a superfície do poliedro.
1”; quando ela tem “n”contornos, é de “co-
nexão n”.

3. Lema do teorema de Euler


Consideremos uma superfície poliédrica con-
vexa aberta com um único contorno (sim-
Superfície poliédrica aberta com
PV-13-31

plesmente convexa), com Va vértices, Aa ares-


um único contorno tas e Fa faces, então:
(simplesmente convexa)
Va – Aa + Fa = 1

Demonstração
Aberta em cima Por indução finita quanto ao número de faces
e em baixo
Fa.
1ª parte
Superfície poliédrica aberta Para Fa = 1: nesse caso, a superfície poliédrica
com dois contornos se reduz a um único polígono com n lados e n
(conexão 2) vértices, então:

20
Geometria de posição e métrica Matemática

Va = n  em que V é o número de vértices, A é o nú-


 mero de arestas e F é o número de faces do
A a = n ⇒ Va − A a + Fa = n − n + 1 = 1 poliedro.
Fa = 1  Demonstração
Logo, a relação está verificada para Fa = 1. Tomamos a superfície de qualquer poliedro
convexo ou qualquer superfície poliédrica
2ª parte convexa fechada (com V vértices, A arestas e
Admitamos que a relação seja válida para uma F faces) e dela retiramos uma face. Ficamos,
superfície com F’ faces (que possui V’ vértices então, com uma superfície aberta (com Va vér-
e A’ arestas) e vamos provar que também vale tices, Aa arestas e Fa faces) para a qual vale a
para uma superfície de F’+ 1 faces (que possui relação:
F’ + 1 = Fa faces, Va vértices e Aa arestas). Va – Aa + Fa = 1
Assim, por hipótese, a superfície de F’ faces, A’ Como:
arestas e V’ vértices vale:
Va = V, Aa = A e Fa = F – 1, temos:
V’ – A’ + F’ = 1 V – A + (F – 1) = 1

Acrescentando a essa superfície (que é aberta)


Então: V – A + F = 2
uma face de p arestas (lados) e considerando
que q dessas arestas (lados) coincidem com
arestas já existentes, obtemos uma nova su- Observação
perfície com Fa faces, Aa arestas e Va vértices,
tais que: A relação de Euler vale para todos os poliedros
convexos, porém existem poliedros que não
Fa = F’ + 1 são convexos e mesmo assim são eulerianos.
Aa = A’ + p – q (q arestas coincidiram.) Exemplos
Va = V’ + p – (q + 1) ( q arestas coincidindo, 01.
q + 1 vértices coincidem.)
Substituindo esses valores na expressão
Va – Aa + Fa, temos:
Va – Aa + Fa = [V’ + p – (q + 1)] – [A’ + p – q] +
+ [F’ + 1] = V’ + p – q – 1 – A’ – p + q + F’ + 1
Assim:
Va – Aa + Fa = V’ – A’ + F’
PV-13-31

Como, por hipótese, V’– A’ + F’ = 1, temos que:


Va – Aa + Fa = 1 Poliedro não convexo:
V = 12 

4. Teorema de Euler A = 18 V − A + F = 2 (euleriano)
Para todo poliedro convexo, ou para sua su- F = 8 
perfície, vale a relação:
V–A+F=2

21
Matemática Geometria de posição e métrica

02. Demonstração
Consideremos um poliedro convexo com V
vértices, A arestas e F faces.
Sejam n1, n2, ..., nf os números de lados das fa-
ces 1, 2, ..., F, respectivamente.
Sabemos que a soma dos ângulos de uma face
com n lados é: S = (n – 2) · 2 r.
Para todas as faces, temos:
Poliedro não convexo S = (n1 – 2) · 2r + (n2 – 2) · 2r + ... + (nF – 2) · 2r
V = 12  S = n1 · 2r – 4r + n2 · 2r – 4r + ... + nF · 2r – 4r
 S = (n1 + n2 + ... + nF) · 2r – (4r + 4r + ... + 4r)
A = 24  V − A + F = 0 (não euleriano) 
F = 12  F vezes

Como n1 + n2 + ... + nF = 2 A, temos:


5. Propriedade S = 2 A · 2r – F · 4r
A soma dos ângulos de todas as faces de um S = (A – F) · 4r
poliedro convexo é:
Mas V – A + F = 2 ⇒ A – F = V – 2
S = (V – 2) · 4 r
em que V é o número de vértices e r é um ân- Assim: S = (V – 2) · 4 r
gulo reto.

EXERCÍCIOS RESOLVIDOS
01. 02.
Determine o número de vértices de um polie- Um poliedro convexo possui apenas 5 faces
dro convexo que possui 12 faces triangulares. quadrangulares e 2 pentagonais. Determine o
Resolução número de vértices, arestas e faces.
F = 12 Resolução
Temos F = 5 + 2 = 7
Cada aresta pertence a 2, e somente 2, po- 4 vértices em cada face quadrangular
lígonos. Se fizermos a conta 12 · 3, estaremos 5 faces
contando todas as arestas 2 vezes; assim:
12 · 3 = 2A \ A = 18
PV-13-31

Pela relação de Euler, temos:


V – A + F = 2 ⇒ V – 18 + 12 = 2 \ V = 8
5 vértices em cada face pentagonal
2 faces

Calculando o total de arestas, temos:


5 · 4 + 2 · 5 = 2A \ A = 15
V – A + F = 2 ⇒ V – 15 + 7 = 2 \ V = 10

22
Geometria de posição e métrica Matemática

03. Em cada ângulo triédrico, temos 3 arestas,


A soma dos ângulos das faces de um poliedro mas cada aresta une 2 vértices consecutivos.
convexo vale 720°. Sabendo-se que o número Assim:
2 16 · 3 = 2 A \ A = 24
de faces vale do número de arestas, deter-
3 Obviamente, teremos 16 vértices, assim:
mine o número de faces. V–A+F=2
Resolução 16 – 24 + F = 2 \ F = 10
S = 360° · (V – 2) = 720° ⇒ V – 2 = 2 \ V = 4 Resposta
2
Como F = · A, temos V – A + F = 2 F = 10
3
2 A 05.
4 − A + ⋅ A = 2 ⇒ 2 − = 0∴ A = 6
3 3 Determinar o número de vértices de um po-
2 liedro convexo que tem apenas 3 faces trian-
F = ⋅6 = 4 gulares, 1 quadrangular, 1 pentagonal e 2 he-
3 xagonais.
Resposta Resolução
F=4
04.
Achar o número de faces de um poliedro con- 3 1 1 2
vexo que possui 16 ângulos triédricos.
F=3+1+1+2=7
Resolução
2 A = 3 · 3 + 1 · 4 + 1 · 5 + 2 · 6 ⇒ 2 A = 30
Vértice triédrico
\ A = 15
V – A + F = 2 ⇒ V – 15 + 7 = 2 \ V = 10
Resposta
V = 10

6. Poliedros de Platão
A. Definição
Um poliedro euleriano é chamado poliedro de Platão quando:
01. todas as faces têm o mesmo número de arestas;
PV-13-31

02. todos os ângulos poliédricos têm o mesmo número de arestas.


Exemplos
O hexaedro da figura é um poliedro de Platão, pois

é euleriano:
F = 6; V = 8; A = 12
V–A +F=2
Todas as faces têm 4 arestas
e
todos os ângulos poliédricos têm 3 arestas.

23
Matemática Geometria de posição e métrica

A pirâmide da figura não é um poliedro de Pla- 1 1


tão, pois: m> 3⇒m≥ 4 ⇒ ≤
m 4  1 1 1
V ⇒ + ≤
1 1  m n 2
n> 3⇒n≥ 4 ⇒ ≤
n 4 
D C ou seja:
1 1 1 1 1 1 1 1
− + ≤ − ⇒ − + ≤0
A B m 2 n 2 2 m 2 n

embora seja euleriana: o que contrariará a condição (IV), pois A é um


número positivo.
F = 5; V = 5, A = 8
Concluímos, então, que, nos poliedros de Pla-
V–A+F=2 tão, m = 3 ou n = 3 (isto significa que, num
Temos faces com 4 arestas (base) e faces com poliedro de Platão, temos obrigatoriamente
3 arestas, temos também vértices com 4 ares- triedro ou triângulo).
tas e vértices com 3 arestas. Assim:
01. Para m = 3 (supondo que tem triedro),
B. Teorema de Platão em (IV) temos:
Existem cinco, e somente cinco, classes de po- 1 1 1 1 1 1 1
liedros de Platão. − + = ⇒ − =
3 2 n A n 6 A
Demonstração
1 1
Usando as condições que devem ser verifica- Assim, > ⇒n< 6
das por um poliedro de Platão, temos: n 6
01. V – A + F = 2 (I) Então, n = 3 ou n = 4 ou n = 5.
02. Cada uma das F faces tem n arestas 02. Para n = 3 (supondo que tem triângulo),
(n ≥ 3), e, como cada aresta está em em (IV) temos:
duas faces, então:
2A 1 1 1 1 1 1 1
n⋅F = 2A ⇒ F = (II) − + = ⇒ − =
n m 2 3 A m 6 A
1 1
03. Cada um dos ângulos poliédricos tem Assim, > ⇒ m < 6
m 6
m arestas (m ≥ 3), como cada aresta
contém dois vértices, então: Então, m = 3 ou m = 4 ou m = 5.
2A Resumindo os resultados encontrados nas con-
m⋅ V = 2A ⇒ V = (III)
m dições 01 e 02, concluímos que os poliedros de
PV-13-31

Platão são determinados pelos pares (m, n) da


Substituindo (II) e (III) em (I), temos: tabela abaixo. Sendo, portanto, cinco, e somen-
2A 2A te cinco, as classes de poliedros de Platão.
−A+ =2
m n m 3 3 3 4 5
Dividindo por 2 A: n 3 4 5 3 3
1 1 1 1
− + = (IV) C. Os poliedros de Platão
m 2 n A
Conhecendo m e n para cada um dos cinco po-
Sabemos que n ≥ 3 e m ≥ 3. Porém, se m e n liedros de Platão, podemos determiná-los usan-
forem, simultaneamente, maiores que 3, tere- do as relações II, III e IV obtidas no subitem an-
mos: terior.

24
Geometria de posição e métrica Matemática

Assim, para m = 3 e n = 3, temos: – dodecaedro regular: as faces são pen-


1 1 1 1 tágonos regulares;
em IV: − + = ⇒ A = 6
3 2 3 A – icosaedro regular: as faces são triângu-
los equiláteros.
2⋅6
em II: F = ⇒F= 4
3
2⋅6
em III: V = ⇒V=4
3
Procedendo do mesmo modo, temos:

m n A V F Nome

3 3 6 4 4 Tetraedro Tetraedro regular


3 4 12 8 6 Hexaedro

4 3 12 6 8 Octaedro

3 5 30 20 12 Dodecaedro

5 3 30 12 20 Icosaedro

7. Poliedros regulares
Denominamos poliedros regulares aos polie- Hexaedro regular Octaedro regular
dros de Platão, cujas faces são polígonos regu-
lares congruentes e cujos ângulos poliédricos
são congruentes (coincidem por superposição).
Há cinco tipos de poliedros regulares:
– tetraedro regular: as faces são triângu-
los equiláteros;
– hexaedro regular: as faces são quadra-
das;
– octaedro regular: as faces são triângu- Dodecaedro regular Icosaedro regular
los equiláteros;
PV-13-31

EXERCÍCIOS RESOLVIDOS

01. FAAP-SP Usando a relação de Euler e substituindo A de


Num poliedro convexo, o número de arestas acordo com a igualdade apresentada:
excede o número de vértices em 6 unidades. V+F=2+A
Calcule o número de faces. V+F=2+V+6
Resolução Eliminando V:
De acordo com o enunciado, temos: F=8
A=V+6 Resposta
O número de faces é igual a 8.

25
Matemática Geometria de posição e métrica

02. Fatec-SP 03.


Um poliedro convexo tem 3 faces com 4 lados, O hexaedro da figura abaixo é um poliedro de
2 faces com 3 lados e 4 faces com 5 lados. Qual Platão?
é o número de vértices desse poliedro?
Resolução
Do enunciado, sabemos que:
Número de faces: 3 + 2 + 4 = 9
Números de arestas:
Resolução
3 faces com 4 lados: 3 · 4 = 12
Não, pois tem faces triangulares (com 3 ares-
2 faces com 3 lados: 2 · 3 = 6
tas) e faces quadrangulares (com 4 arestas).
4 faces com 5 lados: 4 · 5 = 20
04.
Somando: 12 + 6 + 20 = 38
O hexaedro da figura abaixo é um poliedro re-
Atenção: as faces são unidas, duas a duas, por gular? Justifique.
uma aresta. Ao contarmos todas as arestas H G
de todas as faces, cada aresta é contada duas
vezes, uma para cada face “grudada” nela. As- E
sim, esse número, na verdade, é o dobro do F
número real de arestas do poliedro. Logo:
A = 38 ÷ 2 = 19 D C
Usando, agora, a relação de Euler, temos:
A B
V+F=2+A
V + 9 = 2 + 19 Resolução
V = 21 – 9 = 12 Não, pois, embora seja um poliedro de Platão,
as faces não são polígonos regulares e con-
Resposta
gruentes entre si.
V = 12

PV-13-31

26
Geometria de posição e métrica Matemática

CAPÍTULO 03 PRISMAS

1. Definição e elementos 2. Nomenclatura e classificação


Prisma é um poliedro convexo tal que duas faces Os prismas recebem nomes de acordo com os
são polígonos congruentes situados em planos polígonos das bases.
paralelos e as demais faces são paralelogramos. Assim:
B • um prisma é triangular quando suas ba-
ses são triângulos;
α • um prisma é quadrangular quando suas
A C
bases são quadriláteros;
h • um prisma é pentagonal quando suas
bases são pentagonais;
B' • um prisma é hexagonal quando suas
bases são hexagonais.
α' Quando as arestas laterais de um prisma forem
A' C'
perpendiculares aos planos das bases, o prisma
Na figura acima, temos: é chamado de reto; caso contrário, de oblíquo.
01. os triângulos ABC e A’B’C’ (polígonos Os prismas retos cujas bases são polígonos re-
congruentes situados em planos para- gulares são chamados de prismas regulares.
lelos) são as bases do prisma; Exemplos
02. os paralelogramos ABB’A’, CBB’C’ e
ACC’A’ (demais faces) são as faces late-
rais do prisma;
03. os lados dos polígonos que são as bases
do prisma, AB, BC, AC, A’B’, B’C’e A’C’,
são as arestas das bases do prisma;
04. os lados das faces laterais que têm uma
extremidade em cada base são as ares- Prisma triangular Prisma triangular
oblíquo reto
tas laterais do prisma;
05. a distância entre os planos das bases é
a altura do prisma.
Observação
Caso as arestas laterais sejam perpendiculares
PV-13-31

aos planos das bases, suas medidas coincidem


com a altura do prisma. Nesse caso, as faces
laterais são retângulos e estão situadas em Prisma triangular Prisma quadrangular
planos perpendiculares aos planos das bases. regular reto

h h

Prisma quadrangular Prisma hexagonal


regular regular

27
Matemática Geometria de posição e métrica

3. Cubo
A. Defi nição e elementos
Cubo é um prisma em que todas as faces são
quadradas. O cubo é um prisma quadrangular
regular cuja altura é igual à medida da aresta
da base.

Cubo planificado

C. Volume
Para o cálculo do volume de um sólido, ado-
O cubo da figura tem arestas de medida l, en- tamos, como unidade de medida, um cubo
tão: de aresta unitária. Assim, um cubo de ares-
• as diagonais de suas faces medem l 2 ta 1 metro (1 m) tem volume 1 metro cúbico
, pois são diagonais de quadrados de (1m3); um cubo de aresta 1 centímentro
lados com medidas iguais a l. (1cm) tem volume 1 centímentro cúbico
(1cm3); um cubo de aresta 1 decímetro (1 dm)
tem volume 1 decímetro cúbico (1 dm3).
d d=l 2 Consideremos agora um cubo de aresta 3 cm.
Observamos que nele “cabem” 27 cubos de
aresta 1 cm. Assim o seu volume é 27 cm3.

• as diagonais do cubo medem l 3 , pois:

 2

De forma genérica, afirmamos que um cubo


d2 = l2 + (l 2 )2 = 3l2 de aresta l tem volume V dado por:
PV-13-31

V = l3
Assim, d = l 3

B. Área 4. Paralelepípedos
A superfície do cubo é a reunião dos seis qua- A. Defi nição
drados que são suas faces. Chamamos a medi-
Chamamos de paralelepípedo o prisma cujas
da dessa superfície de área do cubo.
bases são paralelogramos; dessa forma, todas
A área de um quadrado de lado l é l2, então a as faces de um paralelepípedo são paralelo-
área A da superfície de um cubo de aresta l é: gramos.
A = 6 l2

28
Geometria de posição e métrica Matemática

Exemplos No triângulo ABC, temos:


AC2 = AB2 + BC2
ou então,
d21 = a2 + c2

Paralelepípedo oblíquo No triângulo ACG, temos:


AG2 = AC2 + CG2
ou então,
d2 = d21 + b2

Como d21 = a2 + c2, temos:


d2 = a2 + c2 + b2 ou
Paralelepípedo reto
d 2 = a 2 + b 2 + c2

B. Paralelepípedo reto retângulo B.2. Área total (AT) de um


Chamamos de paralelepípedo reto retângulo o paralelepípedo retângulo
paralelepípedo reto cujas bases são retângu- Sendo a, b e c as dimensões de um paralelepí-
los; dessa forma, todas as suas seis faces são re- pedo retângulo, as áreas de cada par de faces
tângulos. Ele também é chamado de ortoedro opostas são: ab, ac e bc.
ou simplesmente paralelepípedo retângulo.
Assim:
AT = 2ab + 2ac + 2bc
c
ou
AT = 2 · (ab + ac + bc)
b
a
ab
c
Dizemos que, no paralelepípedo da figura, a, b
e c são as suas dimensões, e percebemos que ac bc
as medidas das 12 arestas são iguais a a, b e
c, sendo quatro delas com cada uma das três b
medidas. a
PV-13-31

B.1. Diagonais de um
paralelepípedo retângulo B.3. Volume (V) de um
paralelepípedo retângulo
No paralelepípedo da figura com dimensões a,
b e c, sejam d1 e d, as diagonais da face ABCD e Consideremos inicialmente um paralelepípe-
do paralelepípedo, respectivamente. do retângulo de dimensões 4 cm, 3 cm e 2 cm.
E F

A D 2 cm
d
c d1
G
b 3 cm
B a C 4 cm

29
Matemática Geometria de posição e métrica

Notamos que nesse paralelepípedo cabem 2 B.4. Relação importante


camadas de cubinhos de aresta unitária, e que Sendo a, b e c as dimensões de um paralelepí-
em cada camada cabem 4 · 3 cubinhos. pedo retângulo, temos:
(a + b + c)2 = a2 + b2 + c2 + 2ab + 2ac + 2bc
1 Como
1
1 a2 + b2 + c2 = d2
e
Como o volume de cada cubinho é 1 cm3, o vo- 2ab + 2ac + 2bc = AT
lume do paralelepípedo é: concluímos que:
V = 2 · 4 · 3 = 24 cm3
(a + b + c)2 = d2 + AT
Sendo a, b e c as dimensões do paralelepípedo
retângulo, temos:
V =a·b·c

EXERCÍCIOS RESOLVIDOS

01. ENEM Resolução


Uma fábrica produz barras de chocolate no As figuras abaixo representam barras de cho-
formato de paralelepípedos e de cubos, com o colate na forma de paralelepípedo reto retân-
mesmo volume. As arestas da barra de chocola- gulo e cubo de mesmo volume.
te no formato de paralelepípedo medem 3 cm
de largura, 18 cm de comprimento e 4 cm de
espessura. 4 cm

Analisando as características das figuras geo-


métricas descritas, a medida das arestas dos 3 cm
chocolates que têm o formato de cubo é igual a: 18 cm

a. 5 cm
b. 6 cm
c. 12 cm
PV-13-31

d. 24 cm
e. 25 cm
Considerando (a) a aresta do cubo, tem-se:
a3 = 18 · 3 · 4
\ a = 6 cm
Resposta
B

30
Geometria de posição e métrica Matemática

02. ENEM a. 12 cm3 d. 1.216 cm3


A siderúrgica “Metal nobre” produz diversos ob- b. 64 cm3 e. 1.728 cm3
jetos maciços utilizando o ferro. Um tipo especial c. 96 cm3
de peça feita nessa companhia tem o formato de
um paralelepípedo retangular, de acordo com as Resolução
dimensões indicadas na figura que segue. O volume do porta-lápis (Vpl) referido pelo
enunciado é:
Vpl = V – v, em que V e v são os volumes de
1,3 m
Metal nobre cubos de aresta 12 e 8 centímetros, respecti-
vamente.
0,5 m
2,5 m Assim, temos:
Vpl = (12)3 – (8)3 = 1.216
O produto das três dimensões indicadas na
∴Vpl = 1.216 cm3
peça resultaria na medida da grandeza:
a. massa. Resposta
b. volume. D
c. superfície. 04.
d. capacidade. Qual é o volume de um cubo de área 54 cm2 ?
e. comprimento. Resolução
Resolução Sendo a a aresta do cubo, temos:
Observe a figura abaixo. 6a2 = 54 ⇒ a2 = 9
Assim, a = 3 cm
1,3 m V = a3 ⇒ V = 33 = 27 cm3
Metal nobre
05.
0,5 m A diagonal de uma face do cubo tem medida
2,5 m 5 2 cm. Qual é a área do cubo?
O produto das 3 dimensões resulta na medida Resolução
de grandeza chamada volume. Sendo a a aresta do cubo, temos:
Resposta a 2 = 5 2 ⇒ a = 5 cm
B A = 6a2 ⇒ A = 6 ⋅ 52
03. ENEM ∴ A = 150 cm2
PV-13-31

Um porta-lápis de madeira foi construído no


formato cúbico, seguindo o modelo ilustrado 06.
a seguir. O cubo de dentro é vazio. A aresta do Aumentando de 1 cm a aresta de um cubo, a
cubo maior mede 12 cm e a do cubo menor, área de uma face aumenta de 7 cm2. Qual é a
que é interno, mede 8 cm. área total do cubo?
Resolução
Sendo a a aresta do cubo, temos:
(a + 1)2 = a2 + 7
a2 + 2a + 1 = a2 + 7 ⇒ a = 3
O volume de madeira utilizado na confecção A = 6a2 ⇒ A = 6 · 32 ⇒ A = 54 cm2
desse objeto foi de:

31
Matemática Geometria de posição e métrica

07. c. V = a · b · c
Num cubo de volume 8a , qual a distância do
3
V=3·4·5
centro (ponto de encontro das diagonais) ao Assim, V = 60 cm3
ponto médio de uma aresta?
09.
Resolução
A área total de um ortoedro é 720 cm2, a dia-
Sendo l a aresta do cubo, temos: gonal de uma face mede 20 cm e a soma de
l3 = 8a3 ⇒ l = 2a suas dimensões é 34 cm. Calcular as dimen-
sões.
Resolução
Sendo d = 20 cm a diagonal da face de arestas
O a e b, temos:
2a a

P a A
2a
b 20 cm
OA2 = OP2 + PA2 c
OA = (a) + (a)
2 2 2
a
OA2 = 2a2
a2 + b2 = 202 (I)
∴ OA = a 2
2ab + 2ac + 2bc = 720 (II)
08. Como: a + b + c = 34
Num paralelepípedo retângulo de dimensões (a + b + c)2 = 342
3 cm, 4 cm e 5 cm, calcular: 2 + b2 + c2 + 2ab + 2ac + 2ba = 1.156
a 
a. a medida da diagonal; 202 720
b. a área total AT;
400 + c2 + 720 = 1.156
c. o volume V.
c2 = 36
Resolução
Assim, c = 6 cm
Então:
3 cm
a2 + b2 = 400

a + b = 28
PV-13-31

4 cm Resolvendo o sistema, encontramos:


5 cm a = 16 cm e b = 12 cm ou
a. d2 = a2 + b2 + c2 a = 12 cm e b = 16 cm
d2 = 32 + 42 + 52 = 50 Nesse caso, as dimensões do ortoedro são:
Assim, d = 5 2 cm 16 cm, 12 cm e 6 cm
b. AT = 2ab + 2ac + 2bc
AT = 2 · 5 · 4 + 2 · 5 · 3 + 2 · 4 · 3
Assim, AT = 94 cm2

32
Geometria de posição e métrica Matemática

10. Resolução
Calcular o volume de um paralelepípedo re- Inicialmente, o volume de água no reservató-
tângulo, sabendo que suas dimensões são rio era de 10 · 20 · 30 = 6.000 m3. Após a eva-
proporcionais a 9, 12 e 20, e que a diagonal poração, restaram 6.000 – 1.800 = 4.200 m3.
mede 100 m. Sendo h a altura atingida pela água restante
no reservatório, temos:
c=9k h · 20 · 30 = 4.200 ⇔ h = 7 m
d = 100 m Resposta
C
b = 12 k
12. Fuvest-SP
a = 20 k
Dois blocos de alumínio, em forma de cubo,
Resolução com arestas medindo 10 cm e 6 cm, são le-
vados juntos à fusão e em seguida o alumínio
d 2 = a 2 + b 2 + c2 líquido é moldado como um paralelepípedo
1002 = (20k)2 + (12k)2 + (9k)2 reto de arestas 8 cm, 8 cm e x cm. O valor de
x é:
1002 = 625k2
a. 16
Assim: 25k = 100 ⇒ k = 4
b. 17
Então: a = 20 · 4 = 80 m
c. 18
b = 12 · 4 = 48 m d. 19
c = 9 · 4 = 36 m e. 20
V = a · b · c = 80 · 48 · 36 Resolução
V = 138.240 m3 Pelo enunciado, o volume do paralelepípedo é
11. Vunesp igual à soma dos volumes dos cubos.
A água de um reservatório, na forma de um pa- Assim:
ralelepípedo retângulo, de comprimento 30 m 8 · 8 · x = 63 + 103
e largura 20 m, atingia a altura de 10 m. Com a
falta de chuvas e o calor, 1.800 metros cúbicos 64 x = 216 + 1.000
da água do reservatório evaporaram. A água 64 x = 1.216 ⇒ x = 19
restante no reservatório atingiu a altura de: Resposta
a. 2 m d. 8 m
D
b. 3 m e. 9 m
c. 7 m
PV-13-31

33
Matemática Geometria de posição e métrica

5. Princípio de Cavalieri A
B

Consideremos um prisma pentagonal e vamos


selecioná-lo por planos paralelos de modo a
dividi-lo em dez pequenos prismas pentago-
nais com as mesmas bases e altura.
d B' d
A'

1
2
3 α
4
5 Como o volume de A é igual à soma dos vo-
6
7 lumes dos pequenos sólidos A’ e o volume de
8 B é igual à soma dos volumes dos pequenos
9
10
sólidos B’, os sólidos A e B têm o mesmo vo-
lume.
Podemos observar que dispomos os dez pris- A partir desses conceitos podemos estabele-
mas de modos diferentes e o volume do sólido cer o princípio de Cavalieri (Francisco Bona-
permanece o mesmo do prisma original. ventura Cavalieri, 1598-1647):
Sejam dois sólidos, A e B, cujas bases estão
contidas num mesmo plano a. Se todo pla-
no b, paralelo a a, interceptar A e B deter-
1 B 1 C minando secções de mesma área, então os
2 2
3 3 sólidos A e B têm o mesmo volume.
4 4
5 5
6
7
6
7 6. Volume de um prisma qualquer
8 8
9 9 Consideremos um prisma P1, de altura h e área
10
10
da base B1 = S, e um paralelepípedo retângulo
P2 de altura h e área da base B2 = S.
Consideremos agora dois sólidos A e B cujas P1
bases têm a mesma área e estão contidas num P2
mesmo plano a. Consideremos ainda que
qualquer plano b paralelo a a intercepta os
dois sólidos em secções de mesma área.
B1 B2
A B β h
PV-13-31

Áreas B1 B2
iguais α
β

α
Como as secções nos dois sólidos, por qual-
quer plano b paralelo a a, apresentam a mes-
Se seccionarmos os sólidos, A e B, por planos ma área S, podemos garantir pelo princípio de
paralelos a a, de modo que a distância d entre Cavalieri que o prisma tem o mesmo volume
cada par de planos seja mínima, cada pequeno do paralelepípedo.
sólido formado em A terá o mesmo volume do VP1 = VP2
correspondente formado em B.

34
Geometria de posição e métrica Matemática

Como VP2= B2 · h ou seja, VP2= S · h , vem que 7. Área e volume de


VP1 = S · h, ou em síntese: prismas regulares
V=S·h Sabemos que um prisma é chamado de regu-
lar quando é reto e tem base regular.
Conclusão A seguir, vamos calcular a área e o volume dos
principais prismas regulares.
O volume de um prisma é o produto da área
da base pela medida da altura. A. Prisma triangular regular
Consideremos um prisma triangular regular
Observação
com aresta da base a e altura h.
É possível provar que o volume de qualquer
prisma pode ser obtido multiplicando a área a a
da secção reta pela medida da aresta lateral a
do prisma.

 V=S·l
S
a

A.1. Área da base (B)


a 3
Exemplo a⋅
2 =a ⋅ 3
2
Calcular o volume ocupado por um salão que B=
2 4
tem a forma de um prisma quadrangular e di-
mensões indicadas na figura.
A.2. Área lateral (AL)
AL = 3 · Aface lateral
AL = 3 · (ah) = 3ah

A.3. Área total (AT)


AT = AL + 2B
5m
a2 ⋅ 3
3m AT = 3ah + 2
PV-13-31

4
8m
a2 ⋅ 3
4m AT = 3ah +
2
Devemos observar que o prisma tem como base A.4. Volume (V)
um trapézio retângulo e a altura mede 6 m.
V=B·h
A área da base (Ab) é:
a2 ⋅ 3
(5 + 3) V= ·h
Ab = ⋅ 4 = 16 m2 4
2
Assim, o volume (V) do prisma é:
V = Ab · h = 16 · 8 = 128 m3

35
Matemática Geometria de posição e métrica

Exemplo B.1. Área da base (B)


Em um prisma triangular regular, a área da B = a2
base é 9 3 m2 e a área lateral é o triplo da
área da base. Calcular o volume desse prisma. B.2. Área lateral (AL)
a2 3 AL = 4 · Aface lateral
9 3= ⇒ a2 = 36
4 AL = 4 · (ah) = 4ah
Então: a = 6 m
B.3. Área total (AT)
AL = 3 B
AT= AL + 2B
a2 3
3 ah = 3 ⋅ AT = 4ah + 2a2
4
62 3 B.4. Volume (V)
3 ⋅ 6 ⋅h = 3 ⋅
4 V=B·h
3 3 V = a2 · h
h= m
2 Exemplo
a a Calcular a área total de um prisma quadran-
a gular regular de volume 54 cm3, sabendo que
a aresta lateral deste sólido tem o dobro da
medida da aresta da base.

V=B·h h = 2a
a2 3
V= ⋅h
4
62 ⋅ 3 3 3 a
V= ⋅
4 2 a
81 3
∴V = m V=B·h
2
V = a2 · 2a = 54
B. Prisma quadrangular regular 2a3 = 54 ⇒ a = 3 cm
PV-13-31

Consideremos um prisma quadrangular regu-


lar com aresta da base a e altura h. Assim: h = 2a = 6 cm
a
a
Então: AT = 2a2 + 4ah
a
a AT = 2 · 32 + 4 · 3 · 6
AT = 90 cm2

C. Prisma hexagonal regular


h
Consideremos um prisma hexagonal regular
com aresta da base a e altura h.

a
a

36
Geometria de posição e métrica Matemática

C.4. Volume (V)


V=B·h
3a2 ⋅ 3
V= ⋅h
2
h
Exemplo
Num prisma hexagonal regular, a área lateral é
75% da área total.
a a
a Calcule a razão entre a aresta lateral e a aresta
da base.
C.1. Área da base (B)
a

a a

a a
a a

 a2 ⋅ 3  3a2 ⋅ 3
B = 6⋅ B=
 4  a a
2
a

C.2. Área lateral (AL) AL = 6ah


AL = 6 · Aface lateral A T = 6 ah + 3a2 3
AL = 6 · (ah) = 6ah AL = 75% de A T
C.3. Área total (AT)
AT = AL + 2B
3
6 ah =
4
(
6 ah + 3a2 3 )
 3a2 ⋅ 3  ah 3 3 h 3 3
= ⇒ =
A T = 6 ah + 2  a2 2 a 2
 2 
A T = 6 ah + 3a2 3
PV-13-31

37
Matemática Geometria de posição e métrica

EXERCÍCIOS RESOLVIDOS
01. Mackenzie-SP Se = 6 ⋅ aH = 12 ⇒ aH = 2
Um prisma regular triangular tem todas as 
 3a2 3 8 3
arestas congruentes e 48 m2 de área lateral. V = ⋅ H = 4 ⇒ a2H =
Seu volume vale:  2 2
a. 16 m3 Resolvendo o sistema:
b. 32 m3 4 3 3 3
a= meH= m
c. 64 m3 9 2
d. 4 3 m3
Resposta
e. 16 3 m3 4 3
Aresta da base = m
Resolução 9
3 3
a a Altura = m
a a Base 2
a
03. Mackenzie-SP
Se a soma dos ângulos internos de todas as
a a
Face faces de um prisma é 6.480°, então o número
de lados da base do prisma é:
a. 8 c. 10 e. 15
a a b. 9 d. 12
a2 3 Resolução
AB =
4 Sendo n o número de lados da base do prisma,
A f = a2 então este possui n faces laterais quadrangula-
res e duas faces que são polígonos de n lados.
AL = 3 · Af ⇒ 3a2 = 48 ⇒ a2 = 16 ⇒ a = 4 m Portanto, a soma dos ângulos internos de to-
das as sua faces é:
V = AB · H
n · 360° + 2 · (n – 2) · 180°
a2 3 16 3 ⋅ 4
V= ⋅a ⇒ V = ⇒ V = 16 3 m3 Consequentemente, n · 360° + 2 · (n – 2) · 180 =
4 4 = 6.480° ⇔ n = 10
Resposta Resposta
E C
PV-13-31

02.
Calcule a altura e a aresta da base de um pris-
ma hexagonal regular, sabendo que seu volu-
me é 4 m3 e a superfície lateral é 12 m2.
Resolução

a
a a
H
a a
a a

38
Geometria de posição e métrica Matemática

04. onde A, B, C, D, E, F, G e H representam um


Um prisma regular é equivalente a um cubo de paralelepípedo retângulo e E, F, G, H, I e J um
aresta a. Determine a altura do prisma saben- prisma, cuja base E, H e I é um triângulo re-
do que sua aresta da base mede a. tângulo (com ângulo reto no vértice H e ân-
3
Resolução gulo a no vértice I tal que sen α =  . A su-
5
Equivalente = mesmo volume perfície interna do tanque será pintada com
um material impermeabilizante líquido. Cada
metro quadrado pintado necessita de 2 litros
a de impermeabilizante, cujo preço é R$ 2,00 o
H litro. Sabendo-se que AB = 3 m, AE = 6 m e
AD = 4 m, determine:
a. as medidas de EI e HI;
a
a b. a área da superfície a ser pintada e
a quanto será gasto, em reais.
Vp = Vc Resolução
BH = a 3 a. No triângulo retângulo EHI, com ângulo
a2 3 EH
⋅ H = a3 reto no vértice H, EH = AB = 3 m, = sen a ⇔
4 EI
3 3
4a 3 ⇔ = ⇔ EI = 5 m e HI = 52 − 32 = 4 m
H= EI 5
3
b. A área da superfície a ser pintada é igual
Resposta à soma das áreas dos retângulos ABCD, ABHE,
4a 3 DCGF, ADFE, EFJI, e das áreas dos triângulos re-
h=
3 tângulos EHI, FGJ. Logo, é igual a 4 · 3 + 6 · 3 +
4⋅3 4⋅3
05. Vunesp +6·3+4·6+4·5+ + = 104 m2.
2 2
Um tanque para criação de peixes tem a forma Serão gastos, portanto, 104 · 2 · 2 = 416 reais.
da figura abaixo:
C G J

H
B α I
F
D
3m
4m
PV-13-31

A 6m E

39
Matemática Geometria de posição e métrica

CAPÍTULO 04 PIRÂMIDES

1. Generalidades • Superfície lateral: a superfície lateral


de uma pirâmide é a superfície polié-
A. Definição drica formada por todas as faces late-
rais.
Consideremos uma região poligonal convexa
A1 A2 ... An de n lados num plano a, e um ponto • Secção transversal: a secção transver-
V fora de a. Chamamos de pirâmide convexa a sal de uma pirâmide é a intersecção
reunião de todos os segmentos com uma ex- dessa pirâmide com qualquer plano pa-
tremidade em V e a outra nos pontos da região ralelo à sua base.
poligonal (polígono).
V

Secção
transversal β//α
A3
A2

α A1 An
α

B. Elementos
C. Nomenclatura
Na pirâmide da figura temos os seguintes ele-
mentos: O nome de uma pirâmide é dado de acordo
com a sua base, assim, temos alguns exem-
V Vértice
plos:
Aresta lateral • Pirâmide triangular: a base é um triân-
gulo.
Face lateral
Altura • Pirâmide quadrangular: a base é um
F E quadrilátero.
• Pirâmide pentagonal: a base é um pen-
A D tágono.
Aresta da base
Base B C D. Classificação
Uma pirâmide pode ser classificada como reta
• Vértice da pirâmide: o vértice da pirâ- ou oblíqua, conforme a projeção ortogonal do
PV-13-31

mide é o ponto V. vértice sobre o plano da base, seja o centro


• Base: a base da pirâmide é o polígono (circuncentro) da mesma ou não.
ABCDEF. V
• Altura: a altura da pirâmide é a distân-
cia do vértice V ao plano da base.
• Arestas da base: as arestas da base são
os lados do polígono da base. E
• Arestas laterais: as arestas laterais são
os segmentos que unem o vértice V aos A V'
vértices do polígono da base. D
• Faces laterais: as faces laterais são os
triângulos determinados pelo vértice V B C
e cada uma das arestas da base. Pirâmide pentagonal oblíqua

40
Geometria de posição e métrica Matemática

V Consideremos uma pirâmide regular com perí-


metro da base 2p, apótema da base a e apóte-
ma da pirâmide m.

A h
m
V' D

B C a
 
Pirâmide pentagonal reta

Propriedade  ⋅m
A = n ⋅ =
As arestas laterais de uma pirâmide reta são 2
todas congruentes. n⋅  2p
= ⋅m = ⋅m = p ⋅m
Chamamos de pirâmides regulares as pirâmi- 2 2
des retas cujas bases são polígonos regulares. A T = AL + B =
Apótema ⋅a
da = p ⋅m + n⋅ = p ⋅m + p ⋅ a
pirâmide 2
Assim: AT = p (m + a)

3. Volume
A. Teorema
Duas pirâmides que têm alturas iguais e ba-
Apótema ses de áreas iguais têm volumes iguais.
da
pirâmide
Demonstração
Consideremos duas pirâmides de mesma altu-
ra H e com bases de mesma área SB, confor-
me a figura. Sejam S1 e S2 as áreas das secções
transversais que obtemos ao seccionarmos os
dois sólidos por um plano à distância h dos
PV-13-31

vértices das pirâmides.

S1 S2
Chamamos de apótema da pirâmide a distân- h
cia do vértice da pirâmide às arestas da base. H H

2. Área lateral e área total SB


SB
A área lateral de uma pirâmide é a área da sua
superfície lateral, isto é, a soma das áreas das Pirâmide 1 Pirâmide 2
faces laterais.
A partir da semelhança dos sólidos, concluímos
A área total de uma pirâmide é a soma da área que S1 = S2, pelo princípio de Cavalieri, têm vo-
lateral com a área da base. lumes iguais, nas duas pirâmides.

41
Matemática Geometria de posição e métrica

B. Pirâmide triangular F

O volume de uma pirâmide triangular é D


igual a um terço do produto da área de sua
base pela sua altura. E

H C
S ⋅H
V= B
3 D

Demonstração
3 C
Consideremos um prisma com a mesma base
e altura de uma pirâmide triangular.
F
D D
B
E
H As pirâmides 1 e 2 têm volumes iguais, pois
C C suas bases ABC e DEF têm áreas iguais e am-
A
bas as pirâmides possuem a mesma altura (a
A
própria altura do prisma).
SB B SB B
Assim: V1 = V2 (I)
Vamos decompor esse prisma em três pirâmides Observemos agora as pirâmides 2 e 3.
(1, 2 e 3), conforme a figura a seguir, e provar
que essas três pirâmides têm volumes iguais. Consideremos como bases os triângulos FEC
F e BCE. As áreas desses triângulos são iguais,
D
pois cada um deles é “metade” da face BCFE
SB
do prisma.
E
Além disso, as pirâmides 2 e 3 têm a mesma
H altura (distância do vértice D ao plano da face
BCFE do prisma).
PV-13-31

C
SB Assim: V2 = V3 (II)
A
Então: V1 = V2 = V3
B
Portanto, o volume de cada uma das pirâmi-
D 1
des é igual a do volume do prisma.
1 3
Particularmente, como a pirâmide (1) tem a
mesma base e a mesma altura do prisma, con-
C cluímos que:
A 1
V1 = ⋅ SB ⋅ H
3
B

42
Geometria de posição e métrica Matemática

C. Pirâmide qualquer As quatro faces do tetraedro são triângulos


equiláteros de lado a, então:
O volume de qualquer pirâmide é igual a um a2 3
terço do produto da área de sua base pela AT = 4 ⋅ A∆ = 4 ⋅
altura. 4
\ A T = a2 3
Demonstração
Consideremos uma pirâmide qualquer e uma
No ∆VGA, temos:
pirâmide triangular, que tenham a mesma al-
tura e bases com mesma área SB. 2 a 3 a 3
AG = ⋅ ⇒ AG = ; VA = a
3 2 3
1 2
e VG = altura H
H
VA2 = VG2 + AG2
2
 a 3
a = (H) + 
2 2

 3 
3a2 6a2
SB SB ⇒ H2 = a2 − =
9 9
No item A mostramos que V1 = V2. Como o vo- a 6
lume da pirâmide triangular é: \ H=
3
SB ⋅ H
V1 =
3 O volume do tetraedro é dado por:

Temos que: a2 3 a 6
S ⋅H ⋅
V= B = 4 3
SB ⋅ H 3 3
V2 =
3
a3 2
\ V=
12
4. Sólidos especiais
B. Octaedro regular
A. Tetraedro regular
Consideremos um octaedro regular ABCDEF
Consideremos um tetraedro regular VABC de de aresta com medida a:
PV-13-31

aresta com medida a:


A
V
a
a a
a
H C a
a E
B a D
O
A a
G C
a a a a
B
F

43
Matemática Geometria de posição e métrica

As oito faces do octaedro são triângulos equi- V


láteros de lado a, então: c
a
a2 3 C
AT = 8 ⋅ A∆ = 8 ⋅ A
A b
4
\ A T = 2a2 3 a
V B
c
C
Sendo O o centro do octaedro, O também é o b
centro do quadrado ABFD, então:
diagonal a 2 B
OA = OB = =
2 2
Para calcularmos o volume desse sólido, pode-
Assim, o volume do octaedro é dado por: mos imaginá-lo com base VBC, assim a altura
V = 2 · Vpirâmide será VA. Dessa maneira:
a 2 S VBC ⋅ VA
SB ⋅ H a2 ⋅ V=
V = 2⋅ = 2⋅ 2 3
3 3 b⋅c
Onde: S VBC = e VA = a
2
a3 2 b⋅c
\ V= ⋅a
3
Logo : V = 2
3
C. Tetraedro trirretangular a⋅b ⋅ c
Consideremos um tetraedro trirretangular \ V=
6
VABC com as arestas do triedro trirretângulo
de medidas a, b e c.

EXERCÍCIOS RESOLVIDOS

01. Resolução
PV-13-31

Determine a altura de uma pirâmide regular Não importa o tipo de pirâmide, a altura será:
cujo apótema mede 13 cm, sendo o apótema
da base 5 cm.
a. 12 cm
h g
b. 10 cm
c. 11 cm
d. 15 cm
a
g = h + a ⇒ 13 = h + 52 ⇒ h = 12 cm
2 2 2 2 2

Resposta
A

44
Geometria de posição e métrica Matemática

02. Fuvest-SP
( )
2
c. m2 = a + h2 ⇒ 62 = 2 2 + h2 ⇒
Qual a altura de uma pirâmide quadrangular
⇒ h = 2 7 dm
que tem as oito arestas iguais a 2?
2
a. 1  
( )
2
d. f = m +   ⇒ f = 6 + 2 2 ⇒
2 2 2 2
b. 1, 5 2
c. 3 ⇒ f = 2 11 m

d. 2, 5  ⋅m
e. 4 ⋅ ⇒ A  = 2 ⋅ 4 2 ⋅ 6 ⇒
e. 5 2
⇒ A  = 48 2 dm2
Resolução
2
  2
a2 = h2 +  f. A T = B + A = 32 + 48 2 ⇒
 2 
2
(
⇒ A T = 16 2 + 3 2 dm2 )
 2⋅ 2
( 2)
2
= h2 +  04. Mackenzie-SP
 2 
Uma barraca de lona tem forma de uma pirâ-
h= 1 mide regular de base quadrada com 1 metro
de lado e altura igual a 1,5 metro. Das alterna-
Resposta
tivas abaixo, a que indica a menor quantidade
A suficiente de lona, em m2, para forrar os qua-
03. tro lados da barraca é:
De uma pirâmide regular de base quadrada a. 2
sabe-se que a área da base é 32 dm2 e que o b. 2,5
apótema da pirâmide mede 6 dm. Calcule: c. 4,5
a. a aresta da base (l); d. 3,5
b. o apótema da base (a); e. 4
c. a altura da pirâmide (h); Resolução
d. a aresta lateral (f); V
e. a área lateral (Al);
f. a área total (At).
Resposta
1,5
D C
PV-13-31

f
O 0,5 M
h m 1
A 1 B

No triângulo VOM, temos:


l 10
a (VM)2 = (0,5)2 + (1,5)2 ⇔ VM = m
2
Sendo Al a área lateral da pirâmide, temos:
B = 32 dm2 ⇒ l2 = 32 ⇒ l = 4 2 dm (BC) ⋅ (VM) 10
A = 4 ⋅ = 2 ⋅1⋅ = 10  3,1
a. l = 4 2 dm 2 2
 Resposta
b. a = ⇒ a = 2 2 dm
2 D

45
Matemática Geometria de posição e métrica

05. B
A área lateral de uma pirâmide quadrangular A
P
regular é de 150 cm2 e o perímetro da base é
20 cm. Calcule o volume dessa pirâmide.
V
V

h m
D C
Cada um dos pontos A e B dista de P:
h m
a a. 5,75 cm d. 1,5 cm
M
O b. 4,25 cm e. 0,75 cm
A B O a M c. 3,75 cm

V
Resolução
B
D C A x
x
P
m f O M
a

10
B M C A B 10
l l
Face lateral Base Seja x (m) a distância dos pontos A e B até o
ponto P.
Resolução
O volume do cubo em metros cúbicos, é:
AL = 150 cm2 2p = 20 cm
Vcubo = 103 = 1.000
 ⋅m
4· = 150 4l = 20 O volume da pirâmide em metros cúbicos, é:
2
2 · 5 · m = 150 l = 5 cm 1 x⋅x 5x 2
m = 15 cm
Vpirâmide = ⋅ ⋅ 10 =
2 3 2 3
 5 5 35
m2 = a2 + h2 ⇒ 152 =   + h2 ⇒ h = cm Como Vpirâmide = 0 , 375% ⋅ Vcubo , temos:
 2 2
5x 2 0 , 375 225
B ⋅ H 52 ⋅ 5 35 1 125 35 = ⋅ 1.000 ⇒ x 2 = ⇒ x = 1, 5
V= ⇒ ⋅ = ⇒ 3 100 100
3 2 3 6
PV-13-31

Resposta
125 35
V= cm3 D
6
07. PUC-SP
06. FGV-SP A distância de um ponto do espaço ao plano
Um cubo de aresta de 10 cm de comprimen- de um triângulo equilátero ABC de lados 6 m
to deve ser seccionado como mostra a figura, equidistante 4 m de cada vértice mede:
de modo que se obtenha uma pirâmide cuja a. 1 m d. 4 m
base APB é triangular isósceles e cujo volume b. 2 m e. 5 m
é 0,375% do volume do cubo.
c. 3 m

46
Geometria de posição e métrica Matemática

Resolução 202 3
B= = 100 ⋅ 3 ≈ 100 ⋅ 1, 73
4
m B ≅ 173 cm2
4 h
202 3
6 AL = 3 ⋅ = 3 ⋅ 100 ⋅ 3 ≈ 3 ⋅ 100 ⋅ 1, 73
4
o m
AL = 519 cm2
C Custo total: 519 · 30 + 173 · 5
 2 6 3
2 C = 16.435 em reais
42 =  · + h2
 3 2  Resposta

16 = 12 + h2 C
h= 2m 09.
Resposta Em uma pirâmide regular hexagonal, a aresta
B da base mede 4 ⋅ 3 cm e a altura é de 8 cm.
Calcule:
08. Mackenzie-SP V
Um objeto, que tem a forma de um tetraedro E D
regular reto de aresta 20 cm será recoberto
com placas de ouro nas faces laterais e com d
F O C
placa de prata na base. Se o preço do ouro
é R$ 30,00 por cm2 e o da prata R$ 5,00 por d a  E D
cm2, das alternativas dadas, assinale o valor
F O C
mais próximo, em reais, do custo desse reco- A B
brimento. M
Base A B
a. 24.000
b. 12.000 V
c. 16.000
d. 18.000
e. 14.000 f

Resolução h m m

B M C
PV-13-31

O a M 
20 cm a. o apótema da base;
b. o apótema da pirâmide;
c. a aresta lateral;
d. a área da base;
e. a área lateral;
20 cm
20 cm f. a área total;
g. o volume.

47
Matemática Geometria de posição e métrica

Resolução 10. UEM-PR


Uma pirâmide de chumbo é mergulhada num
tanque cúbico de aresta 1 m, cheio de água até
a borda. Se a base da pirâmide é um triângulo
retângulo cujos catetos medem 0,5 m e se sua
8
altura também é de 0,5 m, então o volume de
água derramada foi:
1 1 3
a. m3 d. m
12 48
1 3 1 3
b. m e. m
4⋅ 3 24 64
1 3
 ⋅ 3 4⋅ 3⋅ 3 c. m
a. a = = ⇒ a = 6 cm 36
2 2
Resolução
b. m = a + h ⇒ m = 6 + 8 ⇒
2 2 2 2 2 2
Catetos: 0,5 m
m2 = 10 cm Altura: 0,5 m
c. f 2 = (2 ⋅ 3)2 + m2 = 12 + 100 ⇒
f = 4 ⋅ 7 cm

m f
Vágua derramada = Vpirâmide
1 1 b ⋅h 1 0, 5 ⋅ 0, 5
Vágua = ⋅ B ⋅ H = ⋅ ⋅H = ⋅ ⋅ 0, 5
3 3 2 3 2
1
2 1 3
Vágua = m
2 ⋅ 3 3 ⋅ (4 ⋅ 3)2 ⋅ 3 48
d. B = 6 ⋅ = ⇒ Resposta
4 2
B = 72 ⋅ 3 cm D
 ⋅m 11. Vunesp
e. AL = 6 ⋅ = 3 ⋅ 4 ⋅ 3 ⋅ 10 ⇒ 1
2 O volume de um tetraedro regular é m3 . Sua
aresta mede: 3
AL = 120 ⋅ 3 cm
PV-13-31

2 2 2
f. A T = B + AL = 72 ⋅ 3 + 120 ⋅ 3 ⇒ a. m d. m
3 3
A T = 192 ⋅ 3 cm2 2 3 2
b. m e. m
B ⋅H 72 ⋅ 3 ⋅ 8 2 2
g. V = ⇒ V= ⇒
3 3
c. 2 m
V = 192 ⋅ 3 cm3

48
Geometria de posição e métrica Matemática

Resolução 12. FGV-SP


Um octaedro regular está inscrito num cubo
de aresta com 4 cm de comprimento, isto é,
seus vértices coincidem com o centro de cada
a h face do cubo, como mostra a figura. O volume
do octaedro é:
a 3 1

a 3 2 2 3

2 3

2
 a 3 2
a2 = h2 + ⋅
 2 3 
a2 64 3 8 3
a2 = h2 + a. cm d. cm
3 3 3
2a2 32 3 4
h= b. cm e. cm3
3 3 3
a 6 16 3
h= c. cm
3 3
1 Resolução
V = ⋅ Ab ⋅ h
3 2
1 a2 3 a 6 1
V= ⋅ ⋅ = d d
3 4 3 3 2
a 18 1
3
= 4
36 3 d
d
a ⋅3 2
3
=1
12
Sendo l a medida, em centímetros, da aresta
4 2
a3 = ⋅ do octaedro regular, temos:
2 2
l2 = 22 + 22 ⇒ l2= 8 ⇒ l2 = 2 2
4 2
a3 = =2 2 O volume do octaedro regular é o dobro do
2 volume da pirâmide regular VABCD de aresta da
PV-13-31

a3 = 8 base 2 2 cm e altura 2 cm.


a3 = 2
3 Assim, sendo V o volume do octaedro, em cen-
tímetros cúbicos, temos:
a = 2m 1 32 3
V = 2 ⋅ ⋅ (2 2 )2 ⋅ 2 = cm
Resposta 3 3
Resposta
C
B

49
Matemática Geometria de posição e métrica

CAPÍTULO 05 CILINDROS

1. Generalidades Base
O'
A. Definição
Consideremos num plano a um círculo de cen- g (geratriz)
tro O e raio r. Seja PQ um segmento não para- Eixo h (altura)
lelo e não contido em a. Chamamos de cilin-
dro circular à reunião de todos os segmentos
congruentes e paralelos a PQ, com uma extre-
O r
midade no círculo e situados num mesmo se-
miespaço dos determinados por a. Base

P • Superfície lateral
Superfície lateral é a reunião de todas as gera-
trizes. A área dessa superfície é chamada área
lateral e indicada por AL.
• Superfície total
Q
Superfície total é a reunião da superfície late-
ral com os dois círculos das bases. A área dessa
superfície é chamada área total e indicada por
O r AT.
• Secção meridiana
α
Secção meridiana de um cilindro é o quadri-
látero que obtemos na interseção do cilindro
B. Elementos com um plano que contém o eixo.
• Bases o meridia
cçã na
As bases de um cilindro circular são os dois Se
círculos determinados pelas extremidades de
todos os segmentos paralelos a PQ que, reuni-
dos, formam o cilindro.
• Eixo g
Eixo de um cilindro circular é a reta determina-
da pelos centros das bases do cilindro.
PV-13-31

• Geratriz 2r
Geratriz (g) de um cilindro circular é qualquer • Secção longitudinal
um dos segmentos com extremidades nas cir-
cunferências das bases e pararelos ao eixo do A secção longitudinal de um cilindro é obtida
cilindro. pela intersecção com um plano que contém o
eixo do cilindro ou é paralelo a ele. Quando o
• Altura cilindro é circular reto, a secção longitudinal é
Altura de um cilindro é a distância h entre os um retângulo e será máxima quando a secção
planos das bases. for meridiana.

50
Geometria de posição e métrica Matemática

g
ção lon itudinal 2. Áreas lateral e total
Sec
O A. Área lateral
A superfície lateral de um cilindro circular reto
é equivalente a um retângulo cujas dimensões
são o comprimento da circunferência da base
e a altura do cilindro.
r
O'

• Secção transversal
A secção transversal de um cilindro circular h
reto é obtida pela intersecção com um plano
perpendicular ao eixo do cilindro. A secção
transversal é um círculo congruente à base.
O

r
h

O'

C. Classifi cação 2πr


• Cilindro circular oblíquo
Assim, a área lateral de um cilindro circular
Cilindro circular oblíquo é aquele que apresen- reto com raio da base r e altura h é:
ta as geratrizes oblíquas aos planos das bases.
AL = 2 · p · r · h
• Cilindro circular reto
Cilindro circular reto é aquele que apresenta
as geratrizes perpendiculares aos planos das B. Área total
bases. O cilindro circular reto também é cha- A área de superfície total de um cilindro circu-
mado cilindro de revolução, pois é gerado lar reto é a soma das áreas da superfície lateral
pela rotação de um retângulo em torno de um com as áreas das duas bases.
eixo que contém um de seus lados.
Assim, a área total de um cilindro circular reto
Cilindro oblíquo Cilindro reto com raio da base r e altura h é:
PV-13-31

AT = AL + 2 B
AT = 2p · r · h + 2p · r2
AT = 2pr · (h + r)

h h=g
Exemplos
01. Num cilindro circular reto de altura 6 cm e
raio da base 2 cm, calcular:
a. a área de uma secção transversal;
Cilindro de revolução b. a área de uma secção meridiana;

51
Matemática Geometria de posição e métrica

c. a área de uma secção feita por um pla- b. Semicilindro reto


no paralelo ao eixo distante 1 cm dele.
Resolução
a.
8 cm

2 cm

3 cm
Resolução
S = p · r = p · 2 = 4p cm2
2 2
a. AL = 2p · r · h = 2p · 1 · 3 = 6p cm2
b. AT = 2p · r · h + 2p · r2
4 cm AT = 2p · 1 · 3 + 2p · 12 = 8p cm2

b. Superfície lateral do semicilindro
6 cm
8 cm


6 cm π · 3 cm
S = 4 · 6 = 24 cm 2

c. AL = 6 · 8 + p · 3 · 8 = 48 + 24p
1
O AL = 24 · (2 + p) cm2

π ⋅r2
AT = AL + 2 · = 48 + 24p + p · 32
x 2
1
AT = (48 + 33p) cm2
x
2
03. A área lateral de um cilindro de revolução
de 10 cm de raio é igual à área da base. Calcule
a altura do cilindro.
x 2 + 1 2 = 22
Resolução
\ x = 3 cm
S = 2x · h = 2 3 · 6 = 12 3 cm2
02. Calcule a área lateral e a área total dos sóli-
PV-13-31

dos, cujas medidas estão indicadas nas figuras h


abaixo.
a. Cilindro de revolução

10 cm
3 cm AL = B
2p · r · h = p · r2
2h = r
2h = 10
1 cm \ h = 5 cm

52
Geometria de posição e métrica Matemática

3. Volume 02. Calcular o volume de um cilindro inscrito


em um prisma quadrangular regular de aresta
Consideremos um cilindro e um prisma de ba-
da base 4 cm e altura 10 cm.
ses equivalentes (áreas iguais) contidas nos
mesmos planos paralelos (alturas iguais). Resolução
Sendo B a área das bases e h a altura dos sóli- r
dos considerados, temos:

r
10 cm 4 cm
h
Secções
área = B
β 4 cm
4 cm
Bases 4 cm
área = B
α
r = 2 cm e h = 10 cm
Um plano paralelo às bases, que secciona os V = p · r2 · h
dois sólidos, determina neles secções tranver-
V = p · 22 · 10 = 40p cm3
sais congruentes às respectivas bases. Pelo
princípio de Cavalieri, concluímos que o cilin- 03. Calcular a massa de ouro utilizado na alian-
dro e o prisma são equivalentes. ça indicada na figura, sabendo que a massa es-
Assim: pecífica do ouro é 20 g/cm3.
V = B · h = p · r2 · h Dados
Raio externo = 22 mm
Exemplos
Raio interno = 21,5 mm
01. Calcule o volume do cilindro oblíquo abai-
Altura = 3 mm
xo, em função da geratriz g.

mm
22 21,5 mm
g 3 mm

60°

g/2 Resolução
PV-13-31

Resolução O volume da aliança é a diferença entre os


volumes de dois cilindros circulares retos de
raios 22 mm e 21,5 mm e altura 3 mm. Assim:
g V = p · 222 · 3 – p · (21,5)2 · 3 ≅ 204,9 mm3
h
V ≅ 0,2049 cm3
Sendo a massa específica 20 g/cm3, temos que
60° em cada cm3 existem 20 g de ouro, então a
3 massa m é:
h = g · sen 60° = ·g m ≅ 0,2049 · 20
2
V = p · r2 · h m≅4g
2
 g   3  π ⋅ 3 ⋅ g3
V = π  ⋅ ⋅g =
 2   2  8

53
Matemática Geometria de posição e métrica

04. A figura mostra um rolo de papel com es- Resolução


pessura 0,2 mm. Calcule o comprimento apro- Sendo V1 o volume do papel enrolado, temos:
ximado do papel enrolado, sabendo que a me-
nor e a maior voltas têm raios 2 cm e 5 cm, V1 = p · 52 · h – p · 22 · h = 21p · h
respectivamente. Sendo V2 o volume do papel esticado, temos:

V2 = x · h · 0,02 (*)
(*) prisma de altura 0,02 cm
Como V1 = V2, temos:
21π
21π ⋅ h = x ⋅ 0, 02 ⋅ h ⇒ x =
0, 02
x ≅ 3.297 cm ≅ 33 metros

EXERCÍCIOS RESOLVIDOS

01. Vunesp 02. FGV-SP


Um produto é acondicionado em três tipos de Deseja-se construir um reservatório cilíndrico
embalagens cilíndricas, todas de mesma altu- com tampa, para armazenar certo líquido. O
ra, mas de raios a, b e c, distintos entre si. Se volume do reservatório deve ser de 50 m3 e
a capacidade da embalagem de raio c é igual à o raio da base do cilindro deve ser de 2 m. O
soma da capacidade da embalagem de raio a material usado na construção custa R$ 100,00
com a de raio b, prove que c2 = a2 + b2. por metro quadrado. Qual o custo do material
Resolução utilizado?
Vc = Va + Vb Resolução
p c2h = p a2h + p b2h (mesma altura) Seja h a altura do reservatório. Temos:
25
c2 = a2 + b2 p · 22 · h = 50 ⇔ h = m
PV-13-31

2p
Assim, supondo o cilindro reto, temos que a
área total do mesmo é:
25
2 · p · 22 + 2p · 2 · = 8p + 50 m2
2p
Logo, o custo do material utilizado para a cons-
trução do cilindro é 100 (8p + 50) ≅ 7.513,27
reais.

54
Geometria de posição e métrica Matemática

03. ENEM Virando a garrafa de cabeça para baixo, sua


Uma garrafa cilíndrica está fechada, contendo capacidade total é igual ao volume do líquido
um líquido que ocupa quase completamente mais o volume da parte não ocupada por ele.
seu corpo, conforme mostra a figura. Suponha Assim, além das duas medidas anteriores, bas-
que, para fazer medições, você disponha ape- ta medir apenas a altura h’ da parte não ocu-
nas de uma régua milimetrada. pada pelo líquido. Logo, o número mínimo de
medições é igual a 3.
Resposta
C
05. FGV-SP
Inclinando-se em 45° um copo cilíndrico reto
de altura 15 cm e raio da base 3,6 cm, derra-
ma-se parte do líquido que completava total-
mente o copo, conforme indica a figura.
Para calcular o volume do líquido contido na
garrafa, o número mínimo de medições a se-
rem realizadas é:
a. 1 c. 3 e. 5
b. 2 d. 4
Resolução
Para calcular o volume do líquido contido na
garrafa, precisamos conhecer o raio R da base
do cilindro reto e a sua altura h, pois esse volu- Admitindo-se que o copo tenha sido inclinado
me é dado por pR2h. com movimento suave em relação à situação
inicial, a menor quantidade de líquido derra-
Logo, o número mínimo de medições é igual mada corresponde a um percentual do líquido
a 2. contido inicialmente no copo de:
Resposta a. 48% c. 28% e. 18%
B b. 36% d. 24%
04. ENEM Resolução
Para calcular a capacidade total da garrafa, Consideremos a figura a seguir:
lembrando que você pode virá-la, o número D
mínimo de medições a serem realizadas é:
a. 1 c. 3 e. 5
PV-13-31

b. 2 d. 4 C A
Resolução
h’ h
F B

Líquido
45°
E

V1 → volume inicial
V2 → volume derramado

55
Matemática Geometria de posição e métrica

1. V1 = π ⋅ (3, 6)2 ⋅ 15 = 194 , 4 π cm3 b. 100 cm

h 60 cm
2. tg 45° ⇒ = 1 ⇒ h = 7,2 cm
7, 2
1
3. V2 = π ⋅ 3, 6 ⋅ 7,2 = 46, 656π cm3
2 c. 100 cm
V2 46, 656π
= = 0,24 = 24% 60 cm
V1 194 , 4 π
Resposta
D d. 60 cm
06. ENEM
Alguns testes de preferência por bebedouros
de água foram realizados com bovinos, envol- 100 cm
vendo três tipos de bebedouros, de formatos
e tamanhos diferentes. Os bebedouros 1 e 2
têm a forma de um tronco de cone circular
reto, de altura igual a 60 cm, e diâmetro da e. 60 cm
base superior igual a 120 cm e 60 cm, respec-
tivamente. O bebedouro 3 é um semicilindro,
com 30 cm de altura, 100 cm de comprimento
e 60 cm de largura. Os três recipientes estão 100 cm
ilustrados nas figuras.
120 cm 60 cm

60 cm
60 cm 60 cm
Resolução
Bebedouro 3:
Bebedouro 1 Bebedouro 2 60 cm
60 cm
100 cm
30 cm

100 cm 30 cm
100 cm
Bebedouro 3

A escolha do bebedouro. In: Biotemas. Planificação do bebedouro 3:


PV-13-31

v. 22, n. 4, 2009. Adaptado. 60 cm

Considerando que nenhum dos recipientes te-


nha tampa, qual das figuras a seguir represen-
ta uma planificação para o bebedouro 3?
a.
100 cm 100 cm

60 cm

60 cm
Resposta
E

56
Geometria de posição e métrica Matemática

07. ENEM Resolução


A ideia de usar rolos circulares para 2V
deslocar objetos pesados provavelmente
surgiu com os antigos egípcios ao constru-
írem as pirâmides. V

V=0

A velocidade do ponto superior do cilindro,


em relação à terra, é o dobro da velocidade do
BOLT, Brian. Atividades matemáticas. Ed.Gradiva. centro do cilindro.
Portanto, y = 2 · 2pR = 4pR
Representando por R o raio da base dos rolos
cilíndricos, em metros, a expressão do deslo- Resposta
camento horizontal y do bloco de pedra em E
função de R, após o rolo ter dado uma volta
completa sem deslizar, é:
a. y = R
b. y = 2 R
c. y = pR
d. y = 2pR
e. y = 4pR
PV-13-31

57
Matemática Geometria de posição e métrica

CAPÍTULO 06 CONES

1. Generalidades C. Classificação
• Cone circular oblíquo
A. Definição
V
Consideremos um círculo contido num plano
e um ponto P fora desse plano. Chamamos de
cone circular, ou simplesmente cone, a reu-
nião de todos os segmentos que tem uma ex-
tremidade em P e a outra num ponto qualquer
do círculo. O
P V'

Cone circular oblíquo é aquele que apresenta


o eixo oblíquo em relação ao plano da base.
Num cone circular oblíquo, a projeção ortogo-
nal do vértice no plano da base não é o seu
centro.
α
• Cone circular reto

B. Elementos V
• Base
A base do cone é o círculo considerado na de-
finição.
• Vértice
O vértice do cone é o ponto P.
O
• Eixo V' 0
O eixo de um cone é a reta que passa pelo vér-
tice P e pelo centro da base.
• Geratriz Cone circular reto é aquele que apresenta
A geratriz de um cone é qualquer segmento o eixo perpendicular ao plano da base. Num
que tenha uma extremidade no vértice do cone circular reto, a projeção ortogonal do
cone e a outra na circunferência da base. vértice no plano da base é o centro da base.
PV-13-31

• Altura Observações
A altura é a distância do vértice do cone ao 01. Num cone reto, todas as geratrizes são
plano da base. congruentes entre si, então a secção
meridiana é um triângulo isósceles.
P
Vértice

g g
Altura Geratriz

O
2R
Eixo Base

58
Geometria de posição e métrica Matemática

02. Todo cone reto pode ser definido como B. Área lateral
o sólido gerado pela rotação de um
triângulo retângulo em torno de um de
seus catetos.
g
Assim, o cone reto também é chamado de
cone de revolução.

03. Um dos catetos desse triângulo será a


altura do cone (h), o outro será o raio
(R) e a hipotenusa será uma geratriz (g).
Então:
g
g θ
h

R
g = h + R2
2 2
2πR

2. Áreas lateral e total Consideremos um cone circular reto (cone de


revolução) de geratriz g e raio da base R. Pla-
A. Fórmulas do setor circular nificando a superfície lateral desse cone, ob-
Consideremos um setor circular de raio R e ân- temos um setor circular de raio g e cujo arco
gulo central θ radianos. correspondente tem comprimento igual a 2pR
(comprimento da circunferência da base).
R Para θ em radianos, temos:
PV-13-31

O θ  2π ⋅ R
θ · g = 2p · R ⇒ θ =
R g

A área total do cone é dada por:


O comprimento l do arco desse setor circular
é dado por: 2π ⋅ R ⋅ g
AL = ⇒ AL = p · R · g
2
l = θ · R (θ em radianos)
C. Área total
A área S do setor circular é dada por:
A superfície total de um cone de revolução é
 ⋅R formada pela superfície lateral (setor circular)
S= e pelo círculo da base. Assim, num cone de ge-
2
ratriz g e raio da base R, temos:

59
Matemática Geometria de posição e métrica

2πR
R

AT = AL + SB ⇒ AT = p · R · g + p · R2
AT = p · R · (g + R)

Exemplo 3. Volume
Num cone de revolução de geratriz 13 cm e Utilizando o princípio de Cavalieri, verificamos
raio da base 5 cm, calcular: que um cone e uma pirâmide, cujas alturas são
a. a altura; iguais e as bases são equivalentes, têm volu-
b. a área lateral; mes iguais.
c. a área total;
d. o ângulo θ, em radianos, da superfície
lateral planificada. H
Resolução

SB SB
g = 13
V cone = V pirâmide
H
Assim, o volume de um cone é igual a um terço
do produto da área da base do cone pela sua
altura.
R=5 1
V = ⋅ SB ⋅ H
PV-13-31

a. g = H + R
2 2 2 3
132 = H2 + 52 \ H = 12 cm 1
V = ⋅ ( π ⋅ R2 ) ⋅ H
3
b. AL = p · R · g = p · 5 · 13
AL = 65p cm2 π ⋅ R2 ⋅ H
V=
3
c. AT = AL + SB = 65p + p · 52
AT = 90p cm2 Exemplo
A superfície lateral de um cone de revolução
d. θ · g = 2p · R planificada é a terça parte de um círculo de
10π raio 6 cm. Calcular o volume desse cone.
θ · 13 = 2p · 5 ⇒ θ = rad
13

60
Geometria de posição e métrica Matemática

Resolução 2π
g = 6 cm e θ = rad
3
θ ⋅ g = 2π ⋅ R

⋅ 6 = 2π ⋅ R ⇒ R = 2 cm
3
g g2 = R2 + H2
H 62 = 22 + H2 ⇒ H2 = 32
6 cm ∴H = 4 2 cm

rad π ⋅ R2 ⋅ H
3 V=
R 3
π ⋅ 22 ⋅ 4 2
V=
3

16π 2 3
∴V = cm
3

EXERCÍCIOS RESOLVIDOS

01. ENEM a. pirâmide.


A figura seguinte mostra um modelo de som- b. semiesfera.
brinha muito usado em países orientais. c. cilindro.
d. tronco de cone.
e. cone.
Resolução
De acordo com a figura mostrada, temos a re-
presentação de uma superfície de revolução
chamada cone.
Resposta
E
PV-13-31

Disponível em: <http://mdmat.psico.


ufrgs.br>. Acesso em: 1 maio 2010.

Esta figura é a representação de uma superfí-


cie de revolução chamada:

61
Matemática Geometria de posição e métrica

02. UFAM • Se R > h ⇒ volume diminui.


Um tanque cônico tem 4 m de profundidade e • Se R < h ⇒ volume aumenta.
seu topo circular tem 6 m de diâmetro. Então, Resposta
o volume máximo de líquido, em litros, que
esse tanque pode conter é: E
(Use p = 3,14) 04. Mackenzie-SP
a. 24.000 d. 14.000 Planificando a superfície lateral de um cone,
b. 12.000 e. 37.680 obtém-se o setor circular da figura, de centro
O e raio 18 cm. Dos valores a seguir, o mais
c. 37.860 próximo da altura desse cone é:
Resolução 0
3
160°

a. 12 cm d. 16 cm
b. 18 cm e. 20 cm
c. 14 cm
1
Vtan que = ⋅ π ⋅ r2 ⋅ h Resolução
3
1 Sendo g, h e r, respectivamente, as medidas
Vtan que = ⋅ π ⋅ 32 ⋅ 4 da geratriz, da altura e do raio da base desse
3 cone, temos:
O
Vtanque = 12 · 3,14 = 37,68 m3
1 m3 = 1.000 L 0
g = 18
g
Então, o volume máximo do tanque é 37.680 160° A
h
litros.
Resposta A
C r 2 πr
E
03. FEI-SP Temos:
2p · 18 l 360°
Num problema em que se pedia o volume de 2p · r l 160°
um cone reto, o aluno trocou, entre si, as me- \r=8
PV-13-31

didas do raio e da altura. Pode-se então afir-


mar que o volume do cone: Do triângulo retângulo OCA, vem:
a. não se alterou. h2 + r2 = g2
b. duplicou. h2 + 82 = 182 ∴ h = 260
c. triplicou. Temos:
d. diminuiu. 256 < 260 < 289
e. nada pode ser afirmado.
16 < 260 < 17
Resolução Logo, dos valores apresentados nas alternati-
1 vas, o mais próximo é 16.
V = ⋅ π ⋅ R2 ⋅ h
3 Resposta
• Se R = h ⇒ volume não se altera. D

62
Geometria de posição e métrica Matemática

05. UFMG Resolução


Um cone é construído de forma que:
• sua base é um círculo inscrito em uma
face de um cubo de lado a; a
• seu vértice coincide com um dos vérti-
ces do cubo localizado na face oposta
àquela em que se encontra a sua base. r
a
Dessa maneira, o volume do cone é de:
a
pa3
a. Sendo h a altura do cone, temos:
6
1
pa3 V = ⋅ π ⋅ r2 ⋅ h
b. 3
12 1
2
 a
V = ⋅ π ⋅  ⋅a
pa3 3  2
c.
9 π ⋅a3
V=
12
pa3
d. Resposta
3
B

4. Cilindro e cone equiláteros Volume


V = AB · h = pR2 · 2R
A. Cilindro equilátero
Cilindro equilátero é um cilindro de revolução V = 2pR3
cuja secção meridiana é um quadrado (h = 2R).
Cilindro B. Cone equilátero
equilátero
Cone equilátero é um cone de revolução cuja
secção meridiana é um triângulo equilátero
2R
(g = 2 R).

h =2R
g =2R g =2R
PV-13-31

Área lateral
AL = 2p R · h = 2pR · 2R 2R

AL = 4pR2 2R 3
h= ⇒ h=R 3
2
Área total Área lateral
AT = 2 · AB + AL = 2pR + 4pR
2 2
AL = pRg = pR · 2 R
AT = 6 pR 2 AL = 2pR2

63
Matemática Geometria de posição e métrica

Área total
2πR
AT = AL + AB = 2pR2 + pR2 θ=
g
AT = 3pR2
Como, para o cone equilátero, g = 2R:
Volume 2πR
1 1 θ= ⇒ θ=p
V = AB ⋅ h = πR2 ⋅ R 3 2R
3 3
Portanto, a superfície lateral planificada de
πR3 3 um cone equilátero é um semicírculo.
V=
3
θ=π
Lembrando que o ângulo θ, em radianos, da
superfície lateral planificada é dado por:

EXERCÍCIOS RESOLVIDOS

01. ITA-SP Resolução


Qual o volume de um cone circular reto, se a AL = 24p cm2 e r = 4 cm
área de sua superfície lateral é 24p cm2 e o AL = 24p
raio de sua base é 4 cm?
16 pRG = 24p
a. 20p cm3 4 G = 24
3
24 G = 6 cm
b. p cm3
4 G2 = h2 + r2
24 62 = h2 + 42
c. p cm3
4 h2 = 36 – 16
8
d. 24 p cm3 h = 2 5 cm
3
PV-13-31

1 B ⋅H
e. 20p cm3 V=
3 3
π ⋅ 42 ⋅ 2 5
V=
3
16
V= 20 π cm3
3
Resposta
A

64
Geometria de posição e métrica Matemática

02. Cesgranrio-RJ 03. UFPE


Para construir uma piscina cilíndrica, com fun- Um plano que passa pelo vértice de um cone
do circular, cava-se, num terreno plano, um reto intercepta o círculo da base deste cone
buraco com raio R e profundidade R/4. A ter- em uma corda de comprimento 6. Esse plano
ra fofa, retirada do buraco, ocupa um volume forma com o plano da base do cone um ângulo
20% maior que o do buraco cavado e é amon- de 40° e a altura do cone é 3,36. Indique o in-
toada na forma de um cone de revolução. Su- teiro mais próximo do volume do cone.
pondo-se que o raio r da base do cone é igual (Dado: use as aproximações tg(40°) ≅ 0,84 e
à sua altura, então a melhor aproximação da p ≅ 3,14.)
razão r/R é:

r
R
r R/4

1
a.
2
b. 1

c. 1,2
Resolução
p
d. V
2
e. 3

Resolução
R π 3
Vpiscina = πR2 ⋅ H = πR2 ⋅ = R 3,36
4 4
πr2 ⋅ H πr2 ⋅ r π 3
Vcone = = = r
3 3 3
A
Vcone = 1,2
2 Vpiscina 40°
3
3 O 3P
π ⋅ r 3 6 π ⋅ R3 r3 9  r
= ⋅ ⇒ 3= ⇒   = 0, 9 ⇒ B
3 5 4 R 10  R 
3 No ∆VOP, temos:
 r r r
⇒   ≅1⇒ = 1 ⇒ =1 3, 36 3, 36
 R R R tg40° = ⇒ 0, 84 = ⇒ OP = 4
PV-13-31

OP OP
Resposta
No ∆BOP, temos:
B
OB2 = OP2 + BP2 ⇒ OB2 = 42 + 32 ⇒
OB = 5
Então:
1 1 3,14 ⋅ 25 ⋅ 3, 36
V = BH = ⋅ π ⋅ 52 ⋅ 3, 36 =
3 3 3
V = 87, 92
Resposta: 88

65
Matemática Geometria de posição e métrica

CAPÍTULO 07 ESFERAS

1. Generalidades Sendo d a distância de um plano ao centro da


esfera de raio R (d < R), a secção será um cír-
A. Defi nição culo de raio r e a relação entre d, R e r é dada
Dados um ponto O e uma distância R, chama- pelo teorema de Pitágoras.
mos esfera ao conjunto de todos os pontos do
espaço cujas distâncias em relação ao ponto
O são menores ou iguais a R. O ponto O é o
centro da esfera e R seu raio. O'
r
d
R O R A

R 2 = d2 + r 2 ∆OO’A

A esfera é também o sólido de revolução gera-


do pela rotação de um semicírculo em torno
D. Elementos
de um eixo que contém o diâmetro. Consideremos uma superfície esférica de cen-
Eixo tro O, raio R e eixo e.
• Polos
R Polos são os pontos de intersecção do
O O eixo com a superfície da esfera.
R • Equador
Equador é a circunferência que obte-
mos ao seccionarmos a superfície por
um plano perpendicular ao eixo pas-
B. Superfí cie esférica sando pelo centro O.
Chamamos de superfície esférica de centro O • Paralelo
e raio R ao conjunto de todos os pontos P do
espaço, tais que a distância OP é igual a R. Paralelo é uma circunferência que ob-
A superfície esférica também é a superfície ge- temos ao seccionarmos a superfície por
rada pela rotação de uma semicircunferência um plano perpendicular ao eixo.
em torno de seu diâmetro. • Meridiano
PV-13-31

Eixo Eixo Meridiano é uma circunferência que


obtemos ao seccionarmos a superfície
R R por um plano que contém o eixo e.
O Polo e
O
R
P1

Paralelo

C. Secção da esfera Equador


Toda secção de uma esfera por um plano é um Meridiano
círculo. Quando o plano passa pelo centro da
esfera, a secção é um círculo com o mesmo
P2
raio da esfera, que chamamos círculo máximo. Polo

66
Geometria de posição e métrica Matemática

2. Volume
Consideremos dois sólidos S1 e S2 apoiados num mesmo plano a, definidos do seguinte modo:
– sólido S1: esfera de centro O1 e raio R;
– sólido S2: anticlepsidra de um cilindro equilátero de centro O2 e raio R. A anticlepsidra é
o sólido que obtemos do cilindro, eliminando dois cones com bases nas bases do cilindro
e vértices no centro do cilindro. O sólido formado pelos dois cones que eliminamos é a
clepsidra.

R
O1 2R O2

Vamos seccionar S1 e S2 por um plano paralelo a a a uma distância de O1 e O2 , e obteremos as


áreas A1 e A2 dessas secções.

O1 O2 2R
R
d d y
A1 x A2
45°

A1
x y
A2
R

x2 + d 2 = R 2 ⇒ x 2 = R 2 – d 2 y=d
PV-13-31

A1 = p · x2 = p · (R2 – d2) A2 = p · R2 – y2 = p · (R2 – d2)


Notamos que as áreas A1 e A2 são sempre iguais qualquer que seja a distância d < R. Assim, pelo
princípio de Cavalieri, concluímos que seus volumes são iguais.
Então, o volume da esfera é igual à diferença entre os volumes do cilindro e dos dois cones.
π ⋅ R2 ⋅ R 4
V = π ⋅ R2 ⋅ 2 ⋅ R − 2 ⋅ = π ⋅ R3
3 3
Assim:

4
V= π ⋅ R3
3

67
Matemática Geometria de posição e métrica

3. Área da superfície esférica V 4


A = lim = π ⋅ [3r2 + 3r ⋅ 0 + 02 ]
Consideremos um sólido formado por um d→ 0 d 3
“empilhamento” de superfícies congruentes,
conforme a figura. \ A = 4p · r2

A
d 4. Partes da esfera
A. Fuso esférico
Fuso esférico é a parte da superfície esférica
A = área da superfície limitada por dois planos que contêm um diâ-
d = altura metro.
O volume do sólido considerado é dado por: O ângulo a, medido na secção equatorial, é o
V=A·d que caracteriza o fuso.

V
Então, A =
d R

Consideremos agora duas superfícies esféricas


de centro O e raios R e r, com R – r = d > 0, e Arco equatorial
vamos calcular o volume desse sólido.

α
r r+d A área do fuso é proporcional à medida do ân-
gulo a, então:
• em graus:
360° l 4p · R2
a l Afuso

α
Afuso = · 4p · R2
4 4 360°
V = π ⋅ (r + d)3 − π ⋅ r3
3 3
4 • em radianos:
V = π ⋅  r3 + 3r2 d + 3rd2 + d3 − r3 
3 2p l 4p · R2
PV-13-31

4 a l Afuso
V = π ⋅ [3r2d + 3rd2 + d3 ]
3 Afuso = 2 · R2 · a
Dividindo a igualdade por d, temos:
V 4 Exemplo
= π ⋅ [3r2 + 3rd + d2 ] A área de um fuso de raio 4 cm e diedro 60° é:
d 3
360° l 4p · 42
Considerando d próximo de zero, o sólido
pode ser considerado como se fosse formado 60° l Afuso
por um “empilhamento” de superfícies con-
gruentes. 4 π ⋅ 42 32π
A fuso = = cm2
V 6 3
Assim, quando d tende a zero, é a área da
superfície da esfera: d

68
Geometria de posição e métrica Matemática

B. Cunha esférica Observação


Cunha esférica é a parte da esfera limitada por A área da superfície de uma cunha é dada pela
dois planos que contêm um diâmetro. soma das áreas do fuso e dos dois semicírculos
de raio R (raio da esfera).
O ângulo a, medido na secção equatorial, é o
que caracteriza o fuso. Exemplo
Dada uma cunha esférica de diedro 45° e raio
4 cm, calcular:
R a. o volume da cunha;
b. a área da superfície da cunha.
Arco equatorial Resolução
45° 4 32π 3
α a. Vcunha = ⋅ π ⋅ 43 = cm
360° 3 3
O volume de uma cunha é proporcional à me-
dida a, então: 45°
b. A fuso = ⋅ 4 π ⋅ 42 = 32π cm2
• em graus: 360°
4 π ⋅ R2
360° l π ⋅R2 A cunha = A fuso + 2 ⋅ = 32π + π ⋅ 42
3 2
a l Vcunha A cunha = 48π cm2

α 4
Vcunha = ⋅ π ⋅ R3
360° 3

• em radianos:
4
2p l π ⋅R3
3
a l Vcunha

2 ⋅ R3 ⋅ α
Vcunha =
3

EXERCÍCIOS RESOLVIDOS
PV-13-31

01. Cuidadosamente, ele retira a bola e sai sem


Fred, após um chute descuidado, vê sua bola ninguém perceber.
cair num piso em que o cimento está fresco. No dia seguinte, o pedreiro que trabalha na
obra vê o sulco que a bola deixou no piso e
NTO descobre que ele tem 1 cm de profundidade e
CIME O
FRE C
S 10 cm de diâmetro. Com isso, pode-se concluir
corretamente que o raio da bola é:
a. 10 cm d. 15 cm
b. 12 cm e. 16 cm
c. 13 cm

69
Matemática Geometria de posição e métrica

Resolução 03.
Pedro Ivo, coloca uma esfera no interior de um
recipiente cilíndrico cuja base tem raio R. Colo-
ca-se água no recipiente até que a esfera fique
exatamente coberta (tangencie a superfície da
O água). Retira-se, então, a esfera e verifica-se
1
que o nível da água reduz-se de .
R 3
R–1

A 5
1 B

No triângulo OAB, temos: r


R2 = (R – 1)2 + 52 R
R2 = R2 – 2R + 1 + 25
r
2R = 26 \ R = 13 A razão é igual a:
R
Isto é, a bola tem 13 cm de raio.
2 1
Resposta a. d.
2 3
C
1 3
02. Unifor-CE b. e.
2 2
Duas esferas, de mesmo raio, são tangentes
externamente. Um cilindro circular reto, de 3
c.
volume 108p dm3, é circunscrito às duas esfe- 3
ras, de modo que seu eixo contém um diâme-
tro de cada esfera. Para cada uma das esferas, Resolução
o volume, em dm3, e a área da superfície, em
dm2, são, respectivamente: 1
a. 30p e 32p d. 36p e 32p · 2r
3
b. 32p e 30p e. 36p e 36p
R
c. 32p e 36p 2r
Resolução
VC = 108p
r
pr2 · 4r = 108p ⇒ r3 = 27 Com a esfera submersa, o nível que a água atin-
PV-13-31

r r ge é igual a 2r. Logo, ao se retirar a esfera, o ní-


\ r = 3 dm
1
4 4 r vel da água baixa o equivalente a ⋅ 2r. Então:
VE = π ⋅ r3 = π ⋅ 33 4r 3
3 3 4 1
r ⋅ πr = πR ⋅ ⋅ 2 ⋅ r
3 2
\ VE = 36p dm3 r 3 3
r 2r 3 ⋅ R2 ⋅ r
SE = 4pr = 4 · p · 3
2 2

\ SE = 36p dm2 2r2 = R2

Resposta r2 1 r 2
= ∴ =
R 2 2 R 2
E
Resposta
A

70
Geometria de posição e métrica Matemática

04. a. a área da casca de cada fatia da melan-


Um cone reto, cujo raio da base tem 4 cm e cia (fuso esférico);
cuja altura tem 8 cm, está inscrito numa esfe- b. quantos cm2 de plástico foram neces-
ra. Calcule o volume dessa esfera. sários para embalar cada fatia (sem ne-
nhuma perda e sem sobrepor camadas
Resolução
de plástico), ou seja, a área da superfí-
cie total de cada fatia.

R Resolução
12 fatias − 4 πR2
a. 
1 fatia − A fuso
R
(8 – R) πR2
A fuso =
R 3
4
b. A total = A fuso + A cir. máx
πR2
Pelo teorema de Pitágoras, temos: A total = + πR2
3
R2 = (8 – R)2 + 42 4 πR2 2
R2 = 64 – 16R + R2 + 16 A total = cm
3
16R = 80 ⇒ R = 5
06. FGV-SP
O volume da esfera é:
Deseja-se construir um galpão em forma
4 de um hemisfério, para uma exposição. Se,
V = π ⋅ R3
3 para o revestimento total do piso, utilizou-se
4 500π 78,5 m2 de lona, quantos metros quadrados
V = ⋅ π ⋅ 53 = de lona se utilizaria na cobertura completa do
3 3 galpão?
500π 3
V= cm (Considerar p = 3,14.)
3
a. 31,4
05. Unesp b. 80
Uma quitanda vende fatias de melancia emba- c. 157
ladas em plástico transparente. Uma melancia d. 208,2
com forma esférica de raio de medida R cm foi
cortada em 12 fatias iguais, tendo cada fatia a e. 261,66
forma de uma cunha esférica, como represen- Resolução
PV-13-31

tado na figura.
SSE 2
Slona = = 4π R →
2 2
→ Slona = 2πR 2
R R Como Spiso = π R 2,
temos que:
Spiso = 78,5 = π R2 ∴
Slona = 2.Spiso
Slona = 157m2
Sabendo-se que a área de uma superfície es- Resposta
férica de raio R cm é 4pR2 cm2, determine, em
função de p e de R: C

71
Matemática Geometria de posição e métrica

CAPÍTULO 08 SÓLIDOS SEMELHANTES

1. Definição
Consideremos uma pirâmide (*) de vértice V e altura H.
Vamos seccionar essa pirâmide por um plano paralelo à sua base a uma distância h do vértice V.

V
Nova
pirâmide h

H–h
Tronco de
pirâmide

Obtemos assim dois sólidos:


• o sólido que contém o vértice V é uma nova pirâmide de altura h.
• o sólido que contém a base da pirâmide considerada é um tronco de pirâmide de altura
H – h.
(*) A mesma análise pode ser feita com um cone.

Propriedade fundamental
h
A nova pirâmide e a pirâmide primitiva são semelhantes e a razão de semelhança é k =
H
2. Consequências
h2
01. A razão entre as áreas das bases é igual ao quadrado da razão de semelhança .
H2
h2
02. A razão entre as áreas laterais é igual ao quadrado da razão de semelhança .
H2
h2
03. A razão entre as áreas totais é igual ao quadrado da razão de semelhança .
PV-13-31

H2
h3
04. A razão entre os volumes é igual ao cubo da razão de semelhança .
H3

Exemplos de aplicação
01. Considere uma pirâmide qualquer de altura h e de base B. Traçando-se um plano paralelo à
3
base B, cuja distância ao vértice da pirâmide é h cm, obtém-se uma secção plana de área
5
4 cm2. Calcule a área B.

72
Geometria de posição e métrica Matemática

Resolução h 3
k= = ⇒h= 9
h+6 5
3 Vsólido = Vcone maior − Vcone menor
h
S = 4cm2 5 π ⋅ 52 ⋅ 15 π ⋅ 32 ⋅ 9
V= −
h 3 3
V = 98π
B=? 03. O apótema (*) de um tronco de pirâmide
regular mede 10 dm, as bases são quadradas,
de lados, respectivamente, 8 dm e 20 dm. Cal-
cule o volume.
3
h (*) O apótema do tronco de pirâmide regular é
5 3 3
K= ⇒K = ⇒ k2 = a altura de cada uma das faces laterais.
h 5 5
Resolução
S 4 3
= k2 ∴ =
B B 5
20 x
B = cm3
3
M2 4
O' O' M
02. Calcule o volume do sólido gerado pela ro-
10 h h 10
tação de 360° do trapézio da figura em torno
do eixo.
O M O 4 6 M'

h2 + 62 = 102 \ h = 8 cm
3 x 8 16
k= = ⇒ x = cm
x + 8 20 3
VTronco = Vpir. maior − Vpir. menor
6
 16 
202 ⋅  8 +  82 ⋅ 16
 3 3
V= −
3 3
5 16.000 1.024
V= − = 1.664 dm3
9 9

Resolução 04. Determine a área lateral e a área total de


PV-13-31

um tronco de cone, sabendo que os raios de


suas bases medem 11 cm e 5 cm e que a altura
do tronco mede 8 cm.
h Resolução
3
h+6 x g x
5
6

5 y 8

11

73
Matemática Geometria de posição e métrica

x 5 20  25  25
k= = ⇒ x = cm AL = π ⋅ 11 ⋅  + 10 − π ⋅ 5 ⋅
x + 8 11 3  3  3
 20 
2
25 605π 125π
AL = − = 160π cm2
  + 5 = g ∴ g = cm
2 2
3 3 3 3
g 5 A T = 160π + π ⋅ 52 + π ⋅ 112 = 306π cm2
=
g + y 11
25
3 = 5 ⇒ y = 10
25
+ y 11
3

EXERCÍCIOS RESOLVIDOS
01.
"Seu" Nélio adora tomar um chopinho com os
h
amigos nos fins de semana. Ele costuma pedir /2
Espuma
um chope na pressão. O garçon lhe serve uma
tulipa, cujo interior tem a forma praticamente
cônica, com chope até à metade da altura e o
resto sendo ocupado por espuma.
Chopp h
/2

Dessa forma ele reparou que a parte de bai-


xo, ocupada pelo chope, também é um cone. E
mais: é um cone semelhante ao cone inteiro. A
1
razão da semelhança é k = , pois as medidas
2
do cone da parte de baixo equivalem à metade
das medidas do cone inteiro.
13 1
Então a razão entre seus volumes é k3 = = .
PV-13-31

Qual a razão entre a quantidade de chope e a 2 8


quantidade de espuma que vem na tulipa de Ou seja, o volume de chope na tulipa, correspon-
"seu" Nélio? 1
de a apenas do que ela pode conter!
Resolução 8
1
Para resolver esse problema, "seu" Nélio con- Foi aí que "seu" Nelio levou um susto: se
siderou a parte interna da tulipa como sendo 7  1 8
um cone perfeito. é de chope, então  1 −  são de espuma!
8 8
1 7
Assim, temos de chope e de espuma,
8 8
Logo a razão é de 1 : 7 ( 1 para 7).

74
Geometria de posição e métrica Matemática

02. Fuvest-SP 1
a. do raio da esfera original.
Uma pirâmide tem como base um quadrado 2
de lado 1, e cada uma de suas faces laterais é
1
um triângulo equilátero. Então, a área do qua- b. do raio da esfera original.
drado, que tem como vértices os baricentros 3
de cada uma das faces laterais, é igual a: 1
5 1 1 c. do raio da esfera original.
a. c. e. 4
9 3 9
1
4 2 d. do raio da esfera original.
b. d. 6
9 9
1
Resolução e. do raio da esfera original.
V
8
Resolução
1
Vantes = ⋅ Vdepois
R 8
V
S Q K3 = antes = 8 ⇔ K = 2
C
Vdepois
D
P
Rantes
k=
M N Rdepois
1
A B Rdepois = R
2 antes
Sejam P, Q, R e S os baricentros dos triângulos Resposta
VAB, VBC, VCD e VDA, respectivamente.
A
Assim, os triângulos VSQ e VMN são seme-
lhantes e a razão de semelhança é 2 : 3, pois 04. UFMG
2 Corta-se uma pirâmide regular de base quadran-
VQ = ⋅ VN .
3 gular e altura 4 cm por um plano paralelo ao pla-
2 2 2
Logo, SQ = . MN = · 1 = e, portanto, a no da base, de maneira que os volumes dos dois
3 3 3 sólidos obtidos sejam iguais. A altura do tronco
área do quadrado PQRS é: da pirâmide obtida é, em centímetros:
2
 2
(SQ )2   2
3
SPQRS = = =
PV-13-31

2 2 9
4
Resposta
D α // base
HT
03. UFES
Um ourives deixou como herança para seus fi-
lhos uma esfera maciça de ouro. Os herdeiros
resolveram fundir o ouro e, com ele, fazer oito a. 1
3
esferas iguais. Cada uma das esferas terá um b. 4 − 2 4
raio igual a: c. 2
d. 4 − 2
e. 4 − 4 2

75
Matemática Geometria de posição e métrica

Resolução 2 
Vcone '  d  3 h 3
(I) Sendo o plano paralelo à base, as pirâmides V'  3   2 8
são semelhantes. =  ⇒ =  =  = ⇒
Vcone  h  V h  3 27
 
Como o volume do tronco e da pirâmide são
iguais, sendo V o volume da pirâmide original 8V
e V’ o volume da nova pirâmide, temos: 8V = 27 V ' =
27
V 3
(II) V = 2V’ ⇒ = 2 ⇒ k3 = 2 ⇒ k = 2 8
V' Se o cone menor é do cone maior, então, o
h' 4 27
(III) = k ⇒ = 3 2 ⇒ h = 4 3 4 volume do tronco do cone será:
h h
VTronco = Vcone maior – Vcone menor
(IV) Como h + HT = H ⇒ 2 3 4 + HT = 4 ⇒
8 19
⇒ HT = 4 – 2 3 4 VTronco = V − V= V
27 27
Resposta
Resposta
B
D
05. UnB-DF
06. Fuvest-SP
Um cone circular reto é seccionado por um
2 Um copo tem a forma de um cone com altura
plano paralelo à sua base a de seu vértice. 8 cm e raio de base 3 cm. Queremos enchê-lo
3 com quantidades iguais de suco e água. Para
Se chamarmos V o volume do cone, então o que isso seja possível, a altura x atingida pelo
volume do tronco de cone resultante vale: primeiro líquido colocado deve ser:
8 4
a. V c. V
27 9
2 19
b. V d. V
3 27
8=H

2 x=h
h
3

8
a. cm
3
PV-13-31

b. 6 cm

Resolução c. 4 cm

d. 4 3 cm
2
d= h 3
3 e. 4 4 cm
r' h

76
Geometria de posição e métrica Matemática

Resolução Resolução
(I) Admitindo os cones semelhantes, temos: O volume do líquido da taça semiesférica é:
V 3 1 4
(II) V = 2V’ ⇒ = 2 = k3 ⇒ k = 2 V = · · π · 33 = 18 π cm3
V' 2 3
H 8 O volume do líquido da taça cônica é:
(III) = k ⇒ = 3 2 ⇒ x = 4 3 4 cm
h x
1
Resposta V = · π · 32 · h = 3 π h cm3
3
E
Como as taças têm o mesmo volume, tem-se:
07. ENEM
Em um casamento, os donos da festa serviam 18p = 3p · h
champanhe aos seus convidados em taças com ∴ h = 6 cm
formato de um hemisfério (figura 1), porém
Resposta
um acidente na cozinha culminou na quebra
de grande parte desses recipientes. Para subs- B
tituir as taças quebradas, utilizou-se um outro
tipo com formato de cone (figura 2). No en-
tanto, os noivos solicitaram que o volume de
champanhe nos dois tipos de taças fosse igual.
R = 3 cm
R = 3 cm

Figura 1 Figura 2

Considere:
4 1
Vesfera = p R3 e Vcone = p R2h
3 3
Sabendo que a taça com o formato de hemis-
fério é servida completamente cheia, a altura
do volume de champanhe que deve ser colo-
cado na outra taça, em centímetros, é de:
PV-13-31

a. 1,33
b. 6,00
c. 12,00
d. 56,52
e. 113,04

77
Matemática Geometria de posição e métrica

ANOTAÇÕES

PV-13-31

78
Exercícios Propostos
Geometria de posição e métrica Matemática

Capítulo 01
01. UFRGS-RS 05.
Quais pontos pertencem à reta r? Qual reta está contida no plano α ?
A t
A t
E F
E F B r
B r

α
D
α
D
s
s 06.
Quantos são os planos determinados por qua-
02. tro pontos distintos dois a dois?
Quais pontos pertencem ao plano α? 07. Unicamp-SP
A t É comum encontrarmos mesas com 4 pernas
F
que, mesmo apoiadas em um piso plano, ba-
E lançam e nos obrigam a colocar um calço em
B r
uma das pernas se a quisermos firme.
Explique, usando argumentos de geometria,
α porque isso não acontece com uma mesa de
D 3 pernas.
s 08. Fuvest-SP
Os segmentos VA, VB e VC são arestas de um
03. cubo. Um plano a, paralelo ao plano ABC, di-
Qual ponto pertence a r ∩ s? vide esse cubo em duas partes iguais. A inter-
secção do plano a com o cubo é um:
A t a. triângulo.
E F b. quadrado.
B r
c. retângulo.
d. pentágono.
D
α e. hexágono.
09. ITA-SP
PV-13-14

s Qual das afirmações abaixo é verdadeira?


a. Três pontos, distintos dois a dois, de-
04. terminam um plano.
Qual ponto pertence a r ∩ t? b. Um ponto e uma reta determinam um
plano.
A t c. Se dois planos distintos têm um ponto
E F em comum, tal ponto é único.
B r d. Se uma reta é paralela a um plano e não
está contida neste plano, então ela é pa-
ralela a qualquer reta desse plano.
α e. Se a é o plano determinado por duas
D
retas concorrentes r e s, então toda
s reta m desse plano, que é paralela à
reta r, não será paralela à reta s.

81
Matemática Geometria de posição e métrica

10. 14.
Assinale a alternativa falsa. Considerando quatro pontos distintos, A, B,
a. Dado um ponto, existe uma reta que C e D, o número de planos determinados por
passa por ele. eles é:
b. Dois pontos são sempre colineares. a. 1
c. Três pontos são sempre coplanares. b. 0 ou 1
d. Dois pontos determinam uma única reta.
c. 0
e. Três pontos não colineares são sempre
distintos. d. 0,1 ou 4
11. e. 5
Sobre a determinação de planos, a afirmação 15. FCC-SP
falsa é: Quatro pontos distintos e não coplanares deter-
a. Três pontos não colineares determinam minam exatamente:
um plano.
a. 1 plano.
b. Uma reta e um ponto fora dela deter-
minam um plano. b. 2 planos.
c. Duas retas paralelas distintas determi- c. 3 planos.
nam um plano.
d. 4 planos.
d. Duas retas concorrentes determinam
um plano. e. 5 planos.
e. Três pontos distintos determinam um 16.
plano.
Quantos dos planos determinados pelos oito
12.
vértices contêm a reta ? Quais são esses
Classifique em verdadeiro (V) ou falso (F):
planos?
a. ( ) Por uma reta passam infinitos planos.
b. ( ) Dados três pontos distintos, o plano
que os contém é único.
c. ( ) Os vértices de qualquer triângulo
são coplanares.
d. ( ) Se três pontos são coplanares, en-
tão eles são colineares.
e. ( ) Três pontos distintos sempre deter-
minam três retas distintas.
PV-13-14

13.
17. FEI-SP
Classifique em verdadeiro (V) ou falso (F):
a. ( ) Dados um ponto e uma reta, o plano Assinale a alternativa falsa.
que os contém é único. a. Dados dois pontos distintos, A e B, exis-
b. ( ) Dadas duas retas paralelas, o plano te um plano que os contém.
que as contém é único. b. Por um ponto fora de uma reta existe
c. ( ) Duas retas paralelas distintas e uma uma única paralela à reta dada.
terceira reta concorrente com as duas c. Existe um e um só plano que contém
estão contidas no mesmo plano. um triângulo dado.
d. ( ) Três retas distintas, concorrentes d. Duas retas não coplanares são reversas.
duas a duas, são sempre coplanares. e. Três pontos distintos determinam um e
e. ( ) Três retas distintas, paralelas duas a um só plano.
duas, são sempre coplanares.

82
Geometria de posição e métrica Matemática

18. FEI-SP 22. Fuvest-SP


Na determinação de um plano, são suficientes Sejam r e s duas retas distintas. Podemos afir-
os seguintes elementos: mar que sempre:
a. duas retas distintas. a. existe uma reta perpendicular a r e a s.
b. uma reta e um ponto. b. r e s determinam um único plano.
c. duas retas reversas. c. existe um plano que contém s e não in-
d. duas retas paralelas. tercepta r.
e. duas retas concorrentes. d. existe uma reta que é paralela a r e a s.
19. e. existe um plano que contém r e um único
ponto de s.
Considere o cubo representado abaixo.
23. Fuvest-SP
H G São dados cinco pontos não coplanares, A, B, C,
D e E. Sabe-se que ABCD é um retângulo, AE ⊥ AB
E
F e AE ⊥ AD. Pode-se concluir que são perpendi-
culares as retas:
C a. EA e EB.
D
b. EC e CA.
A c. EB e BA.
B
d. EA e AC.
Os pontos A, B, C, D, E, F, G e H determinam e. AC e BE.
quantos planos que contêm a reta ? Quais 24. Fuvest-SP
são esses planos?
Dada uma circunferência de diâmetro AB ,
20.
levanta-se por A um segmento AP perpendi-
Assinale verdadeiro (V) ou falso (F). cular ao plano da circunferência, e une-se P a um
a. ( ) Três retas, duas a duas concorren- ponto qualquer da circunferência,
 C distinto de B.
tes, são coplanares. a. Prove que as retas BC e PC são perpen-
b. ( ) Três retas, duas a duas paralelas dis- diculares.
tintas, determinam três planos. b. Sabendo que AB = AP = 8 e que C é o
c. ( ) Três retas, duas a duas concorrentes ponto médio do arco AB  , determine a
em pontos distintos, são coplanares. medida do ângulo CPB. 
d. ( ) Duas retas distintas determinam um
25. FEI-SP
plano.
e. ( ) Uma reta e dois pontos distintos Considere duas retas, r e s, paralelas e distin-
PV-13-14

fora dela determinam dois planos. tas. Pode-se afirmar corretamente que:
a. não existe um único plano que as con-
21. PUC-SP modificado
tenham.
Se uma reta a é perpendicular a uma reta b b. todo plano que contém r contém s.
e a reta b é paralela a uma reta c, podemos
concluir que: c. todo plano que contém r e um ponto
de s contém s.
a. a ∩ c = ∅
d. todo plano que contém dois pontos de
b. a ⊥ c s contém r.
c. a ≡ c e. tão existe plano que contenha apenas
d. a // c uma das retas r ou s.
e. a e c formam um ângulo de 90°.

83
Matemática Geometria de posição e métrica

26. Unifesp modificado 30. Fuvest-SP


Considere o sólido geométrico exibido na figu- a. Defina retas paralelas.
ra, constituído de um paralelepípedo reto re- b. Sejam r e s retas paralelas e P um ponto
tângulo encimado por uma pirâmide reta. Seja não pertencente ao plano rs. Prove que
r a reta suporte de uma das arestas do sólido, a reta t, intersecção dos planos (Pr) e
conforme mostrado. (Ps), é paralela a r e a s.
31.
Classifique em verdadeiro (V) ou falso (F).
r a. ( ) Se duas retas não têm ponto co-
mum, então elas são paralelas.
b. ( ) Se duas retas são paralelas, então
Quantos pares de retas reversas é possível elas não têm ponto comum.
formar com as retas suportes das arestas do c. ( ) A condição r ∩ s = ∅ é suficiente
sólido, sendo r uma das retas do par? para que as retas r e s sejam reversas.
a. 12 d. ( ) A condição r ∩ s = ∅ é necessária
b. 10 para que as retas r e s sejam reversas.
c. 8 e. ( ) Se duas retas não são coplanares, en-
tão elas são reversas.
d. 7
e. 6 32. UFAL modificado
27. Na figura abaixo, tem-se um cubo de vértices
A, B, C, D, E, F, G, H. Assinale V ou F.
As semirretas não são copla- A B
nares, então os pontos A, B e C determinam
um plano. Verdadeiro ou falso? Justifique. C
D
28. PUC-SP
Qual das afirmações abaixo é verdadeira? F
E
a. Se duas retas distintas não são parale-
las, elas são concorrentes. H G

b. Duas retas não coplanares são reversas.  


a. ( ) As retas BG e AD são reversas.
c. Se a intersecção de duas retas é o con-  
b. ( ) A reta FG é paralela à reta AB .
junto vazio, elas são paralelas.  
c. ( ) As retas AB e CG são ortogonais.
d. Se três retas são paralelas, existe um  
plano que as contém. d. ( ) A reta EF é paralela à reta DC .
 
e. Se três retas distintas são duas a duas e. ( ) As retas AE e BD são ortogonais.
PV-13-14

concorrentes, elas determinam um e 33. FAAP-SP


um só plano.
O galpão da figura a seguir está no prumo e a
29.
cumeeira está “bem no meio” da parede.
Dada a figura a seguir, determine os pares de
retas reversas. u
Cumeeira
V 4m
s
A
3m
t
A v
B r

D C 4m

84
Geometria de posição e métrica Matemática

Das retas assinaladas, podemos afirmar que: 37. FAAP-SP


a. t e u são reversas. Duas retas são reversas quando:

b. s e u são reversas. a. não existe plano que contém ambas.


c. t e u são concorrentes. b. existe um único plano que as contém.
d. s e r são concorrentes.
c. não se interceptam.
e. t e u são perpendiculares.
34. FAAP-SP d. não são paralelas.

A figura abaixo mostra uma porta entreaberta e e. são paralelas, mas estão contidas em
o canto de uma sala: planos distintos.
x 38. AFA-RJ
r Assinale a única alternativa falsa.
a. Se um plano a é perpendicular a um
t
plano b, então existem infinitas retas
z contidas em a e perpendiculares a b.
s
y
b. Se a e b são planos perpendiculares
entre si e g é um plano perpendicular
As retas r e s; s e t; x e r têm, respectivamente, as
à reta comum a a e b, então pode-se
posições relativas:
afirmar que as retas r, r = a ∩ g e s,
a. paralelas, paralelas e perpendiculares. s = b ∩ g são perpendiculares entre si.
b. paralelas, perpendiculares e reversas.
c. Se duas retas r e s são reversas, então
c. paralelas, perpendiculares e perpendi- não existem dois planos a e b, perpen-
culares. diculares entre si, tais que r ∈a e s ∈b.
d. reversas, paralelas e perpendiculares.
d. Duas retas do espaço, paralelas a uma
e. perpendiculares, reversas e paralelas. terceira, são paralelas entre si.
35. UFSCar-SP 39. Unifesp
Dadas duas retas reversas r e s, então: Dois segmentos dizem-se reversos quando
a. existe plano paralelo a ambas. não são coplanares. Nesse caso, o número
b. existe um único plano paralelo a am- de pares de arestas reversas num tetraedro,
bas. como o da figura, é:
c. todo plano, perpendicular a uma, en- A
contra a outra em um ponto.
d. existe sempre plano perpendicular a
PV-13-14

uma, que contém a outra.


e. r e s são perpendiculares. B D
36. Mackenzie-SP modificado
Se r e s são duas retas paralelas a um plano a,
C
então:
a. r // s. a. 6
b. r ⊥ s. b. 3
c. r e s se interceptam. c. 2
d. r e s são reversas. d. 1
e. r e s possuem mais de uma posição re- e. 0
lativa.

85
Matemática Geometria de posição e métrica

40. Vunesp 43. Vunesp


Entre todas as retas suportes das arestas de Sejam α e β planos perpendiculares, α ∩ β = r.
um certo cubo, considere duas, r e s, reversas. Em α considera-se uma reta s perpendicular a
Seja t a perpendicular comum a r e a s. Então: r, s ∩ r = {A}, e em β considera-se t oblíqua a
a. t é a reta suporte de uma das diagonais r, t ∩ r = {A}.
de uma das faces do cubo.
Dentre as afirmações:
b. t é a reta suporte de uma das diagonais
do cubo. I. s é perpendicular a β.
c. t é a reta suporte de uma das arestas
II. t é perpendicular a s.
do cubo.
d. t é a reta que passa pelos pontos mé- III. O plano determinado por s e t é per-
dios das arestas contidas em r e s. pendicular a β.
e. t é a reta perpendicular a duas faces do
cubo, por seus pontos médios. IV. Todo plano perpendicular a s e que não
contém A é paralelo a β.
41. Unifesp
Pode-se garantir que:
Considere a figura seguinte. Temos r ⊥ b e s ⊂ b
e também AP = 4 cm e PQ = 2 cm. Tomemos a. somente I é falsa.
em s um ponto R tal que QR = 1 cm, calcule AR: b. somente II é falsa.
r c. somente III é falsa.
A
d. somente IV é falsa.
e. nenhuma é falsa.
s β
44. UEL-PR
P R Sobre a posição relativa de planos no espaço,
é correto afirmar:
Q
a. Se os planos a e b são perpendiculares
a um plano λ, então a é paralelo a b.
b. Se dois planos, a e b, são paralelos en-
tre si, então a interseção de qualquer
42. PUC
outro plano λ com estes é um par de
Um triângulo isósceles ABC, com AB = BC = 30 retas paralelas.
PV-13-14

e AC = 24, tem o lado AC contido em um pla- c. Por uma reta r perpendicular a um pla-
no a e o vértice B a uma distância 18 de a. A no a passam apenas dois planos, b e λ,
projeção ortogonal do triângulo ABC sobre o perpendiculares ao plano a.
plano a é um triângulo:
d. Por um ponto P não pertencente a um
a. retângulo. plano a passam infinitos planos parale-
b. obtusângulo. los ao plano a.
c. equilátero. e. Dois planos, a e b, paralelos a uma reta
r são paralelos entre si
d. isósceles, mas não equilátero.
e. semelhante ao triângulo ABC.

86
Geometria de posição e métrica Matemática

45. Fuvest-SP 48. Espcex-SP


Sejam p’ e p’’ as faces de um ângulo diedro de Se a reta r é paralela ao plano a, então:
45° e P um ponto interior a esse diedro. Sejam
a. todas as retas de a são paralelas a r.
P’ e P’’ as projeções ortogonais de P sobre p’
e p’’, respectivamente. Então, a medida, em b. existem em a retas paralelas a r e retas
graus, do ângulo P’PP’’ é: reversas a r.
45° c. existem em a retas paralelas a r e retas
perpendiculares a r.
P”
d. todo plano que contém r intercepta a,
P’
P
segundo uma reta paralela a r.
49. Fuvest-SP
Dados um plano a e uma reta r, podemos afirmar
π’’
que:
π’ a. existe um plano b que contém r e é per-
pendicular a a.
a. 30
b. existe um único plano b que contém r e
b. 45
é perpendicular a a.
c. 60
c. existe um plano b que contém r e é pa-
d. 90
ralelo a a.
e. 135
d. existe um único plano b que contém r e
46. UFMT é paralelo a a.
Sobre geometria espacial de posição, assinale
e. qualquer plano b que contém r inter-
a afirmativa correta.
cepta o plano a.
a. Se dois planos são paralelos a uma reta,
então eles são paralelos entre si. 50. PUC-SP
b. Quatro pontos no espaço determinam Dois planos, β e γ cortam-se na reta r e são
quatro planos. perpendiculares a um plano a. Então:
c. Três planos distintos podem se cortar, a. β e γ são perpendiculares.
dois a dois, segundo três retas duas a
b. r é perpendicular a α.
duas paralelas.
d. A interseção de dois planos secantes c. r é paralela a α.
pode ser um único ponto. d. todo plano perpendicular a α encontra r.
e. Duas retas reversas determinam um plano.
PV-13-14

e. existe uma reta paralela a α e a r.


47.
51. FEI-SP
Um triângulo isósceles EPC, com EP = EC = 5 cm
e PC = 4 2 cm, tem o lado EC contido em um Sejam quatro pontos, A, B, C e D, não copla-
plano a e o vértice P a uma distância de 4 cm nares. O número de planos determinados por
de a. A projeção ortogonal do triângulo EPC dois desses pontos e pelo ponto médio do seg-
sobre o plano a é um triângulo: mento que liga os outros dois é:
a. 4
a. obtusângulo.
b. 6
b. equilátero.
c. 8
c. isósceles, mas não equilátero.
d. 10
d. retângulo. e. infinito
e. acutângulo.

87
Matemática Geometria de posição e métrica

52. ITA-SP Observe a figura abaixo.


Quais as sentenças falsas, nos itens a seguir? t π α e π são planos secantes
I. Se dois planos são secantes, todas as A
A ∈π
retas de um deles sempre interceptam B 60°
o outro plano. B ∈t
II. Sejam dois planos. Se em um deles AB ⊥ t
existem duas retas distintas, paralelas α
AB = 10 cm
ao outro plano, os planos são sempre
paralelos.
Determine a medida da projeção ortogonal do
III. Em dois planos paralelos, todas as retas segmento AB sobre o plano α.
de um são paralelas ao outro plano.
55. Mackenzie-SP
IV. Se uma reta é paralela a um plano, nes-
te existe uma infinidade de retas para- Assinale a única proposição verdadeira.
lelas àquela reta. a. Uma reta é perpendicular a um plano,
V. Se uma reta é paralela a um plano, será quando ela é perpendicular a todas as
paralela a todas as retas do plano. retas do plano.
a. I, II, III b. Dois planos distintos perpendiculares a
um terceiro são paralelos entre si.
b. I, II, V
c. A projeção ortogonal de uma reta num
c. I, III, IV
plano é sempre uma reta.
d. II, III, IV
d. Um plano paralelo a duas retas de um
e. I, II, IV plano é paralelo ao plano.
53. Vunesp e. Duas retas perpendiculares, respecti-
Sabe-se que o arco mostrado na figura adiante vamente, a três planos paralelos, são
é o arco de uma circunferência de centro e raio paralelas.
desconhecidos. Sobre a circunferência marca-se 56. Fatec-SP
uma corda AB de 4 cm de comprimento. Sendo
Na figura exposta, tem-se: o plano α definido
D o ponto médio do arco AB e C o pé da per-
pelas retas c e d, perpendiculares entre si; a
pendicular baixada de D sobre AB , verifica-se
reta b, perpendicular a α em A, com A ∈ c,
que o segmento de reta CD mede 1,2 cm.
o ponto B, intersecção de c e d. Se X é um
A
ponto de b, X ∉ α , então a reta s, de -
finida por X e B:
b
C
d
PV-13-14

D A B C
B α

Considerando esse dados, calcule a medida do


raio da circunferência. a. é paralela à reta c.
54. Unimontes-MG b. é paralela à reta b.
“Chama-se projeção ortogonal de uma figura c. está contida no plano α.
sobre um plano o conjunto de todas as pro- d. é perpendicular à reta d.
jeções ortogonais dos pontos da figura sobre
esse plano.” e. é perpendicular à reta b.

88
Geometria de posição e métrica Matemática

57. Vunesp 59.


Na figura a seguir, o segmento AB é perpendi- Sabendo que A, B, C, D e E são 5 pontos, os
cular ao plano a, CD e BC estão contidos nesse quatro primeiros formam um quadrilátero re-
plano e CD é perpendicular a BC . Se AB = 2 gular com os vértices em ordem alfabética e o
cm, BC = 4 cm e CD = 3 cm, ache a distância último tem projeção ortogonal sobre o primei-
de A a D ro, pode-se concluir:
 

A a. EA e EB .

 
C b. EC e CA .
B

 
c. EB e BA .
D  
α d. EA e AC .
 
58. Fuvest-SP e. AC e BE .
Uma formiga resolveu andar de um vértice 60. Fuvest-SP
a outro do prisma reto de bases triangulares
ABC e DEG, seguindo um trajeto especial. Ela O segmento AB é um diâmetro de uma circun-
partiu do vértice G, percorreu toda a aresta ferência e C, um ponto dela, distinto de A e de B.
perpendicular à base ABC, para, em seguida, A reta VA, V ≠ A, é perpendicular ao plano da
caminhar toda a diagonal da face ADGC e, fi- circunferência. O número de faces do tetrae-
nalmente, completou seu passeio percorren- dro VABC que são triângulos retângulos é:
do a aresta reversa a CG. A formiga chegou ao a. 0
vértice: b. 1
G c. 2
d. 3
D E
e. 4

A B

a. A
b. B
PV-13-14

c. C
d. D
e. E

89
Matemática Geometria de posição e métrica

Capítulo 02
61. 64. PUC-SP
Descreva completamente o poliedro abaixo, O número de vértices de um poliedro convexo
considerando: que possui 12 faces triangulares é:
• o número de faces de cada tipo; a. 4
• o número total de lados das faces; b. 12
• o número de arestas do poliedro; c. 10
• o número de vértices das faces; d. 6
• o número de vértices do poliedro; e. 8
• a soma dos ângulos das faces. 65. PUCCamp-SP modificado
V O "cubo octaedro" é um poliedro que possui
6 faces quadrangulares e 8 triangulares. O
número de vértices desse poliedro é:
a. 12
A B b. 16
c. 10
D d. 14
C
e. 16
Verifique a relação de Euler. 66. ITA-SP
62. Cesgranrio-RJ Determine o número de arestas de um poliedro
Um poliedro convexo é formado por 4 faces convexo de 20 faces e 12 vértices.
triangulares, 2 faces quadrangulares e 1 face 67. UEPG-PR
hexagonal. O número de vértices desse poliedro
Um poliedro convexo possui 2 faces triangula-
é:
res e 4 pentagonais. Sobre ele, afirma-se:
a. 6
I. o número de arestas excede o número
b. 7 de vértices em cinco unidades.
c. 8 II. a soma dos ângulos das faces é igual a
d. 9 28 retos.
e. 10 III. o número de vértices é 9.
63. PUC-RS IV. o número de arestas é 12.
PV-13-14

Um poliedro convexo de 33 arestas possui fa- Estão corretas as afirmativas:


ces triangulares e hexagonais. Sendo 6.840° a a. I, II e III.
soma dos ângulos internos das faces, o núme- b. II e III.
ro de faces triangulares e hexagonais é, res-
pectivamente: c. II, III e IV.
a. 4 e 10 d. I e II.
b. 7 e 7 68. UFPE
c. 6 e 8 Um poliedro convexo possui 10 faces com 3
d. 5 e 9 lados, 10 faces com 4 lados e 1 face com 10
lados. Determine o número de vértices desse
e. 8 e 6 poliedro.

90
Geometria de posição e métrica Matemática

69. UFPA 73. Unifei-MG


Um poliedro convexo tem 6 faces e 8 vértices. Considere a seguinte proposição: “Construir
O número de arestas é: um octaedro convexo que possua três faces
a. 6 triangulares e as outras faces quadrangulares”.
b. 8 – Se essa proposição for possível, calcule
o número de vértices desse octaedro.
c. 10
– Se essa proposição for impossível, justifique.
d. 12
74. Fuvest-SP
e. 14
O ponto P é vértice de um poliedro e pertence a k
70. Cesgranrio-RJ faces. Cada face tem n lados. Determine o número
O poliedro da figura (uma invenção de Leonar- de segmentos contidos nas faces que unem P a
do da Vinci utilizada, modernamente, na fabri- um outro vértice qualquer do poliedro.
cação de bolas de futebol) tem como faces 20 75. Fuvest-SP
hexágonos e 12 pentágonos, todos regulares.
Um poliedro convexo tem p faces triangulares,
O número de vértices do poliedro é: q faces quadrangulares e 8 vértices. Sabendo-se
que a 6 dos seus vértices concorrem q + 1 ares-
p
tas e aos outros vértices concorrem ares-
2
tas, determine o número de faces de cada tipo
nesse poliedro.
76. Mackenzie-SP
A soma dos ângulos de todas as faces de uma
pirâmide é 18π radianos. Então, o número de
lados do polígono da base da pirâmide é:
a. 8
b. 9
c. 10
a. 64
d. 11
b. 90 e. 12
c. 60
77. ITA-SP
d. 72 Numa superfície poliédrica convexa aberta, o
e. 56 número de faces é 6 e o número de vértices é
71. Fuvest-SP 8. Então, o número de arestas é:
a. 8
Quantas faces tem um poliedro convexo com 6
b. 11
PV-13-14

vértices e 9 arestas? Desenhe um poliedro que


satisfaça essas condições. c. 12
d. 13
72. Fuvest-SP
e. 14
O número de faces triangulares de uma pirâ- 78. UEL-PR
mide é 11. Pode-se, então, afirmar que esta Para explicar a natureza do mundo,
pirâmide possui: Platão [...] apresenta a teoria segundo a qual
a. 33 vértices e 22 arestas. os quatro elementos admitidos como cons-
b. 12 vértices e 11 arestas. tituintes do mundo – o fogo, o ar, a água e
a terra – [...] devem ter a forma de sólidos
c. 22 vértices e 11 arestas. regulares. [...] Para não deixar de fora um
d. 11 vértices e 22 arestas. sólido regular, atribuiu ao dodecaedro a re-
e. 12 vértices e 22 arestas. presentação da forma de todo o universo.
DEVLIN, Keith. Matemática: a ciência dos
padrões. Porto: Porto Editora, 2002, p.119

91
Matemática Geometria de posição e métrica

As figuras a seguir representam esses sólidos 79. FCC-SP


geométricos, que são chamados de poliedros O tetraedro regular ABCD tem centro O. O ân-
regulares. gulo diedro (entre dois planos) de faces OAB e
Fogo Terra Ar OAC mede:
a. 30°
b. 60°
c. 120°
d. 135°
Universo Água
e. 150°
80. Fuvest-SP
De cada uma das pontas de um tetraedro re-
gular de aresta 3a corta-se um tetaedro regu-
lar de aresta a.
a. Qual o número de vértices, faces e
Um poliedro é um sólido limitado por polígo- arestas de poliedro resultante?
nos. Cada poliedro tem um certo número de
polígonos em torno de cada vértice. Uma das b. Calcule a área total da superfície desse
figuras anteriores representa um octaedro. A poliedro.
soma das medidas dos ângulos em torno de
cada vértice desse octaedro é:
a. 180°
b. 240°
c. 270°
d. 300°
e. 324°

PV-13-14

92
Geometria de posição e métrica Matemática

Capítulo 03
81. Cesgranrio-RJ O volume do sólido representado é dado por:

a. 9x
3
Se a diagonal de uma face de um cubo mede
5 2 , então o volume desse cubo é: 8
a. 600 3 x3
b.
b. 625 8
c. 225 c. 3x3
d. 125 3x 3
d.
e. 100 3 2
82. Unesp e. 7x3
Sendo ABCDA´B´C´D um cubo, calcule o seno 85. Unesp
do ângulo a.
Quantos cubos A precisa-se empilhar para
C´ D´
formar o paralelepípedo B?

D
C
α
A B A B
83. PUC-RJ
a. 60
Considere um paralelepípedo retangular com
lados 2, 3 e 6 cm. A distância máxima entre b. 47
dois vértices desse paralelepípedo é: c. 94
a. 7 cm d. 39
b. 8 cm e. 48
c. 9 cm 86. Unicamp-SP
d. 10 cm Ao serem retirados 128 litros de água de uma
e. 11 cm caixa d´água de forma cúbica, o nível da água
84. UEL-PR baixa 20 centímetros.
O sólido representado na figura a seguir é a. Calcule o comprimento das arestas da
formado por um cubo de aresta de medida x referida caixa.
2 b. Calcule sua capacidade em litros (1 litro
que se apoia sobre um cubo de aresta de medida x.
PV-13-14

equivale a 1 decímetro cúbico).


x

H
2
I
87. Fatec-SP
A diagonal da base de um paralelepípedo reto
retângulo mede 8 cm e forma um ângulo de
J G 60° com o lado menor da base.
M
L H G

E
F
x
D
C
8 60°
D C

A B A B

93
Matemática Geometria de posição e métrica

Se o volume desse paralelepípedo é 144 cm3, 1


então a sua altura mede, em centímetros: c. a2
2
a. 5 3
2
b. 4 3 d. a2
3
c. 3 3
5
d. 2 3 e. a2
6
e. 3
88. UFMG 91. Fuvest-SP
Todos os possíveis valores para a distância Em um bloco retangular (isto é, paralelepípedo
entre dois vértices quaisquer de um cubo de 27
reto-retângulo) de volume , as medidas
aresta 1 são: 8
a. 1, 2 e 3 das arestas concorrentes em um mesmo vér-
tice estão em progressão geométrica. Se a me-
b. 1, 2 e 3
dida da aresta maior é 2, a medida da aresta
c. 1, 3 e 2 menor é:
d. 1, 2
a. 7
89. Vunesp 8
Seja x um número real positivo. O volume de 8
b.
um paralepípedo reto-retângulo é dado, em 8
função de x, pelo polinômio x3 + 7x2 + 14 x + 8.
Se uma aresta do paralelepípedo mede x + 1, a c. 9
área da face perpendicular a essa aresta pode 8
ser expressa por: 10
d.
a. x2 – 6x + 8 8
b. x2 + 14x + 8 11
e.
c. x2 + 7x + 8 8
d. x2 – 7x + 8 92. UFSC modificado
e. x2 + 6x + 8 Usando um pedaço retangular de papelão, de
90. Unifesp dimensões 12 cm e 16 cm, desejo construir
uma caixa sem tampa, cortando, em seus can-
Um cubo de aresta de comprimento a vai ser
tos, quadrados iguais de 2 cm de lado e dobran-
transformado num paralelepípedo reto-retân-
do, convenientemente, a parte restante. Qual
gulo de altura 25% menor, preservando-se,
será a terça parte do volume da caixa, em cm3?
porém, o seu volume e o comprimento de uma
93. UFG-GO modificado
PV-13-14

de suas arestas.
Uma caixa d’água com 5,5 m de comprimento,
5 m de largura e 4,4 m de altura tem a forma
de um paralelepípedo reto-retângulo como
mostra a figura abaixo.
A diferença entre a área total (a soma das áre-
as das seis faces) do novo sólido e a área total
do sólido original será: 4,4
1
a. a2
6
1 2 5
b. a
3 5,5

94
Geometria de posição e métrica Matemática

Dentre as afirmações: Se M, N, P e Q são os pontos médios das ares-


I. O volume da caixa é 1,21 · 108 cm3. tas a que pertencem, então o volume do pris-
ma DMNCHPQG é:
II. Se uma torneira tem vazão de 12 m3
por hora, ela levará mais de 12 horas a. 0,625 cm3
para encher essa caixa d’água. b. 0,725 cm3
III. A área das faces laterais da caixa é 96 m2. c. 0,745 cm3
IV. Se uma lata de tinta é suficiente para d. 0,825 cm3
pintar uma superfície de 8,8 m2, então e. 0,845 cm3
serão gastas mais de 10 latas para pintar
as faces laterais internas dessa caixa. 96. PUC-SP
V. Aumentando-se o comprimento e a lar- A área total de um paralelepípedo retângulo
gura da caixa em 10%, a área da base
também aumentará em 10%. mede 28 m2 e a diagonal 21 m. Sabendo que
pode-se garantir que: as dimensões desse sólido estão em progressão
geométrica, calcule-as.
a. somente uma é verdadeira.
b. somente duas são verdadeiras. 97. PUCCamp-SP
c. somente três são verdadeiras. Um bloco maciço de ferro tem a forma de um
d. somente quatro são verdadeiras. paralelepípedo retângulo com dimensões de
e. todas são verdadeiras. 15 cm de comprimento, 7,5 cm de largura e
4 cm de altura. Quantos gramas tem esse blo-
94. UFSC co se a densidade do ferro é 7,8 g/cm3?
Na figura a seguir, que representa um cubo, o perí- a. 35,1
metro do quadrilátero ABCD mede 8(1 + 2 ) cm. b. 234
B c. 351
d. 2.340
A e. 3.510
98. ITA-SP
C São dados dois cubos, I e II, de áreas totais S1
e s2 e de diagonais d1 e d2, respectivamente.
D
Sabendo-se que S1 – S2 = 54 m2 e que d2 = 3 m,
d
então o valor da razão 1 é:
Calcule o volume do cubo em cm3. d2
a. 3
PV-13-14

95. Vunesp 2
As arestas do cubo ABCDEFGH, representado
b. 5
pela figura, medem 1 cm. 2
A M B c. 2

C N d. 7
D 3
e. 3

E
99. ITA-SP
P
F As dimensões x, y e z de um paralelepípedo
Q retângulo estão em progressão aritmética.
H G Sabendo que a soma dessas medidas é igual

95
Matemática Geometria de posição e métrica

a 33 cm e que a área total do paralelepípedo é 103. UFRGS-RS


igual a 694 cm2, então o volume desse parale- A figura a seguir representa a planificação de
lepípedo, em cm3 é igual a: um sólido.
a. 1.200
5
b. 936
4 3
c. 1.155
5
d. 728
e. 834 5 5
100. ITA-SP modificado
4 3
Dado um paralelepípedo retângulo de volume
5
V, cujas arestas estão em progressão geomé-
trica de razão q, podemos garantir que sua
área total é dada por: 4 3
2

a. 2 V3
⋅ (q2 + q + 1)
q
2 O volume desse sólido é:
b. V ⋅ (q2 + q + 1)
3
a. 20 3
q
b. 75
2
c. 50 3
c. V 3 ⋅ (q2 + q + 1)
q+1 d. 100
e. 100 3
V2
d. ⋅ (q + 1)
q3 104. FEI-SP
V3 De uma viga de madeira de seção quadrada de
e. 2 ⋅ (q2 + q + 1) lado d = 10 cm extrai-se uma cunha de altura
q h =15 cm, conforme a figura.
101.
Seja um prisma reto de base hexagonal regu-
lar. A aresta lateral mede o dobro da aresta
da base e o perímetro de cada base é 72 cm.
Calcule a razão entre o volume do prisma e o
apótema da base.
102. 15
PV-13-14

Seja um prisma hexagonal regular de aresta da


base a e altura igual a 2a. Calcule o valor do
10
volume e da área total desse sólido.
10
F
a E
a
A
O volume da cunha é:
a
a
D a. 250 cm3
B
C b. 500 cm3
L 2a c. 750 cm3
K
G d. 1.000 cm3
J
e. 1.250 cm3
H
I

96
Geometria de posição e métrica Matemática

105. ITA-SP 108. UFES


Dado um prisma hexagonal regular, sabe-se O canal de um rio tem a forma de uma calha
que sua altura mede 3 cm e que sua área la- cujo corte transversal é dado pela figura abai-
teral é o dobro da área de sua base. Calcule o xo. O fundo e cada um dos lados da seção
volume do prisma em cm3. transversal medem 4 metros. O canal é retilí-
106. Unicamp-SP neo e tem 50 km de extensão.
A figura abaixo representa um prisma reto
cujas bases são hexágonas regulares. Os lados 4m 4m
dos hexágonos medem 5 cm cada um e a altu-
60° 60°
ra do prisma mede 10 cm.
4m
A’

10 cm O volume desse canal, em milhares de m3, é:


A a. 200 3
5 cm C
b. 300 3
a. Calcule o volume do prisma.
c. 400 3
b. Encontre a área da secção deste prisma
pelo plano que passa pelos pontos A, C e A’. d. 500 3
107. UFMG e. 600 3
Observe a figura: 109. UFOP-MG

3 3
Uma chapa retangular de alumínio de 1 m por
60 cm será utilizada para fazer um abrigo de
forma triangular, sendo dobrada na linha mé-
3 2 2 2 2 3 dia de sua extensão de modo que as abas for-
3
mem um ângulo a. Veja a seguinte figura:
50 cm
5 5 5 5 5 5

3 2 3 2 3
2 2
60 cm

3 3

Um prisma reto de base pentagonal foi desdo-


PV-13-14

brado obtendo-se essa figura, na qual as linhas 1m


tracejadas indicam as dobras. O volume desse
prisma é:
9 3
a. 6 + B
4
α
45 3 50 cm
b. D
4
9 3 60 cm
c. 30 + A C
4
45 3
d. 30 +
4

97
Matemática Geometria de posição e métrica

a. A área do triângulo ABC depende de α.


Seja A(α) essa área, em cm2. Calcule o b.
( 7)
volume do abrigo em função de A(α), 7
em cm3.
b. Determine α de modo que o volume do c.
( 3)
abrigo seja máximo. Calcule esse volu- 2
me em cm3, em litros e em m3. 1
d.
110. UFPE 4
Um prisma com 3 m de altura tem seção trans- e. 2
versal como se mostra na figura a seguir: 5
7m
113. Unifor-CE
A figura a seguir é um prisma oblíquo cuja base
2m 4m é um triângulo equilátero de perímero 18 cm.
2m

3m
Calcule o volume, em m , desse prisma.
3

111. Unifei-MG 10 cm
3
A área total de um cubo é 18 4 m2 e esse
cubo é equivalente a um prisma triangular 60°
regular cuja aresta lateral mede o triplo da
aresta da base. Então, a aresta da base desse
prisma mede, em metros: O volume desse prisma, em centímetros cúbidos,
3
a. 4 é igual a:
b. 3 a. 270
c. 2 b. 135
d. 1 c. 45 3
d. 45 2
112. Fatec-SP
e. 45
A figura a seguir é um prisma reto, cuja base é
114. UEG-GO
um triângulo equilátero de 10 2 cm de lado e
Com uma folha de zinco retangular de dimen-
cuja altura mede 5 cm. sões 40 cm por 3 m, constrói-se uma calha na
forma “V”, conforme ilustra a figura abaixo.
PV-13-14

120°
20 cm
E 3m

Considerando-se que seja possível encher to-


A C
talmente a calha de água, o volume da água
acumulada, em m3, é de:
D F a. 0,03 3
Se M é o ponto médio da aresta DF, o seno do b. 0,04 3
ângulo BME é: c. 0,05 3

a.
( 5) d. 0,06 3
5 e. 0,07 3

98
Geometria de posição e métrica Matemática

115. Mackenzie-SP 118. PUC-SP


Um prisma reto de base quadrada teve os la- A base de um prisma reto é um triângulo de
dos da base e a altura diminuídos de 50%. O lados iguais a 5 m, 5m e 8 m e a altura tem 3 m;
seu volume ficou diminuído de: o seu volume será:
a. 50% a. 12 m3
b. 75% b. 24 m3
c. 87,5% c. 36 m3
d. 48 m3
d. 85%
e. 60 m3
e. 60%
119. Unifesp
116. Vunesp-SP Considere a equação x3 – Ax2 + Bx – C = 0, onde
Considere o sólido da figura (em azul), cons- A, B e C são constantes reais. Admita essas
truido a partir de um prisma retangular reto. constantes escolhidas de modo que as três raí-
zes da equação são as três dimensões, em cen-
E tímetros, de um paralelepípedo reto-retângu-
C
F lo. Dado que o volume desse paralelepípedo é
9 cm3, que a soma das áreas de todas as faces
B é 27 cm2 e que a soma dos comprimentos de
A G todas as arestas é 26 cm, pedem-se:
a. os valores de A, B e C;
b. a medida de uma diagonal (interna) do
Se AB = 2 cm, paralelepípedo.
D
AD = 10 cm, 120. Vunesp
FG = 8 cm e Para calcularmos o volume aproximado de
um iceberg, podemos compará-lo com sóli-
BC = EF = x cm,
dos geométricos conhecidos. O sólido da figura
o volume do sólido, em cm3, é: a seguir, formado por um tronco de pirâmide
a. 4x (2x + 5) regular de base quadrada e um paralelepípedo
b. 4x (5x + 2) reto-retângulo, justapostos pela base, represen-
ta aproximadamente um iceberg no momento
c. 4 (5 + 2x) em que se desprendeu da calota polar da Terra.
d. 4x2 (2 + 5x) As arestas das bases maior e menor do tronco de
e. 4x2 (2x + 5) pirâmide medem, respectivamente, 40 dam e
30 dam, e a altura mede 12 dam.
117. Mackenzie-SP
30 dam
A área total do sólido abaixo é:
PV-13-14

4 5
2
3
12 dam
7 H
40 dam

13
a. 204
Passado algum tempo do desprendimento do
b. 206 iceberg, o seu volume era de 23.100 dam3, o que
3
c. 222 correspondia a do volume inicial. Determine a
4
d. 244 altura H, em dam, do sólido que representa o
e. 262 iceberg no momento em que se desprendeu.

99
Matemática Geometria de posição e métrica

Capítulo 04
121. Unirio-RJ 124. UFRGS-RS
Uma pirâmide está inscrita num cubo, como Considere uma pirâmide regular de base qua-
mostra a figura a seguir. drada, construída a partir do padrão plano
abaixo.

a
Sabendo-se que o volume da pirâmide é de 6 m3,
então o volume do cubo, em m3, é igual a:
a. 9
b. 12
c. 15 Se a altura da pirâmide é o dobro do lado a da
d. 18 base, o valor de h no padrão é:
e. 21  17 
a. h =  a
122. UECE  2 
Em uma pirâmide quadrangular regular, uma aresta
da base mede 2 2 cm e uma aresta lateral mede
b. h = ( 5 )a
 22 
22 cm . O volume dessa pirâmide, em cm3, é: c. h = 
 2 
a. 7 2
b. 8 2
d. h = ( 6 )a
c. 9 2 5
e. h =   a
d. 10 2  2
123. UFRGS-RS 125. Cesgranrio-RJ
Em uma pirâmide regular, a base é um quadra- Uma pirâmide quadrangular regular tem todas
do de lado a. Suas faces laterais são triângulos as arestas iguais a x. O volume dessa pirâmide é:
equiláteros. O volume dessa pirâmide é:
(x 2 )
3
PV-13-14

a. 2 a3 a.
12 3

b. 2 a3 b.
(x 2 )
3

6 6

c.
2 3
a c.
(x 3 )
3

3 2
d. 3 a3 (x 3 )
3
12 d.
6
e. 3 a3 e. x3
6

100
Geometria de posição e métrica Matemática

126. ITA-SP 129. Unesp


Uma pirâmide regular tem por base um qua- As arestas do prisma triangular reto mostrado
drado de lado 2 cm. Sabe-se que as faces for- na figura a seguir têm todas a mesma medida.
mam com a base ângulos de 45°. Então a razão Secciona-se o prisma por meio de um plano
entre a área da base e a área lateral é igual a: pelos vértices R e Q e por um ponto M da ares-
a. 2 ta AB.
b. 1
A
3 C P
M
c. 6

d.
( 2)
B
2

e. ( )
3 R Q
3 Para que o tetraedro MBQR tenha volume
1
127. ITA-SP igual a do volume do outro sólido em que
A área lateral de uma pirâmide quadrangular re- 3
gular de altura 4 m e de área da base 64 m2 vale: se dividiu o prisma, deve-se ter BM igual a:
a. 128 m2
a. 3 BA
b. 64 2 m2 4
c. 135 m2
b. 2 BA
d. 60 2 m2 3
e. 32( 2 + 1) m2
c. 3 BA
128. UFRGS-RS 5
O valor numérico de cada aresta de um cubo
é 2, e os pontos P, Q e R são pontos médios d. 1 BA
de três arestas, como no desenho a seguir. Um 3
plano passando pelos pontos P, Q e R secciona
o cubo em dois sólidos. e. 1 BA
6
Q
P 130. UFPR
R Na figura abaixo, está representada uma pirâ-
mide de base quadrada que tem todas as ares-
PV-13-14

tas com mesmo comprimento.


A razão entre o volume do sólido menor e o
volume do cubo é: V

a. 1
48
C
b. 1 D
32
A B
c. 1
24 a. Sabendo-se que o perímetro do triân-
gulo DBV é igual a 6 + 3 2, qual é a altura
d. 1
16 da pirâmide?
b. Quais são o volume e a área total da
e. 1 pirâmide?
12

101
Matemática Geometria de posição e métrica

131. UFPR b. determine o volume da pirâmide em


Um cubo tem área total igual a 150 m . O volume
2 função de a;
da pirâmide quadrangular regular que tem c. determine os valores de a para os quais
como vértice o centro de uma das faces desse se pode construir uma pirâmide da ma-
cubo e como base a face oposta a esse vértice neira descrita.
é, em m3, igual a: 134. Vunesp
a. 125 Na periferia de uma determinada cidade bra-
3 sileira, há uma montanha de lixo urbano acu-
b. 125 mulado, que tem a forma aproximada de uma
6 pirâmide regular de 12 m de altura, cuja base
c. 125 é um quadrado de lado 100 m. Considere os
dados, apresentados em porcentagem na tabe-
d. 150 la, sobre a composição dos resíduos sólidos
e. 25 2 urbanos no Brasil e no México.
132. UEG-GO Orgânico Metais Plásticos Papelão Vidro Outros
País (%)
Uma barraca de lona, em forma de pirâmide (%) (%) (%) (%) (%) (%)

de base quadrada, tem as seguintes medidas: Brasil 55 2 3 25 2 13


base com 3 metros de lado e laterais com tri-
ângulos de 2,5 m de altura. México 42,6 3,8 6,6 16,0 7,4 23,6

A lona utilizada na construção da barraca, nas Cempre/Tetra Pak Américas/EPA 2002.


laterais e na base, perfaz um total de:
Supondo que o lixo na pirâmide esteja com-
a. 9 m2 pactado, determine o volume aproximado de
b. 15 m2 plásticos e vidros existente na pirâmide de lixo
c. 20,5 m2 brasileira e quantos metros cúbicos a mais
d. 24 m2 desses dois materiais juntos existiriam nessa
mesma pirâmide, caso ela estivesse em terri-
e. 39 m2 tório mexicano.
133. UFMG 135. UFU-MG
Uma pirâmide de base quadrada é construída Na figura abaixo, temos um cubo ABCDEFGH
recortando-se e dobrando-se uma cartolina de aresta a = 6 cm. Os pontos I, J, K, L, M e N
quadrada de 100 cm de lado, como mostrado são pontos médios das arestas a que pertencem.
nesta figura:
E
a M
F
H
PV-13-14

L G
a
N
100

K A
a 50
I
B
D
a 50 J
C

Considerando essas informações: Determine o volume da pirâmide de base


a. determine a altura da pirâmide em fun- hexagonal IJKLMN e vértice H.
ção de a;

102
Geometria de posição e métrica Matemática

136. UFRGS-RS 139. Fuvest-SP


A figura abaixo representa a planificação de O cubo ABCDEFGH possui arestas de compri-
uma pirâmide de base quadrada com AB = 6 cm, mento a. O ponto M está na aresta AE e
sendo ADV triângulo equilátero. AM = 3 · ME. Calcule:
H G

E F
D C

M
V C
D

A B
A B

a. o volume do tetraedro BCGM;


O volume desta pirâmide, em centímetros b. a área do triângulo BCM;
cúbicos, é: c. a distância do ponto B à reta suporte do
a. 12 3 segmento CM.
b. 27 3 140. PUC-SP
c. 36 3 A distância de um ponto do espaço ao plano
d. 72 3 de um triângulo equilátero ABC de lados 6 m
e. 108 3 equidistante 4 m de cada vértice mede:
137. Vunesp a. 1 m
Calcule a altura H e o seno do ângulo diedro b. 2 m
formado por duas faces quaisquer de um te- c. 3 m
traedro regular cujas arestas medem a cm. d. 4 m
e. 5 m
141. PUCCamp-SP
a a
H Um octaedro regular é um poliedro constituí-
do por 8 faces triangulares congruentes entre
si e ângulos poliédricos congruentes entre si,
a
conforme mostra a figura a seguir.
a a

138. Fuvest-SP
PV-13-14

A figura a seguir é a planificação de um polie-


dro convexo (A = B = C = D; E = F). Calcule seu
volume.
B

A C
Se o volume desse poliedro é 72 2 cm3, a
G H medida de sua aresta, em cm, é:
5 2 5 2
a. 2
I
E 13
13
D b. 3
13
c. 3 2
13 5 2
d. 6
F
V 13 e. 6 2

103
Matemática Geometria de posição e métrica

142. ITA-SP 144. Mackenzie-SP


Um tetraedro regular tem área total igual a Pelo centro A de um quadrado MNPQ de lado
6 3 cm2. Então, sua altura, em cm, é igual a: d = 1, levanta-se uma perpendicular ao plano
do quadrado e une-se um ponto T dessa per-
a. 2
pendicular aos vértices do quadrado, obtendo-se,
b. 3 deste modo, quatro triângulos equiláteros.
c. 2 2 O volume do poliedro de vértice T e base AMN
é:
d. 3 2
a. 2
e. 2 3 6

143. Mackenzie-SP b. 2
12
O volume do sólido da figura a seguir é:
c. 2
A
24

d. 2
36
1
e. 2
48
145.
B
C Na figura a seguir, PMN é a seção do prisma
reto, triangular e regular com um plano a, que
D faz 60° com sua base.
k
CÂB = DÂC = 30°
 
Dados : BCD = 60°

AC ⊥ BDC

P
a. 3
12 A N B

b. 3 M
PV-13-14

18 C

c. 3 Se M e N são pontos médios e se o volume do


20 sólido assinalado é 3 , então k mede:
a. 1
d. 3
24 b. 2
c. 3
e. 3 d. 4
36 e. 5

104
Geometria de posição e métrica Matemática

146. Cesgranrio-RJ 149. Cesgranrio-RJ


Uma folha de papel colorido, com a forma de Para fazer o telhado de uma casa de cartolina,
um quadrado de 20 cm de lado, será usada um quadrado de centro O e de lado 2d é re-
para cobrir todas as faces e a base de uma pi- cortado, como mostra a figura I.
râmide quadrangular regular com altura 12 cm
e apótema da base medindo 5 cm. Após se ter A B
concluído essa tarefa, e levando-se em conta
que não houve desperdício de papel, a fração
percentual que sobrará dessa folha de papel
H C
corresponde a:
a. 20%
2
b. 16%
O
c. 15%
G D
d. 12%
e. 10%
147. Fuvest-SP F E
A pirâmide de base retangular ABCD e vértice  3
E representada na figura tem volume 4. Se M é
o ponto médio da aresta AB e V é o ponto mé- Figura I
dio da aresta EC , então o volume da pirâmide
Os lados AB = CD = EF = GH medem d 3.
de base AMCD e vértice V é:
E
Montado o telhado (figura II), sua altura é:

O
V

C
D
DE
A
M B AH
h
a. 1
b. 1,5 GF
c. 2
d. 2,5 Figura II
e. 3
a. 
148. Fuvest-SP 2
PV-13-14

Um telhado tem a forma da superfície lateral 2


de uma pirâmide regular de base quadrada. O b.
lado da base mede 8 m e a altura da pirâmi- 5
de 3 m. As telhas para cobrir esse telhado são
vendidas em lotes que cobrem 1 m2. Supondo c. 3
que possa haver 10 lotes de telhas desperdiça- 10
das (quebras e emendas), o número mínimo
de lotes de telhas a ser comprado é: (
d. 2 – 3  )
a. 90
b. 100 e.  3
5
c. 110
d. 120
e. 130

105
Matemática Geometria de posição e métrica

150. UFPE 153. UFSCar-SP


O tetraedro ABCD tem aresta AB medindo 12; Os segmentos de reta que unem os pontos
a face ABD tem área 48, e a face ABC tem área centrais das faces adjacentes de um cubo de-
60. Se o ângulo entre as faces ABC e ABD mede terminam um octaedro (ver figura a seguir). Se
30°, qual o volume do tetraedro? a aresta do cubo mede dcm, então o volume
D do octaedro é igual a:

C
a.  cm3
3
A 30o
8
 3
b. cm3
B
4
151. Fuvest-SP
c.  cm3
3

Dois planos, p1 e p2, interceptam-se ao longo 5


de uma reta r, de maneira que o ângulo entre
d.  cm3
3
p
eles meça a radianos, 0 < a < . Um triângulo 7
2
equilátero ABC, de lado d, está contido em p2, e.  cm3
3

de modo que AB esteja em r. Seja D a proje- 6


ção ortogonal de C sobre o plano p1, e suponha 154. ITA-SP
que a medida θ, em radianos, do ângulo CÂD,
6 As arestas laterais de uma pirâmide regular de
satisfaça sen θ = . 12 faces laterais têm comprimento d. O raio
4
Nessas condições, determine, em função de d: do círculo circunscrito ao polígono da base
a. o valor de a; 2
desta pirâmide mede  . Então, o volume
b. a área do triângulo ABD; 2
desta pirâmide vale:
c. o volume do tetraedro ABCD.
a. 3 2 3
152. UFOP-MG
De um cubo de 2 cm de aresta (ABCDEFGH), b. 2d3
retirou-se uma pirâmide (EFGB), resultando c. 3
3
PV-13-14

num sólido ABCDEGH, conforme ilustram as 2


figuras a seguir.
H H G
d. 23
G G
A E 23
F – E F = e.
4
D C D C
155. Unicamp-SP
A B B A B
Os segmentos de reta que unem os pontos
centrais das faces adjacentes de um cubo, cuja
Para o sólido resultante (ABCDEGH), determine:
aresta mede l cm, determinam um octaedro.
a. a área total;
a. Faça uma figura ilustrando a situação
b. o volume. descrita.
b. Calcule o volume do octaedro.

106
Geometria de posição e métrica Matemática

156. Fuvest-SP A distância de B até o plano que contém A, D


No sólido S representado na figura a seguir, a e C é igual a:
base ABCD é um retângulo de lados AB = 2λ e a. 11
AD = λ ; as faces ABEF e DCEF são trapézios; as 4
faces ADF e BCE são triângulos equiláteros e o
seguimento EF tem comprimento λ . 14
b.
Determine, em função de λ, o volume de S. 4

c. 11
E

2
F
d. 13
C
2
B
e. 3 7
2
D

A 159. Fuvest-SP
157. Fuvest-SP A base ABCD da pirâmide ABCDE é um retângulo
Pedrinho, brincando com seu cubo mágico, de lados AB = 4 e BC = 3.
colocou-o sobre um copo, de maneira que: As áreas dos triângulos ABE e CDE são, respec-
• apenas um vértice do cubo ficasse no in- tivamente, 4 10 e 2 37 . Calcule o volume
terior do copo, conforme ilustra a foto; da pirâmide.
• os pontos comuns ao cubo e ao copo E
determinassem um triângulo equilátero.
Sabendo-se que a borda do copo é uma circun-
ferência de raio 2 3 , determine o volume da
parte do cubo que ficou no interior do copo. C
D

A B

160. ITA-SP
Dada uma pirâmide regular triangular, sabe-se
que sua altura mede 3a cm, em que a é a
medida da aresta da base. Então, a área total
dessa pirâmide, em cm2, vale:
PV-13-14

158. UFSCar-SP a2 327


a.
A figura indica um paralelepípedo reto-retân- 4
gulo de dimensões 2 , 2 e 7 , sendo A, B, C
e D quatro de seus vértices. a2 109
b.
2
A
a2 3
c.
7 2
a2 3 (2 + 33)
d.
D 2
B
2 a2 3 (1 + 109 )
e.
2 C 4

107
Matemática Geometria de posição e métrica

Capítulo 05
161. ITA-SP Essa água é despejada no recipiente menor,
A área lateral de um cilindro de revolução de x alcançando a altura h de:
metros de altura é igual à área de sua base. O a. 32 cm
volume desse cilindro, em m3, é:
b. 24 cm
a. 2πx3
b. 4πx3 c. 16 cm
c. π 2 x3 d. 12 cm
d. π 3 x3 e. 10 cm
e. 6πx3 164. PUC-SP
162. Unicamp-SP As projeções ortogonais de um cilindro sobre
Um cilindro circular reto é cortado por um pla- dois planos perpendiculares são, respectiva-
no não paralelo à sua base, resultando no sóli- mente, um círculo e um quadrado. Se o lado
do ilustrado na figura a seguir. do quadrado é 10, qual é o volume do cilindro?
B a. 1.000π
b. 750π
D c. 500π
a d. 250π
b e. 100π
165. ITA-SP
A
C
Num cilindro circular reto, sabe-se que a altura
h e o raio da base R são tais que os números
p, h e R formam, nesta ordem, uma progres-
Calcule o volume desse sólido em termos do são aritmética de soma 6p. O valor da área to-
raio da base r, da altura máxima AB = a e da tal desse cilindro é:
altura mínima CD = b .
a. π3
Justifique seu raciocínio. b. 2π3
163. UEL-PR c. 15π3
Dois recipientes cilíndricos têm altura de 40 cm e d. 20π3
e. 30π3
PV-13-14

raios da base medindo 10 cm e 5 cm. O maior


deles contém água até 1 de sua capacidade. 166. Unisa-SP
5 Se a altura de um cilindro circular reto é igual
ao diâmetro da base, então a razão entre a
área total e a área lateral do cilindro é igual a:
a. 3
b. 3
h 2
c. 2pR2
d. 2
10 5 e. 1

108
Geometria de posição e métrica Matemática

167. Vunesp 171. Fatec-SP


Num tonel de forma cilíndrica, está depositada Um tanque, com a forma de uma cilindro cir-
uma quantidade de vinho que ocupa a metade cular reto, tem 4 m de altura. Ao esboçar um
de sua capacidade. Retirando-se 40 litros de projeto para a reforma desse tanque, um en-
seu conteúdo, a altura do nível do vinho bai- genheiro percebeu que, independentemente
xa 20%. O número que expressa a capacidade de aumentar o raio de sua base ou a sua altura
desse tonel, em litros, é: em 5 m, o volume, nos dois casos, sofreria o
mesmo acréscimo de x m3.
a. 200 Assim, o volume do tanque original, em metros
b. 300 cúbicos, é:
c. 400 a. 300 π
d. 500 b. 350 π
e. 800 c. 375 π
168. UFPE d. 400 π
Um queijo tem a forma de um cilindro circu- e. 425 π
lar reto com 40 cm de raio e 30 cm de altura. 172. PUC-MG
Retira-se do mesmo uma fatia, através de dois Um pedaço de cano cilíndrico reto tem espessu-
cortes planos contendo o eixo do cilindro e ra igual a 50 mm. Se o volume interno do cano é
formando um ângulo de 60°. Se V é o volume, igual ao volume do material gasto para construí-
em cm3, do que restou do queijo (veja a figura lo, então o seu raio interno, em mm, é igual a:
V a. 50 2
a seguir), determine .
103 ⋅ π b. 50( 2 – 1)
c. 50(2 – 2 )
d. 50(1 + 3 )
60o
e. 50( 2 + 1)
173. Mackenzie-SP

169. UFPE Uma lata tem forma cilíndrica com diâmetro


da base e altura iguais a 10 cm. Do volume
Um contêiner, na forma de um cilindro circular
reto, tem altura igual a 3 m e área total (área total, 4 é ocupado por leite em pó. Adotando-se
da superfície lateral mais áreas da base e da 5
tampa) igual a 20π m2. Calcule, em metros, o π = 3, o volume de leite em pó, em cm3, contido
raio da base desse contêiner. na lata é:
a. 650
170.
b. 385
PV-13-14

Felipe possui um recipiente cilíndrico de 40 cen- c. 600


tímetros de diâmetro de base e 6 metros de al-
tura que possui duas torneiras. Quando somente d. 570
a primeira está aberta, ele esvazia em 2 horas e e. 290
quando somente a segunda está aberta, ele 174. Unimep-SP
esvazia em 3 horas. Qual a vazão aproximada, Faz-se girar um quadrado de lado 1 cm em torno
sabendo que Felipe manteve as duas torneiras de um de seus lados. A área total do sólido
abertas. resultante vale:
a. 10 litros/minuto a. 4π cm2
b. 30 litros /minuto b. π cm2
c. 8 π cm2
c. 1 m3/minuto
d. p cm2
d. 36 litros/minuto 2
e. 10 litros/minuto e. 2p cm2

109
Matemática Geometria de posição e métrica

175. FGV-SP altura, obtemos um retângulo. Nos cálculos,


Um produto (creme de leite) pode ser embalado use o valor aproximado π = 3.
6,9 m
em dois tipos de latas, A e B, ambas com formato
de cilindro reto. Suas características são:
4,5 m

• Tipo A: raio da base 8 cm e altura 2 cm;


Dimensões do telhado
• Tipo B: altura igual ao diâmetro da base.
0,05 m
As duas latas devem ter o mesmo volume.
Uma delas gasta de material na sua constru- 0,23 m
ção x% a mais em relação à outra. O valor de x
é aproximadamente igual a: Caixa Caixa
desmontada
a. 33,4
178. Fuvest-SP
b. 44,5 Uma garrafa de vidro tem a forma de dois cilindros
c. 66,7 sobrepostos. Os cilindros têm a mesma altura,
d. 55,6 4 cm, e raios de bases R e r, respectivamente.
e. 77,8
176. UEL-PR 4 cm

Considere um cilindro circular reto que tem 4 cm r


de altura. Aumentando-se indiferentemente o
raio da base ou a altura desse cilindro em 12 cm,
obtém-se, em qualquer caso, cilindros de volu- 4 cm
mes iguais. A medida, em centímetros, do raio x
do cilindro original é: R
a. 12 Se o volume V(x) de um líquido que atinge a
b. 10 altura x da garrafa se expressa segundo o gráfico
c. 8 a seguir, quais os valores de R e r?
V (x) (cm3)
d. 6
e. 4 44π
177. Unesp
Por ter uma face aluminizada, a embalagem 18π

de leite “longa vida” mostrou-se convenien-


te para ser utilizada como manta para sub- 0 2 4 6 8 x (cm)

coberturas de telhados, com a vantagem de 179. ITA-SP


PV-13-14

ser uma solução ecológica que pode contri- O raio de um cilindro de revolução mede 1,5 m.
buir para que esse material não seja jogado Sabe-se que a área da base do cilindro coincin-
no lixo. Com a manta, que funciona como de com a área da seção determinada por um
isolante térmico, refletindo o calor do sol plano que contém o eixo do cilindro. Determi-
para cima, a casa fica mais confortável. De- ne a área total do cilindro em m2.
termine quantas caixinhas precisamos para 180. ITA-SP
fazer uma manta (sem sobreposição) para Sabe-se que a média harmônica entre o raio
uma casa que tem um telhado retangular e a altura de um cilindro de revolução vale 4.
com 6,9 m de comprimento e 4,5 m de lar- Quanto valerá a relação do volume para a área
gura, sabendo-se que a caixinha, ao ser des- total desse cilindro?
montada (e ter o fundo e o topo abertos), a. 1
toma a forma aproximada de um cilindro b. 2
oco de 0,23 m de altura e 0,05 m de raio, de c. 2,5
modo que, ao ser cortado acompanhando sua d. 3

110
Geometria de posição e métrica Matemática

Capítulo 06

181. PUC-MG 185. UnB-DF


Se um cone tem 10 cm de altura e a circun- Um lápis novo de altura h é apontado.
ferência de sua base mede 9π dm, então seu
volume, em dm3, é:
h1
a. 67,5 π
b. 30π
c. 6,75π
h
d. 675π3
e. 30π3
182. Fatec-SP modificado
Considere um cone circular reto onde a altura
é o triplo do diâmetro da base. Se R é o raio da
base e V é o volume desse cone, então:
a. V = 2πR3 Se a altura h1 da ponta (veja figura) for igual a
b. V = πR3 1
da altura do lápis, então a porcentagem
c. V = 3πR3 10
de material desperdiçado (raspas de madeira
d. V = 6πR3
e grafite) é:
e. V = 8πR3
183. UFU-MG a. 4%
Desenvolvendo a superfície lateral de um cone b. 10%
circular reto de raio 4 e altura 3, obtém-se um
c. 5%
setor circular, cujo ângulo central mede:
a. 216o d. 6,6%
b. 240o 186. UFRGS-RS
c. 270o O volume do sólido gerado pela revolução de
d. 288o um triângulo equilátero de lado a em torno de
e. 180o um de seus lados é:
184. UFES a. 1 pa3
4
PV-13-14

Desenvolvendo-se a superfície lateral de um


1
b. pa3
cone reto, obtém-se um setor circular de raio
9 cm e ângulo central 200°. A área lateral des- 3
se cone mede: 1
c. pa3
a. 50π cm2 2
b. 45π cm2 3 3
d. pa
c. 31π cm2 4
d. 14π cm2
e. 4 pa3
e. 9π cm2 3

111
Matemática Geometria de posição e métrica

187. Mackenzie-SP 189. Fuvest-SP


Na figura, a base do cone reto está inscrita na Um pedaço de cartolina possui a forma de um
face do cubo. semicírculo de raio 20 cm. Com essa cartolina,
um menino constrói um chapéu cônico e o
coloca com a base apoiada sobre uma mesa.
Qual é a distância do bico do chapéu à mesa?
a. 10 3 cm
b. 3 10 cm
c. 20 2 cm
d. 20 cm
Supondo π = 3, se a área total do cubo é 54, e. 10 cm
então o volume do cone é: 190. Unesp
a. 81
2 Um paciente recebe, por via intravenosa, um
medicamento à taxa constante de 1,5 ml/min.
b. 27 O frasco do medicamento é formado por uma
2 parte cilíndrica e uma parte cônica, cujas me-
9 didas são dadas na figura, e estava cheio quan-
c. do se iniciou a medicação.
4
4 cm
d. 27
4
e. 81
4
9 cm
188. Mackenzie-SP
O setor circular da figura a seguir é a superfície
lateral de um cone cuja base tem diâmetro 4 e
área igual a k% da área total do cone. 3 cm

(Figura fora de escala)


120°
Após 4h de administração contínua, a medica-
PV-13-14

ção foi interrompida. Dado que 1 cm3 = 1 ml,


e usando a aproximação π = 3, o volume, em
ml, do medicamento restante no frasco após a
interrupção da medicação é, aproximadamente:
Então, k vale:
a. 20 a. 120
b. 25 b. 150
c. 30 c. 160
d. 35 d. 240
e. 40
e. 360

112
Geometria de posição e métrica Matemática

191. Ufla-MG 3
c. da altura do cilindro.
Parte do líquido de um cilindro completamen- 4
te cheio é transferido para dois cones idênti- 1
d. da altura do cilindro.
cos, que ficam totalmente cheios. 3
R R 193. ITA-SP
Sabendo-se que um cone circular reto tem 3 dm
H H de raio e 15 π dm2 de área lateral, o valor de
seu volume, em dm3, é:
h1
a. 9 π
b. 15 π
A relação entre as alturas do líquido restante c. 36π
no cilindro (h1) e a altura (H) do cilindro é: d. 20 π
H e. 12 π
a. h1 =
4
194. UEL-PR
H
b. h1 = Uma chapa com forma de um setor circular
2 de raio 20 cm e ângulo de x graus é manuseada
H para se transformar num cone. Se o raio da base
c. h1 = do cone obtido é r = 5 cm, então o valor de x é:
2
H
d. h1 = x
20 cm
3
H
e. h1 =
3 r = 5 cm
192. Unicamp-SP
Depois de encher de areia um molde cilíndri- a. 60°
co, uma criança virou-o sobre uma superfície b. 75°
horizontal. Após a retirada do molde, a areia c. 80°
escorreu, formando um cone cuja base tinha d. 85°
raio igual ao dobro do raio da base do cilindro. e. 90°
195. ITA-SP
O desenvolvimento da superfície lateral de um
cone reto é um setor circular de raio a e ângu-
PV-13-14

lo central igual a 60°. O volume desse cone é:


a. a ⋅ π
2

6
b. p 35 a3

R 2R c. 1 ⋅ πa3
3
A altura do cone formado pela areia era igual a: 3
a
a.
1
da altura do cilindro. d. π  
2  6
2 3
b. da altura do cilindro.
3 e. 1 ⋅ π  a  ⋅ 35
 
3  6

113
Matemática Geometria de posição e métrica

196. UFOP-MG terra retirada tenha volume 20% maior que o


Um circo com a forma de um cone circular reto volume da piscina. Nessas condições, a altura
sobre um cilindro circular reto de mesmo raio está do cone, em metros, é de:
com a lona toda furada. O dono do circo, tendo a. 4,0
obtido bom lucro com as apresentações, resol- b. 3,8
veu comprar uma nova lona. Para saber quanto c. 3,0
de lona precisava comprar, ele considerou as se-
guintes especificações: a altura do mastro central d. 2,8
vertical que sustenta a lona é de 10 m , a altura do e. 2,0
cilindro é de 3 m e o raio da circunferência, de 199. FURG-RS
24 m, como indica a figura. Que quantidade de
lona, em m2 , será necessário comprar? Um artesão produz velas natalinas na forma de
árvore de natal, conforme a figura abaixo. O só-
lido A corresponde a um cilindro equilátero e o
sólido B é um cone cuja geratriz é igual ao diâme-
tro de sua base. Sabendo que as dimensões são
dadas em centímetros e que o raio do cilindro, r ,
10 m é a terça parte do raio do cone, R , o volume, em
cm³, do molde desse enfeite, em função do R, é:

3m
24 m B

197. ITA-SP
Num cone circular reto, a altura é a média R
geométrica entre o raio da base e a geratriz. A
razão entre a altura e o raio da base é: A
r
a. 1 + 5
2 a. π R3 9 3 + 1
( )
27
b. –1 + 5 p
2 b. 20 R3
27
–1 + 5 c. π R3 (9 3 + 2)
c.
2 27
PV-13-14

3
d. –1 + 5 d. 10p R3
3 27

1+ 5 e. 11 3p R3
e. 27
2
200. ITA-SP modificado
198. UFG-GO Um cilindro equilátero de raio 3 cm está inscri-
A terra retirada na escavação de uma piscina to num prisma triangular reto, cujas arestas da
semicircular de 6 m de raio e 1,25 m de pro- base estão em progressão aritmética de razão
fundidade foi amontoada, na forma de um a; a > 0. Sabendo-se que a razão entre o volu-
cone circular reto, sobre uma superfície ho-
rizontal plana. Admita que a geratriz do cone me do cilindro e do prisma é p , calcule a área
lateral do prisma. 4
faça um ângulo de 60° com a vertical e que a

114
Geometria de posição e métrica Matemática

CAPÍTULO 07
201. Ufla-MG 205. Fuvest-SP
A intersecção de um plano com uma esfera é Uma superfície esférica de raio 13 cm é corta-
um círculo de 16p dm2 de área. Sabendo-se da por um plano situado a uma distância de
que o plano dista 3 dm do centro da esfera, o 12 cm do centro da superfície esférica, deter-
volume da esfera é: minando uma circunferência.
a. 100p dm3 O raio dessa circunferência, em cm, é:
100 a. 1
b. pdm3
3 b. 2
c. 400p dm3 c. 3
d. 500p dm3 d. 4
e. 500 pdm3 e. 5
3 206. Cesgranrio-RJ
202. UFES Uma laranja pode ser considerada uma esfera
Sejam duas esferas, uma inscrita e outra de raio R, composta por 12 gomos exatamente
cirscunscrita num cubo de aresta a = 6 cm. iguais. A superfície de cada gomo mede:
Os raios dessas duas esferas medem, respec- a. 2p R2
tivamente: b. 1 p R2
a. 6 cm e 3 2 cm 3
b. 6 cm e 3 3 cm
c. 3 p R2
c. 3 cm e 3 2 cm 4
d. 3 cm e 3 3 cm d. 3 p R2
2 cm
e. 6 cm e 6 e. 4 p R2
203. Unifenas-MG 3
O volume de uma esfera cresce 72,8% quando 207. UFRGS-RS
o raio dessa esfera aumenta: Uma esfera de raio 2 cm é mergulhada num
a. 26% copo cilíndrico de 4 cm de raio, até encostar
b. 25% no fundo, de modo que a água do copo recu-
bra exatamente a esfera.
c. 24%
d. 20%
PV-13-14

e. 15%
204. Unesp
Seja r um número real positivo e P um ponto
do espaço. O conjunto formado por todos os
pontos do espaço que estão a uma distância
de P menor ou igual a r é:
a. um segmento de reta medindo 2r e Antes de a esfera ser colocada no copo, a altu-
tendo P como ponto médio. ra de água era:
b. um cone cuja base é um círculo de cen-
27
tro P e raio r. a. cm
c. um cilindro cuja base é um círculo de 8
centro P e raio r.
d. uma esfera de centro P e raio r. b. 19 cm
6
e. um círculo de centro P e raio r.

115
Matemática Geometria de posição e métrica

211. UEL-PR
c. 18 cm
5 Na figura a seguir, são dados uma esfera de
centro O, uma reta que contém O e intercep-
d. 10 cm ta a superfície esférica nos pontos A e B e um
3 ponto C na superfície esférica.
e. 7 cm A
2
C
208. Unesp
O
Um copinho de sorvete, em forma de cone,
tem 10 cm de profundidade, 4 cm de diâme-
tro no topo e tem aí colocadas duas conchas
semiesféricas de sorvete, também de 4 cm de B
diâmetro. Se o sorvete derreter para dentro
do copinho, podemos afirmar que: Em relação às medidas dos segmentos deter-
a. não transbordará. minados na figura, é sempre verdade que:
b. transbordará. a. OC < OA
c. os dados são insuficientes. b. OB > OA
d. os dados são incompatíveis. c. AC = OC
e. todas as afirmações anteriores são fal- d. OB = OC
sas. 2
e. AB = 2OC
209. FGV-SP
212. Unitau-SP
Deseja-se construir um galpão em forma de
um hemisfério para uma exposição. Se para o Aumentando em 10% o raio de uma esfera, a
revestimento do piso utilizaram-se 78,5 m2 de sua superfície aumentará:
lona, quantos metros quadrados se utilizariam a. 21%
na cobertura do galpão? b. 11%
a. 31,4 c. 31%
b. 80 d. 24%
c. 157 e. 30%
d. 208,2 213. Fatec-SP modificado
e. 261,66 Os volumes do cilindro reto, do cone reto e da
210. Mackenzie-SP esfera, a seguir, formam, nessa ordem, uma
PV-13-14

Um frasco de perfume de forma esférica, com PG; então:


1 r
raio de 4 cm, contém perfume em de seu
4
volume total. Se uma pessoa utilizar, todos os h1 h2 r
dias, 2 ml do perfume, das alternativas abaixo,
a que indica o maior período de tempo de du- r
ração do perfume é:
a. 16 dias a. (h2)2 = 12 rh1
b. 31 dias b. h2 = 4h1
c. 54 dias c. h1 = 12 (h2)2
d. 43 dias d. h1h2 = 4r
e. 26 dias. e. (h2)2 = 4 rh1

116
Geometria de posição e métrica Matemática

214. UFF-RJ Se o diâmetro de uma bola tem aumento de


Na figura, estão representados três sólidos de mes- 1%, então o seu volume aumenta x%.
ma altura h — um cilindro, uma semiesfera e um Dessa forma, é correto afrmar que:
prisma — cujos volumes são v1, v2 e v3, respectiva- a. x ∈ [5,6)
mente.
b. x ∈ [2,3)
2r
r r c. x = 1
h
d. x ∈ [3,4)
2r e. x ∈ [4,5)
A relação entre v1, v2 e v3 é: 217. Fuvest-SP
a. V3 < V2 < V1 Um cálice em forma de cone contém V cm³
b. V2 < V3 < V1 de uma bebida. Uma cereja de forma esféri-
ca, com diâmetro de 2 cm, é colocada dentro
c. V1 < V2 < V3 do cálice. Supondo-se que a cereja repousa
d. V3 < V1 < V2 apoiada nas laterais do cálice e o líquido re-
e. V2 < V1 < V3 cobre exatamente a cereja a uma altura de 4
cm do vértice do cone, determine o valor de V.
215. Cesgranrio-RJ
Um tanque cilíndrico com água tem raio da base R.
Mergulha-se totalmente nesse tanque uma esfera
de aço e o nível da água sobe 9 R . O raio da esfera é:
16 4 cm
a. 3R
4
b. R
9
16
9
c. R
3 R
16
5
d. R
2
218. UFRGS-RS
e. R
2 R
3 Em cada um dos vértices do cubo está centra-
216. UFF-RJ da uma esfera cuja medida do diâmetro é igual
Para ser aprovada pela FIFA, uma bola de futebol à medida da aresta do cubo.
deve passar por vários testes. Um deles visa ga- A razão entre o volume da porção do cubo
rantir a esfericidade da bola: o seu “diâmetro” é ocupado pelas esferas e o volume do cubo é:
PV-13-14

medido em dezesseis pontos diferentes e, então,


a média aritmética desses valores é calculada. p
Para passar nesse teste, a variação de cada uma a.
6
das dezesseis medidas do “diâmetro” da bola com
relação à média deve ser no máximo 1,5%. Nesse p
b.
teste, as variações medidas na Jabulani, bola oficial 5
da Copa do Mundo de 2010, não ultrapassam 1%. p
c.
4
p
d.
3

p
e.
2

117
Matemática Geometria de posição e métrica

219. ITA-SP 220.


Considere um retângulo de altura h e base b A área total de um octaedro regular inscrito
e duas circunferências com diâmetro h e cen- numa esfera de volume igual a 36π m3 é:
tros nos lados do retângulo, conforme a figura
abaixo. Seja z um eixo que passa pelo centro a. 30 3 m2
destas circunferências. Calcule a área da su-
perfície do sólido gerado pela rotação da re- b. 36 3 m2
gião hachurada em torno do eixo z. c. 40 2 m2
d. 36 m2
h z e. 24 2 m2

a. πh(b –h)
b. πh(b + h)
c. πb(b – h)
d. πb(b + h)
e. πbh

PV-13-14

118
Geometria de posição e métrica Matemática

Capítulo 08
221. FAAP-SP 223. Mackenzie-SP
Um copo de chope é um cone (oco), cuja altu- Uma mistura de leite batido com sorvete é ser-
ra é o dobro do diâmetro. Se uma pessoa bebe vida em um copo, como na figura a seguir.
desde que o copo está cheio até o nível da be-
bida ficar exatamente na metade da altura do 4 cm
copo, a fração do volume total que deixou de
ser consumida é:
20 cm
a. 3
4

b. 1
2

c. 2
3 Se na parte superior do copo há uma camada
3 de espuma de 4 cm de altura, então a porcen-
d.
8 tagem do volume do copo ocupada pela espu-
ma está mais bem aproximada na alternativa:
e. 1
8 a. 65%
222. UFMG b. 60%
c. 50%
Observe a figura.
d. 45%
e. 70%
224. Vunesp
Marcos, sentindo muito calor, senta-se em um
bar e pede um chope, o qual lhe é servido em
uma “tulipa”, que é um copo na forma de um
cone invertido. O garçom chega com a bebida
ao mesmo tempo em que “Purê”, seu grande
amigo, passa em frente ao bar. Marcos grita:
— “Purê, sente-se aqui e tome a metade do
Essa taça, cujo interior tem a forma de um cone, chope desta tulipa comigo!” Purê senta-se, faz
contém suco até a metade da altura do cone in- cara de quem não sabe o que fazer e diz: —
terno. Se o volume do cone interno é igual a V, “Marcos, mas até que altura do copo eu devo
então o volume do suco nele contido é: beber o chope para que sobre exatamente a
PV-13-14

V metade para você?” Marcos pega um guarda-


a. napo de papel, uma caneta e mede a altura da
16 tulipa, que era de 20 cm. Após alguns minu-
V tos e algumas contas, Marcos diz ao amigo: —
b.
9 “Você deve beber os primeiros...
V (Use: 41/3 ≈ 1,6)
c.
8 a. 4 cm de chope na tulipa”.
b. 5 cm de chope na tulipa”.
d. V
4 c. 10 cm de chope na tulipa”.
d. 15 cm de chope na tulipa”.
e. V
3 e. 16 cm de chope na tulipa”.

119
Matemática Geometria de posição e métrica

225. UERJ 227. UTFPR


Um sólido com a forma de um cone circular Seja um enfeite em vidro na forma de um cone
reto, constituído de material homogêneo, de raio da base 9 cm e altura 12 cm. Este enfei-
flutua em um líquido, conforme a ilustração te contém areia, conforme mostra a figura (a).
abaixo. Se virarmos este enfeite com o vértice do cone
para baixo, conforme a figura (b), pode-se afir-
mar que a medida h, em cm, será igual a:

4 cm h
Se todas as geratrizes desse sólido forem di-
vididas ao meio pelo nível do líquido, a razão Areia Areia
entre o volume submerso e o volume do sólido
Figura (a) Figura (b)
será igual a:
a. 1 3
171
2 a.
16
b. 3 b. 6 3
4 3
228
c. 5 c.
3
6
d. 4 3 19
d. 7
8 e. 2 3 61
226. UFPR
228. FGV-SP
A parte superior de uma taça tem o formato
de um cone, com as dimensões indicadas na Uma pirâmide de base quadrada é seccionada
figura. por um plano paralelo à sua base, distante 2 m
dela. A área total da pirâmide menor, obtida
a. Qual o volume de líquido que essa taça pela secção, é igual à metade da área total da
comporta quando está completamente pirâmide original.
cheia?
a. Calcule a altura da pirâmide original.
b. Obtenha uma expressão para o volume
V de líquido nessa taça, em função da b. Calcule o volume do tronco de pirâmide ob-
altura x indicada na figura. tido pela secção para o caso em que a ares-
ta da base da pirâmide maior mede 3 m.
4 cm
PV-13-14

229. UEM-PR
Em uma pirâmide de altura h metros, a área
da sua base é 360 metros quadrados. Se um
R plano a, paralelo à base, determina na pirâmi-
12 cm de uma seção transversal que dista h metros
x
2
do vértice da pirâmide, então a área da seção
transversal, em metros quadrados, é igual a:
a. 160
b. 68
c. 86
d. 100
e. 90

120
H
2
Geometria de posição e métrica O Matemática
O
Figura I Figura II
230. FCMSC-SP
Um cone circular reto tem 24 cm de altura e
raio de base medindo 9 cm. Esse cone é corta-
do por dois planos paralelos à sua base e que
dividem sua altura em três partes iguais. Em
cm3, o volume do tronco do cone compreendi-
do entre esses dois planos é: Figura III

a. 24p a. 3V
b. 168p 4
c. 192p b. V
d. 504p 2
e. 648p
c. 5V
231. Udesc-SC 8
Considere um tronco de pirâmide regular,
cujas bases são quadrados com lados medindo d. 2V
4 cm e 1 cm. Se o volume deste tronco é 35 cm3, 3
então a altura da pirâmide que deu origem ao
e. 4 V
tronco é::
7
a. 5 cm
233. Fuvest-SP
b. 5 cm Um copo tem a forma de um cone com altura
3
8 cm e raio de base 3 cm. Queremos enchê-lo
20 com quantidades iguais de suco e de água.
c. cm
3 Para que isso seja possível, a altura x atingida
d. 20 cm pelo primeiro líquido colocado deve ser:
e. 30 cm 3

232. Cesgranrio-RJ
De um cone de centro da base O e de altura H 8
(Fig. I), obtém-se um tronco de cone de altura
H x
(Fig. II). Nesse tronco, faz-se um furo cônico
2
com vértice O, como indicado na figura III. Se o
volume do cone da figura I é V, então o volume 8
PV-13-14

do sólido da figura III é: a. cm


3
b. 6 cm
c. 4 cm
H

H d. 4 3 cm
2
O O e. 4 3 4 cm
Figura I Figura II

Figura III 121


Matemática Geometria de posição e métrica

234. Cesgranrio-RJ Sabendo-se que H = 4 cm, AB = 6 cm, BC = 3 cm


Um recipiente cônico, com altura 2 e raio da e a altura h = AE = 6 cm, determine:
base 1, contém água até a metade de sua altu- a. o volume da pirâmide EA´B´C´D´;
ra (figura I). Inverte-se a posição do recipiente, b. o volume do tronco de pirâmide.
como mostra a figura II. A distância do nível da
237. Fuvest-SP
água ao vértice, na situação da figura II, é:
E Um torneiro mecânico dispõe de uma peça de
metal maciça na forma de um cone circular
reto de 15 cm de altura e cuja base B tem raio
D´ C´ 8 cm (figura 1). Ele deverá furar o cone, a partir

h
B´ de sua base, usando uma broca, cujo eixo cen-
tral coincide com o eixo do cone. A broca per-
H D C furará a peça até atravessá-la completamente,
abrindo uma cavidade cilíndrica, de modo a
A B
obter-se o sólido da figura 2. Se a área da base
3 2
a. deste novo sólido é da área de B, determine
2 seu volume. 3
4 Figura I
b.
3
Figura II
c. 3
3
d. 7
3
e. 6
235. Fuvest-SP
Antes Depois
As bases de um tronco de cone circular reto
são círculos de raios 6 cm e 3 cm. Sabendo-se 238. Mackenzie-SP
que a área lateral do tronco é igual à soma das
áreas das bases, calcule: Uma xícara de chá tem a forma de um tronco
de cone reto, conforme a figura. Supondo
a. a altura do tronco do cone; π = 3, o volume máximo de líquido que ela
b. o volume do tronco do cone. pode conter é:
236. Vunesp 8 cm
A figura a seguir representa uma pirâmide com
vértice num ponto E. A base é um retângulo
PV-13-14

ABCD e a face EAB é um triângulo retângulo


com o ângulo reto no vértice A. A pirâmide
apresenta-se cortada por um plano paralelo à 6 cm
base, na altura H. Esse plano divide a pirâmide
em dois sólidos: uma pirâmide EA´B´C´D´ e um
tronco de pirâmide de altura H.
E
4 cm

a. 168 cm3
D´ C´
h
A´ b. 172 cm3

c. 166 bcm3
H D C d. 176 cm3
A B e. 164 cm3

122
Geometria de posição e métrica Matemática

239. Mackenzie-SP 240. Vunesp


Um frasco de perfume, que tem a forma de Um recipiente tampado, na forma de um cone
um tronco de cone circular reto de raios 1 cm circular reto de altura 18 cm e raio 6 cm, con-
e 3 cm, está totalmente cheio. Seu conteúdo é tém um líquido até a altura de 15 cm (figura1).
despejado em um recipiente que tem a forma A seguir, a posição do recipiente é invertida
de um cilindro circular reto de raio 4 cm, como (figura 2).
mostra a figura. Raio 6 cm Raio R

8 cm H
d Raio r
18 cm
4 cm
15 cm

Se d é a altura da parte não preenchida do re- Figura 1 Figura 2


cipiente cilíndrico e, adotando-se p = 3, o vaJor
de d é: Sendo R e r os raios mostrados nas figuras:
10
a. a. determine R e o volume do líquido no
6
cone, em cm3 (figura 1), como múltiplo
11 de π.
b.
6 b. dado que r = 3 91, determine a altura H
12 da parte sem líquido do cone na figura
c. 2. (Use a aproximação 3 91 ≅ 9 / 2 )
6
13
d.
6
14
e.
6
PV-13-14

123
Matemática R: Geometria de posição e métrica

GABARITO DOS EXERCÍCIOS PROPOSTOS


Capítulo 01 20. F, F, V, F, F.
01. Os pontos B e E 21. E
02. Os pontos D, E e F 22. A
03. O ponto B 23. D
04. Os ponto E  ) = 90° ⇒ BC ⊥ AC
24. a. m (ACB
05. A reta t 2
b. Temos: BC = AB · =4 2
06. 2
DAPC: PC2 = AP2 + AC2 ⇒ PC = 4 6
Há três situações:
 = BC  = 3 ⇒ CPB
 = 30°
• Infinitos planos quando os tg CPB ⇒ tg CPB
pontos são colineares; PC 3

• Apenas um plano quando os 25. C


pontos são coplanares e não co- 26. C
lineares; 27. Verdadeiro
• Quatro planos quando os pon- 28. B
tos não são coplanares. 29.
07.
V
Pelo postulado da determina-
ção do plano, sabemos que
três pontos não colineares de-
terminam um único plano, não A B
permitindo, assim, o balanço
da mesa.
08. E D C

09. E • AD é reversa com VC e com VB;


10. D • AB é reversa com VC e com VD.
• BC é reversa com VA e com VD.
11. E
• CD é reversa com VA e com VB.
12. V, F, V, F, F • AC é reversa com VB e com VD.
PV-13-14

13. F, F, V, F, F • BD é reversa com VA e com VC.


14. D 30.
15. D a. Duas retas são paralelas quando coincidem ou quando
são coplanares e não têm ponto em comum.
16. São 4 plano – ABC, ACG,
ACH, ACF. b.
17. E t
P
18. E
19. São três planos. O plano
da face ABEF do cubo, o plano
da face ABCD do cubo e o pla-
s
no ABGH que passa pelo cen- r
α
tro do sólido.

124
Geometria de posição e métrica R: Matemática

As retas t e r são coplanares. 56. D 74. k · (n – 2)


Logo, ocorre uma, e apenas 57. AD = 29 cm 75.
uma, das alternativas: t con-
58. E n0 de arestas pode ser calcula-
corre com r (I); ou t // r. (II). Se
t concorre com r, então exis- 59. D do de dois modos:
te um único ponto Q tal que 60. E p
t ∩ r= {Q}. 6 ( q+1) + 2
2 ou A = 3p + 4 q
Seja γ ∩ α = s, temos Q ∈ s. Capítulo 02 A=
2 2
Concluímos, então, que as re- 61. Assim: 6(q + 1) + p = 3p + 4q
tas r, s e t têm, em comum, o • Faces:
ponto Q, o que é um absurdo, 6q + 6 + p = 3p + 4q
triângulares: 4
pois, por hipótese, r e s são 2p – 2q = 6 ⇒ p – q = 3 (I)
paralelas distintas. Portanto, a quadrangulares:1
alternativa (I) é impossível, res- • Lados das faces: Mas SF = (v - 2) · 360°
tando apenas a alternativa (II), triangulares: 3 p · 180 + q · 360° = (8 – 2) · 360°
ou seja, t // r. De modo análogo, quadrangulares: 4 p + 2q = 12 (II)
prova-se que t // s (cqd).
• Arestas: 8 De (I) e (II) obtêm-se:
31. F, F, F, V, V
• Vértices das faces: p = 6 (triângulos) e q = 3 (qua-
32. V, F, V, V, F triangulares: 3 driláteros)
33. A quadrangulares: 4 76. C
34. B • Vértices do poliedro: 5 77. D
35. A • Soma dos ângulos:
78. B
36. E s = 360 (v – z) = 360° · 3 = 1080°
79. C
37. A • Euler:
80.
38. C v+f=A+2⇒5+5=8+2V
a. 12 vértices, 8 faces e
39. B 62. C 18 arestas
40. C 63. C b. 7a2 3
64. E
41. AR = 21 cm Capítulo 03
42. C 65. A
81. D
66. O número de arestas é 30. 6
43. E 82. senα =
67. B 3
44. B 83. A
68. O número de vértices é
PV-13-14

45. E 84. A
21.
46. C 85. A
69. D
47. D 86.
70. C
48. B a. a = 0,8 m
71. 5 faces
49. A b. V = 512 d
72. E
50. B 87. C
73. Número de arestas:
51. B 88. B
52. B 89. E
53. r = 2,26 cm A = 14,5 arestas 90. A
54. 5 cm Impossível, pois o número de 91. C
55. E arestas é um número inteiro. 92. 128 cm3

125
Matemática R: Geometria de posição e métrica

93. B 116. A
94. 64 cm 3 117. D
95. A 118. C
96. O paralelepípedo tem dimensões 1 m, 2 m e 4 m. 119.
13 27
97. E a. A = ,B= eC=9
2 2
98. C
b. A medida da diagao-
99. C
100. A nal é 61
cm
2
101. 864 cm2
102. V = 3 3 ⋅ a3 e A T = 3a2 ( 3 + 4) 120. H = 22 dam
Base: hexágono regular Capítulo 04
a2 · 3 3 2 121. D
Área = 6 · = ⋅a 3
4 2 122. B
123. B
Volume: 3 · a2 3 ⋅ 2a = 3a3 3
2 124. A
125. B
3
ÁreaTotal = 2 ⋅ ⋅ a2 ⋅ 3 + 6 ⋅ a ⋅ 2a = 3a2 3 + 12a2 = 3a2
2
( 3+4 ) 126. D
127. B
Volume = 3 3 ⋅ a3 e A Total = 3a2 ( 3+4 ) 128. A
129. A
103. B
130.
104. C
105. V = 18 3 cm3 3 2
a. H =
2
106.
a. V = 375 3 cm3
b. Asecção = 50 3 cm2 9b. 2
V= e A T = 9 + 18 3
107. D 2
108. D 131. A
PV-13-14

109. 132. D
a. V = 60 · A(a) cm3 133.
b. a = 90 e V = 75 litros a. h = 10 a – 25 cm
V = 75 000 cm3
V = 0,075 m3 b.
10
V= ⋅ (100 2a)2 · a – 25 cm3
110. V = 135 cm 3
3
111. C c. 25 < a < 50
112. B 134. VP = 40.000 m3
113. B VB = 2.000 m3
114. A VA = 3.600 m3
115. C 135. V = 81 cm3

126
Geometria de posição e métrica R: Matemática

136. C
a 6 2 2 b.
3 Capítulo 06
137. H = e senθ = 6 181. C
3 3
850 5λ 3 2 182. A
138. Vsólido = 156. VS =
3 12 183. D
139.
a3 157. V = 9 2 cm3 184. B
a. VBCGM =
6 185. D
5a2 158. B 186. A
b. VBCM = 159. V = 24
8 187. D
5a 41 160. E 188. B
c. d =
140. B
41 Capítulo 05 189. A
161. B 190. A
141. D
142. A 162. O volume do cilindro é 191. D
dado por V = pr2 192. C
143. E
V = πr2h(a + b) = 2Vtronco 193. E
144. C
πr2 (a + b) 194. E
145. D Vtronco =
2 195. E
146. E
196. 2.338 m2
147. B 163. A
197. E
148. A 164. D
198. C
149. A 165. E
199. C
150. V = 80 166. B
200. A  = 144 cm2
151. 167. C 
  
a. a = 45° V 144 cm2
168. = 40 cm3
b. A ABD =
2 6 103 ⋅ π Capítulo 07
8 169. r = 2m 201. E
3 170. A 202. C
c. VABCD =
16 203. D
171. D
152. 204. D
172. E
PV-13-14

a. A T = 18 + 2 3 cm2 205. E
173. C
20 3 206. E
b. V = cm 174. A
3 207. D
175. C 208. A
153. E 176. A 209. C
154. E
177. 950 caixinhas 210. B
155.
178. R = 3cm e r = 2 cm 211. E
a.

179. A T = (π + 2) m2 212. A
4
213. A
180. A 214. E

127
Matemática R: Geometria de posição e métrica

215. A x3 p 235.
b. V(x) = cm3 a. 4 cm
216. D 108
4π 3 b. 84π cm3
217. V = cm
3 227. D 236.
218. D 4
228. a) Vpirâmide = cm3
219. B 3
a. h = 2 (2 + 2) m 104
220. B b) Vtronco = cm3
b. VTronco = 3 (3 + 2) m3 3
Capítulo 08
221. E 640π 3
237. V2 =
229. E 9
222. C
230. B
223.C 238. A
231. C
224. A 239. B
225. D 232. A
240. E
226. 233. E
a. V = 16 π cm3 234. D

PV-13-14

128

Você também pode gostar